You are on page 1of 146

05.

Thermodynamics
3. The pressure of gas changes linearly with
First and Zeroth Law of volume from A to B as shown in figure. If no
(A) heat is supplied to or extracted from the gas
Thermodynamics then change in the internal energy of the gas
will be
1. 1g of a liquid is converted to vapour at 3 ×105
Pa pressure. If 10% of the heat supplied is used
for increasing the volume by 1600 cm3 during
this phase change, then the increase in internal
energy in the process will be:
(a) 4800 J (b) 4320 J
(c) 4.32 × 108 J (d) 432000 J
JEE Main-24.01.2023, Shift-I (a) 4.5 J (b) –4.5 J
Ans. (b) : Given, m = 1 g = 10–3 kg, P = 3 × 105 Pa (c) Zero (d) 6 J
Increase in volume (∆V) = 1600 cm3 = 1600 × 10–6 m3 JEE Main-31.01.2023, Shift-I
Work done, W = P∆V Ans. (a) : Given, P1 = 10 × 103 Pa, P2 = 50 × 103 Pa
W = 3 × 105 × 1600 × 10–6 V1 = 200 × 10–6 m3, V2 = 50 × 10–6 m3
W = 480 J No heat is supplied,
Now 10% of heat (Q) is used in work done, So, ∆Q = 0
W = 10% × Q = 0.1 Q From first law of thermodynamic,
Q = 10 W
∆Q = ∆ U + W
From first law of thermodynamics,
0=∆U+W
Heat supplied, Q = ∆U + W
Internal energy, ∆ U = – W
∆U = Q – W = 10 W – W
Hence work done by system,
∆U = 9 W = 9 × 480
W = Area under P-V curve
∆U = 4320 J
P +P
2. Given below are two statements: One is labelled W = 1 2 × ( V2 − V1 )
as Assertion A and the other is labelled as 2
10 × 103 + 50 × 103 (
× 50 × 10−6 − 200 × 10−6 )
Reason R.
W=
Assertion A : If dQ and dW represent the heat 2
supplied to the system and the work done on the W = –4.5 J
system respectively. Then according the first ∆U = –W = 4.5 J
law of thermodynamics dQ = dU – dW.
Reason R : First law of thermodynamics is 4. Five moles of an ideal monoatomic gas with an
based on law of conservation of energy. initial temperature of 150ºC expand and in the
process absorb 1500 J of heat and does 2500 J
In the light of the above statements, choose the
of work. The final temperature of the gas in ºC
correct answer form the options given below:
(a) Both A and R are correct but R is not is (ideal gas constant, R=8.314 JK -1mol -1 )
the correct explanation of A (a) 134 (b) 126
(b) A is correct but R is not correct (c) 144 (d) 166
(c) A is not correct but R is correct (e) 174
(d) Both A and R are correct and R is the Kerala CEE -2018
correct explanation of A Ans. (a) : Given, n = 5, T = 150°C, dQ = 1500J, W =
JEE Main-29.01.2023, Shift-I 2500J, R = 8.314JK.mol
Ans. (d) : First law of thermodynamic based on For monoatomic gas,
conservation of energy and it state, 5 3
CP = R , Cv = R and γ = 1.67
dQ = dU – dW 2 2
(When work done on the system) For ideal gas,
Hence, both A and R are correct and R is the correct dQ = dU + W
explanation of A. dU = dQ – W
574
nCv∆T = dQ – W Process CD
dQ – W 1500 – 2500 P = Constant
∆T = = W = PdV
nC v 5 × (3/ 2) × R
= 1 × 105 × ( 2 – 4) ×10–6
–1000 × 2
= = –16.04 = –2 × 10–1 J = – 0.2J
5 × 3 × 8.314 W = 0.2J
T2 – T1 = – 16.04
Process BC,
T2 = T1 – 16.04 = 150 – 16.04
V = constant
T2 = 133.96°C ≈ 134°C
W = PdV = 0
5. The Zeroth law of thermodynamics leads to the
W=0
concept of
Process DA,
(a) internal energy (b) heat content
(c) pressure (d) temperature V = constant
(e) work done W = PdV
Kerala CEE - 2016 W=0
Ans. (d) : Zeroth law of Thermodynamics:- When a Wnet = w ( A→B) + w ( C→D ) + w ( B→C ) + w ( D→A )
body A is in thermal equilibrium with a body B, and = 0.4 + (–0.2) + 0 + 0
also separately with a body C, then B and C will be in = 0.4 – 0.2 = 0.2J
thermal equilibrium with each other.
Wnet = 0.2J
Zeroth law of thermodynamics leads to the concept of
temperature. Hence, option (e) is the incorrect statement.
6. The p-V diagram of a gas undergoing in cyclic 7. First law of thermodynamics is a consequence
process (ABCDA) is shown in the graph where of the conservation of :
–2 3
p is in units of Nm and V in cm . Identify the (a) energy (b) charge
incorrect statement. (c) heat (d) all of these
AP EAMCET (23.09.2020) Shift-I
SRMJEEE - 2009, BCECE-2004
J&K CET- 2001, 1997
AIPMT-1988
Ans. (a) : The first law of thermodynamics is based
upon conservation of energy which means energy
neither created nor destroyed it will change from one
form into other form.
(a) 0.4 J of work is done by the gas from A to B. ∆Q = ∆U + ∆W
(b) 0.2 J of work is done on the gas from C and
8. Water falls from a height 500m. The rise in
D.
temperature of water at bottom if whole of the
(c) No work is done by the gas from B to C.
energy remains in water, will be : (specific heat
(d) Net work done by the gas in one cycle is 0.2
of water is c = 4.2 kJ/kg)
J.
(a) 0.230C (b) 1.160C
(e) Work is done by the gas in going from B and 0
C and on the gas from D to A. (c) 0.96 C (d) 1.020C
Kerala CEE - 2008 BCECE-2004
Ans. (e) : Ans. (b) : Given,
h = 500m and specific heat of water (c) = 4.2 kJ/kg =
4.2 × 103 J/kg
∆U = mc∆T
mgh = mc∆T
1 × 10 × 500 = 1 × 4.2 × 103(∆T)
500 × 10 50
∆T = = = 1.19°C
4.2 × 103 42
∆T = 1.19°C ≈ 1.16°C
Process AB, Constant pressure
P = constant 9. A constant volume gas thermometer works on
W = PdV (a) Archimedes’ principle (b) Pascal’s law
5 –6 (c) Boyles law (d) Charles’ law
= 2 × 10 × (4 – 2) × 10 J
W = 0.4J UPSEE - 2014
575
Ans. (d) : A constant volume gas thermometer works 1 
on the fact that the pressure of a gas depends on = ( VB – VA )  ( p A – p B ) + p B 
2 
temperature. It is work on the principle of Charles’s law
1
10. Six moles of O2 gas is heated from 20ºC to 35ºC W = (p A + p B ) ( VB – VA )
at constant volume. If specific heat capacity at 2
constant pressure is 8 cal/mol-K and R = 8.31 12. A system goes from A to B via two process I
J/mol-K, what is change in internal energy of and II as shown in the figure. If ∆ U1 and ∆ U2
gas? are the changes in internal energies in the
(a) 180 cal (b) 300 cal processes I and II respectively, then
(c) 360 cal (d) 540 cal
UPSEE - 2006
Ans. (d) : Given,
n = 6moles, ∆T = 35 – 20 = 15°C = 15K
Cp = 8cal/mol–K = 8 × 4.2 J/mol–K = 33.6J/mol.K
R = 8.31 J/mol–K
Mayor's relation can be written as (a) ∆ U1 = ∆ U2 (b) ∆ U1 > ∆ U2
∵ Cp – Cv = R (c) ∆ U1 < ∆ U2 (d) ∆ U1 ≠ ∆ U2
Cv = Cp – R AP EAMCET (23.09.2020) Shift-I
= 33.6 – 8.31 TS EAMCET (11.09.2020) Shift-I
= 25.29 J/mol.K TS-EAMCET-11.09.2020, Shift-1
∆U = n Cv∆T AIIMS-2014
∆U = 6 × 25.29 × 15 Ans. (a) :
∆U = 2276.1J
∆U = 541.93 cal ≈ 540cal
11. An ideal gas is taken from point A to the point
B, as shown in the p-V diagram, keeping the
temperature constant. The work done in the
process is : Internal energy is a state function not a path function.
So,
∆U1 = ∆U2
The change in internal energy does not depend upon
path followed by the process. It is only depends on
initial and final states.
13. 306 J of heat is required to raise the
(a) (pA – pB)(VB – VA)
temperature of 2 moles of an ideal gas at
1
(b) ( p B − pA )( VB + VA ) constant pressure from 25°C to 35°C. The
2 amount of heat required to raise the
1 temperature of the same gas through the same
(c) ( p B − pA )( VB − VA )
2 range at constant volume is
1 (a) 306 J (b) 153 J
(d) ( p A + p B )( VB − VA ) (c) 140 J (d) 80 J
2
UPSEE - 2005 AP EAMCET (21.09.2020) Shift-II
Ans. (d) : Ans. (c) : For constant pressure process,
∆Q = 306 J, n = 2moles
∆T = 35 –25 = 10°C = 10K
∆Q = n CP ∆T
306 = 2 × CP × 10
306
Cp = = 15.3J / mol.K
Constant temperature process, 20
Work done = Area of ABDECA ∴ CP – CV = R
= Area of ∆ABE + Area of rectangle BCED CV = CP – R (∴R = 8.314J/mol.K)
1 = 15.3 – 8.314
W = (p A – p B ) × ( VB – VA ) + p B × ( VB – VA )
2 CV = 6.986 J/mol.K

576
For constant volume process. 16. First law of thermodynamics is represented by
∆Q = n CV∆T (a) dQ = dU + dW (b) dU = dW + pdV
= 2 × 6.986 × 10 (c) dW = dQ + dU (d) dU = dQ + pdV
AP EAMCET (17.09.2020) Shift-II
∆Q = 139.72 J ≈ 140J
Ans. (a) : First law of thermodynamics is based upon
14. A gas is compressed from a volume of 2 m3 to a the conservation of energy,
volume of 1 m3 at a constant pressure of 100
dQ = dU + dW
Nm-2. Then it is heated at constant volume by
supplying 150J of energy. As a result, the 17. Work done
internal energy of the gas (a) Can only be positive
(a) increases by 250 J (b) decreases by 250J (b) Can only be negative
(c) decreases by 50J (d) increases by 50 J (c) Can either be positive or negative
(d) Cannot be assigned a sign
AP EAMCET (21.09.2020) Shift-I
AP EAMCET (17.09.2020) Shift-I
Ans. (a) :
Ans. (c) : Work done can either be positive or negative.
Work done on the system → negative
Work done by the system → positive
18. An insulated system contains 4 moles of an
ideal diatomic gas at temperature T. When a
heat Q is supplied to the gas, 2 moles of the gas
is dissociated into atoms and the temperature
remained constant. Then the relation between
Q and T is
(R = universal gas constant.)
For constant pressure process, (a) Q = RT (b) Q = 2RT
∆Q = 0 (c) Q = 3RT (d) Q = 4RT
W = PdV AP EAMCET (22.04.2019) Shift-I
= 100 (1 – 2) Ans. (a) : Internal energy for gas,
= –100J f
∆U =∆Q – W U = RT f → degrees of freedom
2
= 0–(–100) f = 3 for monatomic gas
∆U = 100J f = 5 for diatomic gas
(UB – UA) = 100J ……(i) The amount of heat ‘Q’ to be supplied
For constant volume process, Q = ∆U = Uf – Ui
W=0 5
∆U = ∆Q ⇒ ∆U = 150J Ui = 4 × RT, for 4 molecules of diatomic gas
2
UC – UB = 150 J …….(ii) Ui = 10RT
Adding, eq. (ii) and (i), We get For 4 moles of monoatomic gas + 2 moles of diatomic
UC – UA = 150 + 100 = 250J gas
∆U = 250J 3 5
Uf = 4 × RT + 2 × RT
So, internal energy of the gas is increases by 250J. 2 2
15. In a thermodynamic process pressure of a fixed Uf = 6RT + 5RT = 11RT
mass of a gas is changed in such a manner that Now, Q = ∆U = Uf – Ui = 11RT – 10RT = RT
the gas releases 30J of heat and 10 J of work Q = RT
was done on the gas. If the initial internal 19. Three moles of an ideal monotomic gas
energy of the gas was 10 J, then the final performs a cycle ABCDA as shown in the
internal energy will be figure. The temperatures of the gas at the
(a) 2J (b) - 18J states, A, B, C and D are 400 K, 800 K, 2400 K
(c) –10J (d) 58J and 1200 K, respectively. The work done by the
AP EAMCET (21.09.2020) Shift-I gas during this cycle is (R is universal gas
constant)
Ans. (c) : Given, Q = –30J, W = –10J, Ui = 10J
From first law of thermodynamics,
∆U = Q – W = –30 – (–10) = –30 + 10 = –20J
∆U = – 20J ⇒ Uf –Ui = –20
Uf = –20 + Ui = –20 + 10 = –10J
Uf = –10J
Then the final internal energy is –10J.
577
(a) 1200 R (b) 3600 R From the first law of thermodynamics
(c) 2400 R (d) 2000 R ∆Q = ∆U + W
AP EAMCET (22.04.2019) Shift-I 5J = WA→B + WB→C + WC→A
Ans. (c) : 5 = 10 + 0 + WC→A
WC→A = – 5J
Magnitude of work done = 5J
21. A graph drawn between absolute temperature
and volume of 3 moles of helium gas as shown
in the figure. If 5 cal of heat is used in the
process, then the work done is
Given,
TA = 400K, TB = 800K
TC = 2400K, TD = 1200K
n = 3moles
Process AB and process CD are at constant volume,
So, WAB=0 and WCD=0
W = PdV (∵ PV = nRT)
W = nRT
WTotal = W(A→B) + W(B→C) + W(C→D) + W(D→A)
WTotal = 0 + W(B→C) + 0 + W(D→A) =W(B→C) + W(D→A) (a) 21.0 J (b) 8.4 J
WTotal = 3R (TC –TB ) + 3R (TA – TD) (c) 12.6 J (d) 6.2 J
= 3R (2400 – 800) + 3R ( 400 – 1200) AP EAMCET (23.04.2018) Shift-2
= 3R × 1600 – 3R × 800 Ans. (b) :
= 4800R – 2400R = 2400R
WTotal = 2400R
20. An ideal gas is taken through the cycle A → B
→ C → A as shown in the figure. If the net heat
supplied to the gas in the cycle is 5 J. The
magnitude of work done during the process C
→ A is

Constant pressure process:-


n = 3 moles
Q = 5 × 4.2 = 21J (1 cal = 4.2 J)
Q = nCP∆T
(a) 5 J (b) 10 J
21
(c) 15 J (d) 20 J 21 = nCPT ⇒ ∆T =
AP EAMCET (20.04.2019) Shift-1 nC P
UP CPMT-2005 W = PdV = nR∆T [∵ PV = nRT]
Ans. (a) : Given, ∆Q = 5 J 21  21 
= nR × ∵ ∆T= 
nC P  nCp 
21R
W=
CP
 1
CP – CV = R ⇒ CP 1 –  = R
 γ
γR
CP =
Work done (A→B) γ –1
W(A→B) = PdV = 0 {V = Constant} ∵ Heliumis the 
Work done (B → C) 21R ( γ – 1)  
W(B→C) = PdV =10 × (2 – 1) = 10J
W=
γR  monoatomicgas 
For cyclic process (i.e. ∆U = 0), ∴ γ = 1.67 

578
21R ( γ – 1) 21(1.67 – 1) 23. Match the following lists.
W= = List-I List-II
γ 1.67
A Zeroth law of I Direction of flow of
21× 0.67
W= = 8.425J ⇒ W = 8.425J thermodynamics heat
1.67 B First law of II Work done is zero
22. An ideal monatomic gas is carried along the thermodynamics
cycle ABCDA as shown in the figure. The total
C Free expansion of a III Thermal
heat absorbed during this process is
gas equilibrium
D Second law of IV Law of conservation
thermodynamics of energy
The correct answer is
A B C D
(a) II IV III I
(b) III IV II I
(c) III I II IV
(d) I III IV II
(a) 10.5 p0V0 (b) 7.5 p0V0 AP EAMCET (22.04.2018) Shift-1
(c) 2.5 p0V0 (d) 1.5 p0V0 Ans. (b) : Zeroth law of thermodynamics →Thermal
AP EAMCET (20.04.2019) Shift-1 equilibrium
Ans. (a) : First law of thermodynamics → Law of conservation of
energy
Free expansion of a gas → Work done is zero
Second law of thermodynamics → Direction of flow of
heat.
24. A thermodynamic system is taken from state A
to B along ACB and is brought back to A along
BDA as shown in the PV diagram. The net
work done during the complete cycle is given
by the area
For monoatomic gas,
3 5
Cv = R, CP = R, γ = 1.67
2 2
Heat absorbed in the process (A→B),
QAB = nCv∆T
3
= n R∆T
2
3 3 (a) P1ACBP2P1 (b) ACBB'A'A
= ( V∆p ) = V0 ( 3p 0 – p0 ) (c) ACBDA (d) ADBB'A'A
2 2
3 BITSAT-2019
= V0 × 2p0 = 3p 0 V0 AIPMT-1992
2
QAB =3p0V0 Ans. (c) :
Heat absorbed in the process (B→C)
5 
Q( BC ) = nCP ∆T = n  R  ∆T
2 
5 5
= ( p∆V ) = 3p 0 ( 2V0 – V0 )
2 2
5 15
= × 3p0 V0 = p 0 V0
2 2 Work done during the path ACB
∵ Total heat absorbed during this process, WACB = Area under the curve ACB on V-axis (A1)
QTotal = QAB + QBC Work done during the path BDA
15 21 WBDA =Area under the curve BDA on V-axis (A2)
= 3p0V0 + p0 V0 = p 0 V0
2 2 ∴Work done during the complete cycle = WACB – WBDA
QTotal = 10.5p0V0 = A1 – A2 = Area under the curve ACBDA.
579
25. Which of the following process is possible (a) greater in process (ii) than in (i)
according to the first law of thermodynamics- (b) the least in process (ii)
(a) W > 0, Q < 0 and dU = 0 (c) the same in (i) and (iii)
(b) W > 0, Q < 0 and dU > 0 (d) less in (iii) than in (ii)
(c) W > 0, Q < 0 and dU < 0 BITSAT-2017, JCECE-2015
(d) W < 0, Q > 0 and dU < 0 Ans. (d) :
BITSAT-2012
Ans. (c) : According to the first law of thermodynamics
dQ = dU + W
dU = dQ – W
When, W > 0, Q < 0 then U < 0
26. A thermodynamical system is changed from
state (P1, V1) to (P2, V2) by two different According to first law of thermodynamics
process, the quantity which will remain same dQ = dU + dW
will be Work done during the process ACB = WACB
(a) ∆Q (b) ∆W
Heat absorbed by the gas,
(c) ∆Q + ∆W (d) ∆Q – ∆W
(dQ)ACB = dU + WACB
BITSAT-2013
Similarly,
Ans. (d) :
(dQ)ADB = dU and (dQ)AEB = dU + WAEB
Here, WACB is clock wise (positive) and WAEB is
anticlockwise (negative).
So,
(dQ)ACB > (dQ)ADB > (dQ)AEB
Hence option (d) is correct.
29. Figure below shows two paths that may be
Here, state is changing by two paths. taken by a gas to go from a state A to a state C
So, internal energy of the system remains constant.
∆Q = ∆U + ∆W
∆U = ∆Q – ∆W
27. In a thermodynamic process, the pressure of a
fixed mass of a gas is changed in such a manner
that the gas releases 20 J of heat and 8 J of
work is done on the gas. If the initial internal
energy of the gas was 30 J, then the final
internal energy will be
(a) 2 J (b) 42 J In process AB, 400 J of heat is added to the
(c) 18 J (d) 58 J system and in process BC, 100 J of heat is
BITSAT-2014 added to the system. The heat absorbed by the
system in the process AC will be
Ans. (c) : Given,
(a) 500 J (b) 460 J
dQ = –20J, dW = –8J
(c) 300 J (d) 380 J
Ui = 30J BITSAT-2016
We know,
AIIMS-2016
dQ = dU + dW
AIPMT-2015
dU = dQ – dW ⇒ Uf – Ui = dQ – dW
Ans. (b) :
Uf – 30 = –20 – (–8) = –20 + 8
Uf – 30 = –12 ⇒ Uf = 30 – 12
Uf = 18J
28. One mole of an ideal gas is taken from state A
to state B by three different processes, (i) ACB
(ii) ADB (iii) AEB as shown in the P-V
diagram. The heat absorbed by the gas is

Given,
dQAB = 400J
dQBC = 100J
580
(A→B) is isochoric process, 31. Calculate the heat required to increases the
dW = 0 temperature of 1 mole of one atomic gas from
dQ = dU + dW 0ºCto 150ºC, when no work is done. [Cp=2.5 R
dQ = dU = 400 J and R= 83 J mol−1 K−1]
dUAB = 400J (a) 867.5J (b) 186.5J
(B→C) is isobaric process, (c) 1867.5J (d) 86.7J
(dQ)BC = (dU)BC + (W)BC CG PET- 2014
100 = (dU)BC + PdV Ans. (c) : Given
= (dU)BC + [6 × 104 × (4 × 10–3–2 × 10–3)] n = 1 mole
100 = (dU)BC + (6 × 104 × 2 × 10–3) ∆T = T2 – T1 = (150 – 0) = 150°C
= (dU)BC + 12 ×10 R = 8.3J/K.mol, CP = 2.5R
(dU)BC = 100 –120 = –20J
CP – CV = R ⇒ CV = CP – R = 2.5R – R = 1.5R
∵ (dU) A→B→C = dUA→C
dUAB + (dU)BC = (dQ)AC –WAC .....(i) dQ = dU + dW ⇒ dQ = dU (∵ dW = 0)
WA→C = Area of ∆ACB + P∆VA dQ = nCV∆T
1  dQ = n1.5R∆T
=  × 2 × 10 –3 × 4 × 104  + ( 2 ×104 × 2 × 10 –3 )
 2  dQ = 1× 1.5 × 8.3 × 150 = 1867.5J
= 40 + 40 = 80J dQ = 1867.5J
Putting the value of WA→C in equation (i) dU+δW= 0 is valid for
32.
400 + (–20) = (dQ)AC – 80 (a) adiabatic process (b) isothermal process
400 – 20 + 80 = (dQ)AC (c) isobaric process (d) isochoric process
(dQ)AC = 460J CG PET- 2009
30. An ideal gas is taken through the cycle Ans. (a) : For adiabatic process,
A→B→C→A, as shown in the figure below. If ∆Q = 0
the net heat supplied to the gas is 5 J, then the
According to the first law of thermodynamics,
work done by the gas in the process C→A is
dQ = dU + δW
dU + δW = 0
33. Ideal gas is contained in a thermally insulated
and rigid container and it is heated through a
resistance 100 Ω by passing a current of 1 A for
five minutes, then change in internal energy of
the gas is
(a) 0 kJ (b) 30 kJ
(a) −5 J (b) −10 J (c) 10 kJ (d) 20 kJ
(c) −15 J (d) −20 J Manipal UGET-2014
CG PET -2018 Ans. (b) : Given, R = 100Ω, i = 1A
Ans. (a) : Given, Qnet = 5J t = 5min. = 5 × 60sec = 300sec.
When current is flowing through a wire then heat
produces,
Q = i2 Rt = 1 × 100 × 300 = 3 × 104 J
Q = 30 kJ
Therefore change in internal energy is 30 kJ.
34. 1 cm3 of water at its boiling point absorbs 540
cal of heat to becomes steam with a volume of
1671 cm3. If the atmospheric pressure =
WBC = PdV = 0 (∵ constant volume ) 1.013×105 N/m2 and the mechanical equivalent
WAB = 10 × (2 – 1) = 10J of heat = 4.19 J/cal, the energy spent in this
process in overcoming intermolecular process
∫ dQ = ∫ dW in overcoming intermolecular forces is
5 = WAB + WBC + WCA (a) 540 cal (b) 40 cal
WCA = 5 – WAB – WBC (c) 500 cal (d) zero
WCA = 5 – 10 – 0 = –5J Manipal UGET-2010
WCA = –5J CG PET-2006
581
Ans. (c) : Given, V1 = 1 cm3, V2 = 1671 cm3 37. An ideal gas in a cylinder is compressed
and Q = 540 cal adiabatically to one-third of its original
Atmospheric pressure = 1.013 × 105 N/m2 volume. A work of 45 J is done on the gas by
Mechanical equivalent (J) = 4.19 J/cal the process. The change in internal energy of
According to first law of thermodynamics the gas and the heat flowed into the gas,
∆Q = ∆U + ∆W respectively are
(a) 45 J and zero
∴ ∆U = ∆Q − ∆W
(b) –45 J and zero
P ( V2 − V1 ) (c) 45 J and heat flows out of the gas
∆U = 540 −
J (d) –45 J and heat flows into the gas
1.013 × 105 × (1671 − 1) × 10 −6  TS-EAMCET-04.05.2018, Shift-2
∆U = 540 − Ans. (a) : As we know that, in adiabatic process there
4.2
is,
∆U = 540 – 40 =500 cal no heat transfer, dQ = 0
35. The internal energy of the air, in a room of Given, W = –45J
volume Vt at temperature T and with outside dQ = dU + W
pressure p increasing linearly with time, varies
dU = dQ – W = 0 – (–45)
as
(a) increases linearly dU = 45J
(b) increases exponentially 38. An ideal gas has molar heat capacity Cv at
(c) decreases linearly constant volume. The gas undergoes a process
(d) remains constant where in the temperature changes as
TS-EAMCET-04.05.2018, Shift-1 T = T0 (1 + αV 2 ) , where, T and V are
Ans. (a) : Volume is constant, temperature and volume respectively, T0 and α
dW = pdV = 0 are positive constants. The molar heat capacity
According to 1st law of thermodynamics C of the gas is given as C = Cv + Rf (V), where,
dQ = dU + dW f(V) is a function of volume. The expression for
Hence, change in internal energy, f(V) is
dU = dQ and dQ = nCvdT αV 2 1 + αV 2
Then, dU ∝ dT (a) (b)
1 + αV 2 2αV 2
Now, pV = nRT
(c) αV 2 (1 + αV 2 )
1
dp dT (d)
.V = nR 2αV (1 + αV 2 )
2
dt dt
dp dT TS-EAMCET-04.05.2018, Shift-2

dt dt Ans. (b) : Given,
As p is linearly increasing with time, then temperature C = CV + Rf(V) …….(i)
also increases with time ∵ T = T0 (1 + αV2) = T0 + T0 αV2
Hence, internal energy (dU) is linearly increases. dT = 0 + T0 α. 2V . dV
36. A gas is at constant pressure 4 × 105 N/m2. dT = 2T0 α.dV
When a heat energy of 2000 J is supplied to the
dT
gas, its volume changes by 3 × 10–3 m3. What is = 2T0 Vα
the increase in its internal energy? dV
(a) 650 J (b) 900 J According to the first law of thermodynamics
(c) 800 J (d) 400 J dQ = dU + dW
TS-EAMCET.11.09.2020, Shift-2 nCdT = nCVdT + PdV
Ans. (c) :Given, P dV
C = CV + .....(ii)
P = 4 × 105 N/m2 n dT
Q = 2000 J ∴ PV = nRT
∆V = 3 × 10–3 m3 P R R
According to the Ist law of thermodynamics, = .T =  T0 C1 + αV 2 
n V V
dQ = dU + dW P R
= T0 (1 + αV 2 ) 
n V
dU = dQ – PdV
dU = 2000 – (4 × 105 × 3 × 10–3)
dU = 2000 – 1200  dT  P n
Putting this value   and in eq (ii) we get.
dU = 800J  dV  n

582
R 1 42. A box (thermally insulated) has two chambers
C = CV + T0 (1 + αV 2  × separated by a membrane. One of volume V
V 2T0 αV
contains an ideal gas at temperature T. The
1 + αV 2 V
C = CV + R other of volume is evacuated. If the
2αV 2 2
n
Compare with eq (i), we get, membrane breaks down, the gas temperature
1 + αV 2 will be
f (V) = (a) 2T/2 (b) 3T/2
2αV 2
(c) T (d) Data is incomplete
39. We consider a thermodynamic system. If ∆U COMEDK-2018, AMU-2009
represents the increase in its internal energy
and W the work done by the system, which of Ans. (c) : Isolated system that does not allow the
the following statements is true? transfer of either mass or energy to or from its
(a) ∆U = –W an adiabatic process surrounding.
(b) ∆U = W in an isothermal process dQ = 0
(c) ∆U = – W in an isothermal process Hence, container is thermally insulated dQ = 0 then dU
= 0 or no change in temperature of the gas.
(d) ∆U = W in an adiabatic process
T = constant
SCRA-2012, JCECE-2007
JIPMER-2008, MHT-CET 2007 43. One mole of an ideal gas requires 207 J heat to
Ans. (a) : From adiabatic process, Q = 0 raise the temperature by 10 K when heated at
constant pressure. If the same gas is heated at
dQ = dU + W
constant volume to raise the temperature by
dU = dQ – W = 0 – W
the same 10K, then the heat required is
dU = – W
(a) 198.7 J (b) 215.3J
From isothermal process, T = constant (c) 124J (d) 24J
dQ = dU + W COMEDK-2019
dQ = nCVdT + W AIPMT-1990
dQ = 0 + W
Ans. (c) : Given, n = 1 mole, P = const, ∆T = 10K
dQ = W and dU = 0
We know that, At constant pressure-
40. If temperature is increased by 1 K at constant
volume, then work done on the gas is : QP = nCP∆T
5 3 and at constant volume, QV = nCV∆T
(a) R (b) R Q V nC V ∆T C V
2 2 ∵ = =
1 Q P nC P ∆T C P
(c) Zero (d) R
2 3 5
For monatomic gas, CV = R and CP = R
JCECE-2006 2 2
Ans. (c) : Given that, at constant volume (V) = const. Q V 3/ 2R
dV = 0 =
Q P 5 / 2R
∵ Work done on a gas (W) = PdV
W=0 3 3
QV = Q P = × 207 = 124.2J
41. If R is universal gas constant, the amount of 5 5
heat needed to raise the temperature of 2 moles QV = 124.2J ≈124J
of an ideal monoatomic gas from 273 K to 373 44. Which of the following relations is correct?
K when no work is done is (a) E = mc (b) E = mc2
(a) 100R (b) 150R
(c) 300R (d) 500R mc2
(c) E = 2mc2 (d) E =
COMEDK 2016 4
Ans. (c) : Given, n = 2 moles, ∆T = (373 – 273) = 100K COMEDK 2017
3 5 Ans. (b): According to Einstein, energy and mass are
For monatomic gas, CV = R and C P = R , γ = 1.67
2 2 related by the relation
We know that, dQ = dU + W E = mc2
dQ = dU + 0 Where, c is the speed of light in vacuum.
dQ = dU = nCV∆T 45. Assertion: Zeroth law of thermodynamics
3 explain the concept of energy.
dQ = 2 × R ×100
2 Reason: Energy doesn't depends on
dQ = 300R temperature.
583
(a) If both Assertion and Reason are correct and 48. Two moles of a monoatomic ideal gas is
Reason is the correct explanation of confined in a container and is heated such that
Assertion. its temperature increases by 10ºC. The
(b) If both Assertion and Reason are correct, but approximate change in its internal energy is
Reason is not the correct explanation of (R= 8.31 J/mole-K)
Assertion. (a) +250 joules (b) +350 joules
(c) If Assertion is correct but Reason is incorrect. (c) –250 joules (d) +450 joules
(d) If both the Assertion and Reason are incorrect. AIIMS-2011
AIIMS-27.05.2018(M) Ans. (a) : Given, n = 2moles, ∆T = 10 ºC = 10 K
Ans. (d) : The concept of temperature comes from the 3
zeroth law of thermodynamic. R= 8.31 J/mol.K, For monoatomic gas, Cv = R
2
Zeroth law of thermodynamics states that if two bodies
are each in thermal equilibrium with some third body, ∵ dU = nCv ∆T
also they are in equilibrium with each other. 3
Energy is dependent on temperature. =n× R ∆T
2
46. Assertion: The heat supplied to a system is 3
always equal to the increase in its internal = 2 × × 8.31× 10
2
energy.
= 249.3J ≈ 250 J
Reason: When a system change from one
thermal equilibrium to another, some heat is 49. If ∆Q and ∆W represent the heat supplied to
absorbed by it. the system and the work done on the system
(a) If both Assertion and Reason are correct and respectively, then the first law of
thermodynamics can be written as
Reason is the correct explanation of
Assertion. (a) ∆Q = ∆U + ∆W (b) ∆Q = ∆U – ∆W
(b) If both Assertion and Reason are correct, but (c) ∆Q = ∆W – ∆U (d) ∆Q = –∆W – ∆U
Reason is not the correct explanation of AIIMS-2013
Assertion. Ans. (b) : Heat supplied to the system = + ∆Q
(c) If Assertion is correct but Reason is incorrect. Work done on the system = –∆W
(d) If both the Assertion and Reason are incorrect. According to the first law of thermodynamics–
AIIMS-2017 ∆Q = ∆U – ∆W
Ans. (d) : For first law of thermodynamics, 50. The latent heat of vaporization of water is 2240
dQ = dU + PdV J/g. If the work done in the process of
If heat is supplied in such a manner that volume does vaporization of 1g is 168 J, then increase in
not change ( dV = 0 ) then work done ( W ) Zero heat internal energy is
energy supplied to the system will increase internal (a) 1940 J (b) 2072 J
energy only. (c) 2240 J (d) 2408 J
dQ = PdV Punjab PET 1998, UPCPMT 2000
Thermal equilibrium is no flow of heat from one portion AIIMS-2002, JCECE-2011
of the system to another. Because if the temperature of Ans. (b) : Given that, LV = 2240 J/g, mass, m = 1 g,
the system remains constant. Hence, if both the ∆W = 168 J
Assertion and reason are in incorrect. ∵ ∆Q= mLV
47. The internal energy change in a system that has = 1 × 2240
a absorbed 2 kcal of heat and done 500 J of = 2240 J
work is According to the first law of thermodynamics–
(a) 6400 J (b) 5400 J
∆Q = ∆U + ∆W
(c) 7900 J (d) 8900 J
∆U = ∆Q – ∆W
BHU 2000, AIPMT 2009
= 2240 – 168 = 2072J
AIIMS-2017
51. A system is given 300 calories of heat and it
Ans. (c) : Given, W = 500J, dQ = 2 kcal = 2 × 103 × does 600 joules of work. How much does the
4.2 = 8.4 × 103J internal energy of the system change in this
We know that, process? (J = 4.18 Joules/cal)
dQ = dU + W (a) 654 Joule (b) 156.5 Joule
dU = dQ – W (c) –300 Joule (d) –528.2 Joule
dU = 8.4 × 103 – 500 MP PET 1991, Odisha JEE 2004
dU = 8400 – 500 BCECE 2006
dU = 7900J AIIMS-26.05.2019(E) Shift-2
584
Ans. (a) : Given, ∆Q = 300 cal = 300 × 4.18 J = 1254
J, ∆W = 600 J
According to first law of thermodynamics–
∆Q = ∆U + ∆W
∆U = ∆Q – ∆W (a) 10J (b) 70J
= 1254 – 600 (c) 84J (d) 134J
= 654 J BCECE-2005
52. An ideal gas is taken through a cyclic thermo Ans. (a) :
dynamical process through four steps. The
amounts of heat involved in these steps are Q1 =
5960J, Q2 = –5585 J, Q3 = –2980 J and Q4 =
3645 J, respectively. The corresponding works
involved are W1 = 2200 J, W2 = –825 J, W3 = –
1100J and W4 , respectively. The value of W4
is–
(a) 1315 J (b) 275 J Work done by the system ( A → B → C )( ∆W ) = 30 J
(c) 765 J (d) 675 J We know that, according to first law of
BCECE-2017 thermodynamic–
Ans. (c) : For cyclic thermodynamic process ∆Q = ∆U + ∆W
∆U = ∆Q – ∆W
∫Q= ∫W = 40 – 30 = 10 J
Q1 + Q 2 + Q3 + Q 4 = W1 + W2 + W3 + W4 56. The state of a thermodynamic system is
5960 + ( −5585 ) + ( −2980 ) + 3645 = 2200 + ( −825 ) + represented by
(a) pressure only
( −1100 ) + W4 (b) volume only
1040 = 275 – W4 (c) pressure, volume and temperature
W4 = 1040-275 = 765 J (d) number of moles
53. If 110 J of heat are added to a gaseous system, MHT-CET–2004, BCECE–2010
whose internal energy is 40J, then the amount Ans. (c) : A quantity of the matter or part of the space
of external work done is– which is under thermodynamic study is called as
(a) 40 J (b) 70 J system.
(c) 110 J (d) 150 J The state of the system can be represented by specifying
BCECE-2013, J&K-CET-2019 its pressure (P), volume (V) and temperature (T)
JIPMER-2004, AIPMT-1993 provided the system is in equilibrium.
Ans. (b) : Given, dQ = 110 J , U= 40 J 57. An ideal gas is subjected to cyclic process
We know that from the first law thermodynamic– involving four thermodynamic states, the
dQ = U + dW amounts of heat (Q) and work (W) involved in
dW = dQ – U each of these states are
= 110 – 40 Q1 = 6000J, Q2 = −5500 J; Q3 = −3000J;
=70 J Q4 = 3500J
54. If 150J of heat is added to a system and the W1 = 2500J; W2 = −1000J; W3= −1200J;
work done by the system is 110J, then change W4 = xJ.
in internal energy will be – The ratio of the net work done by the gas to the
(a) 40 J (b) 110 J total heat absorbed by the gas is η). The values
(c) 150 J (d) 260 J of x and η respectively are
BCECE-2007 (a) 500;7.5% (b) 700;10.5%
Ans. (a) : Given that, ∆Q = 150 J, work done by the (c) 1000;21% (d) 1500;15%
system (W) = 110 J VITEEE-2009
We know that, ∆Q = ∆U + ∆W AP EAMCET(Medical)-2006
∆U = ∆Q – ∆W Ans. (b) : For cyclic process of thermodynamic–
= 150 – 110 = 40 J
∫Q = ∫W
55. The P–V diagram of a system undergoing Q + Q + Q + Q = W + W + W + W
1 2 3 4 1 2 3 4
thermodynamic transformation is shown in
6000–5500–3000+3500 = 2500 – 1000 – 1200 + W4
figure. The work done by the system in going
from A → B → C is 30J, and 40 J heat is given 9500 – 8500 = 300 + W4
to the system. The change in internal energy 1000 – 300 = W4
between A and C is : W4 = 700 J

585
output
Efficiency (η) = × 100
input
W1 + W2 + W3 + W4
= ×100
Q1 + Q 4
2500 − 1000 − 1200 + 700
= ×100 ∆U = 0 = Q – W
6000 + 3500
1000 ∴Q=W
= × 100 Heat added to the system to do work
9500
V 
η = 10.5 % w = nRT ln  f 
58. Consider the following figure. Let ∆U1 and  Vi 
∆U2 be the change in internal energy in 60. What is the external work done
processes A and B respectively, ∆Q be the net (approximately) when 1 g of helium is heated
through 1 °C at constant pressure?
heat given to the system in the process A + B
(a) 1 J (b) 2 J
and ∆W be the net work done by the system in
(c) 4 J (d) 8J
the process A + B.
SCRA-2012
Ans. (b) : Given,
Temperature, T = 1ºC
Mass, m = 1g
No. of moles in 1g of Helium, n = 0.25 mole
Which of the following are correct? R = 8.314J mol–1 K–1
(1) ∆U1 + ∆U2 = 0 (2) ∆U1 – ∆U2 = 0 W= ∫ Pdv
(3) ∆Q – ∆W = 0 (4) ∆Q +∆W = 0 = nR × 1 = nR
Select the correct answer using the code given = 0.25 × 8.314
below: W ≈ 2J
(a) 1 and 3 (b) 1 and 4
61. Consider the following statements:
(c) 2 and 3 (d) 2 and 4 Statement I: A thermally insulated vessel
SCRA-2015 contains some water. If the vessel is shaken
Ans. (a) : The process that occurs through A and vigorously, the temperature of the water will
returns to the same state through B is cyclic being a rise.
state function, the net change in internal energy, (U) Statement II: In this process energy is given to
will be zero i.e. the water.
Which one of the following is correct with
∆U = ∆U1 +∆U2 = 0
reference to the above statements?
Using the first law of thermodynamics, we get (a) Both statements I and statement II are true
∆Q – ∆W =∆U and statement II is the correct explanation of
Q is the net heat given to the system in the process A + statement I
B and W is the net work done by the system in the (b) Both statements I and statement II are true
process A + B and statement II is not the correct explanation
So, ∆Q – ∆W = 0 of statement I
(c) Statement I is true and statement II is false
59. Heat supplied in a process is used completely to
(d) Both statements I and statement II are false
do work against the external surroundings.
SCRA-2010
Then, the process may be identified as which
one of the following? Ans. (a) : A thermally insulated vessel contains some
water. If the vessel is shaken vigorously, the temp of the
(a) Adiabatic (b) Isochoric water will rise.
(c) Isothermal (d) None of the above In this process energy is given to the water.
SCRA-2012 Hence, Both statement I and statement II are true and
Ans. (c) : In a thermodynamic process, heat supplied in statement II is the correct explanation of statement I.
a process is used completely in doing work against the In thermally insulated vessel, energy cannot be
external surroundings. Hence, this process may be transferred in from of heat, so by shaking energy is
designated as isothermal. transferred in the form of work.

586
62. What is the change in internal energy of one 64. A gas for which γ = 4/3 is heated at constant
mole of a gas, when volume changes from V to pressure. Fraction of heat supplied for doing
2V at constant pressure P? external work is
R (a) 1/2 (b) 1/3
(a) (b) PV
( γ –1) (c) 1/4 (d) 3/4
J&K CET- 2006
PV γPV
(c) (d) 4
( γ –1) ( γ –1) Ans. (c) : Given, γ =
3
Where γ is the ratio of specific heat of the gas at
At constant pressure,
constant pressure to that at constant volume
SCRA-2009 ∆Q = nCp∆T
Ans. (c) : Given that, and ∆W = PdV
Mole of gas, n = 1 = nR∆T
Initial volume = V Then,
Final volume = 2V Fraction of external work,
Pressure, P = Constant ∆W nR∆T R
= =
According to ideal gas equation, ∆Q nCp ∆T Cp
PV = nR∆T γR 
R 
=  Cp =
γ − 1
P(2V – V) = nR∆T γR 
PV = nR∆T .....(i) ( γ − 1)
As we know that,
γ −1
Cp =
γ= γ
Cv
4
Substract one in both side, −1
= 3
Cp 4
γ −1 = −1
Cv 3
Cp − Cv 1/ 3 1
γ −1 = = =
Cv 4/3 4
R 65. What is the amount of heat needed to raise the
= temperature of the gas in a cylinder of fixed
Cv
capacity (44.8 litres) that contains helium gas at
R standard temperature and pressure, by
Cv = .....(ii)
γ −1 15.0°C?
From equation (i) and (ii), (R = 8.31 J mol–1K–1)
PV (a) 374 J (b) 37.4 J
∆u = (c) 5.42 J (d) 54. 2 J
γ −1
63. The volume of one mole of an ideal gas changes J&K-CET-2018
from V to 2V at temperature 300 K. If R is Ans. (a) : Volume = 22.4l for 1 mole of gas for 44.8l of
universal gas constant, then work done in this volume we have 2 mole of He atom.
process is n=2
(a) 300Rln2 (b) 600Rln2 ∆Q = nCv∆T ∆T = 15ºC
(c) 300ln2 (d) 600ln2 For

{
J&K CET- 2010
Cv =
fR He → mono atomic
Ans. (a) : Given, V1 = V, V2 = 2V 2 f =3
We know that-:
3
Work done in an isothermal process Cv = R
2
V 
W = nRT ln  2  3
 V1  ∆Q = 2 × R × 15
2
 2V  = 45 × 8.31
∴ W = 1 × R × 300 ln  
 V  ∆Q = 373.95 J
W = 300 R ln2 ∆Q ≈ 374 J

587
66. No work is done on the system, but q amount of Ans. (b) : Work done per cycle is given by area of the
heat is taken out from the system and given to PV curve
the surroundings. Express the change in
internal energy (∆U) of this system and what
type of wall does the system have?
(a) ∆U = w, wall is adiabatic
(b) ∆U = q – w, closed system
(c) ∆U = –q thermally conducting walls
(d) ∆U = q thermally conducting walls
J&K-CET-2018
Ans. (c) : Given, 1
Work done on the system (dW) = 0 W = × (30 − 10)(8 − 2)
2
Heat out (dQ) = –q = 60 Jule
First law of thermodynamics- Cycle is repeated 100 times, then work done
dQ = dU + dW
= 60 × 100 = 6000 Jule
–q = dU + 0
The power generated is-
dU = –q
Work done
67. The zeroth law of thermodynamics for three P=
systems A, B and C in contact demands that Time taken
(a) A and B are in thermal equilibrium 6000
P=
(b) B and C are in thermal equilibrium 60
(c) A and C are in thermal equilibrium P = 100 Watt
(d) A, B and C are in thermal equilibrium 70. One mole of an ideal monoatomic gas is heated
J&K-CET-2013 at a constant pressure from 0°C to 100°C. Then
Ans. (d) : According to the zeroth law of the change in the internal energy of the gas is
thermodynamics when the thermodynamic systems A (Given, R = 8.32 J mol–1K–1)
and B are separately in a thermal equilibrium with a (a) 0.83 × 103 J (b) 4.6 × 103 J
3
third thermodynamic system C, then the systems, A, B (c) 2.08 × 10 J (d) 1.25 × 103 J
and C are in thermal equilibrium with each other. Ans. (d) : Given that
68. Which of the following laws of thermodynamics T1 = 0°C = 273K
forms the basis for the definition of
temperature? T2 = 100°C = 373K
(a) First law (b) Zeroth law For monoatomic n = 1
(c) Second law (d) Third law We know that
J&K-CET-2012 ∆U = nCV ∆T
Ans. (b) : Zeroth law of thermodynamics forms the 3
∆U = n R ∆T
basis of temperature. 2
Zeroth law of thermodynamics states that if two bodies 3
are each in thermal equilibrium with some third body, ∆U = 1× × 8.32 ( T2 − T1 )
2
also they are in equilibrium with each other.
3
69. A gas undergoes the cyclic process shown in = 1× × 8.32 ( 373 − 273)
figure. The cycle is repeated 100 times per 2
minute. The power generated is 3
= × 8.32 × 100
2
= 3 × 8.32 × 50
∆U= 1248 kJ
= 1.25×103 J
71. The pressure p, volume V and temperature T
AT − BT2
for a certain gas are related by p = ,
V
(a) 240W (b) 100W where A and B are constants. The work done
(c) 60W (d) 120W from T1 to T2 while the pressure remains
UPSEE - 2016 constant, is given by

588
(a) A (T2 – T1) + B (T22 − T12 ) Ans. (a) : From first law of thermodynamics,
Q=U+W
A(T2 − T1 ) B(T − T ) 2 2
(b) − 2 1
f
V2 − V1 V2 − V1 nC∆T = nR∆T + PdV
2
B 2 For constant volume [dV = 0]
(c) A (T2 – T1) – (T2 − T12 )
2 f
nC v ∆T = nR∆T + 0
A(T2 − T12 ) 2
(d)
V2 − V1 [∆W = PdV = 0]
WB JEE 2015 f
Cv = R .....(i)
AT − BT 2 2
Ans. (c) : p = Again from first law,
V f
PV = AT – BT2 nC∆T = nR∆T + PdV
Differentiation of both side, 2
For constant pressure,
P ∆ V = A∆T – 2BT
f
Integrating nC p ∆T = nR∆T + PdV .....(ii)
We get, 2
T2 T2 PV = nRT
W= ∫ PdV = A ∫ dT − B ∫ TdT
T1 T1
PdV = VdP = nR∆T
For constant pressure,
B PdV = nR∆T
= A (T2 – T1) – [(T2)2 – (T1)2] From equation (ii),
2
f
72. When a system is taken from state i to a state ƒ nC p ∆T = nR∆T + PdV
along path iaƒ, Q = 50J and W = 20J. Along 2
path ibƒ, Q = 35J. If W = – 13J for the curved f
Cp = R + R
return path ƒi Q for this path is 2
From equation (i),
Cp = C v + R
Cp – C v = R
74. Which one of the following is a thermodynamic
functions?
(a) Work (b) Internal energy
(c) Heat (d) Temperature
(a) –7J (b) 23J UP CPMT-2001
(c) –43J (d) 33J Ans. (b) : Thermodynamic function (point function)
UP CPMT-2011 represent the state of a system i.e. enthalpy, internal
Ans. (c) : Along path iaf: - energy, Helmholtz free energy, Gibbs free energy.
Q = 50 J Work and heat are path function.
W = 20 J 75. Match the columns for the first law of
Then, Q = W + ∆U thermodynamics.
∆U = Q – W A) Adiabatic processes I) W≠ 0, ∆Eint ≠ Q
Uf – Ui = 50 – 20 B) Constant – Volume II) Q =W; ∆Eint = 0
Uf – Ui = 30 J processes
Along path ibf :- C) Isothermal processes III) Q = 0; ∆Eint = –W
Q = W + ∆U D) Constant – pressure IV) W = 0; ∆Eint = Q
Q = W + (Ui – Uf) processes
Q = –13 + (–30) (W-Work done, Q – heat absorbed, ∆Eint –
Q = –43 J change of internal energy)
73. If CP and CV are molar heats at constant The correct match is
pressure and constant volume respectively and A B C D
R is gas constant for 1 mole, then the correct (a) III II IV I
relation is (b) III IV II I
(a) CP – CV = R (b) CP – CV < R (c) IV I II III
(c) CP – CV > R (d) CP – CV = 0 (d) I III IV II
UP CPMT-2002 TS EAMCET 31.07.2022, Shift-II
589
Ans. (b) : 80. In a thermodynamic process the pressure of a
(A) Adiabatic process (iii) Q = 0; ∆Eint = –W fixed mass of a gas is changed in such a manner
that the gas released 30 joules of heat and 18
(B) Constant volume (iv) W = 0; ∆Eint = Q joules of work was done on the gas. If the initial
Process internal energy of the gas was 60 joules, the
(C) Isothermal process (ii) Q =W; ∆Eint = 0 final internal energy will be:
(D) Constant pressure (i) W≠ 0, ∆Eint ≠ Q (a) 32 J (b) 48 J
process (c) 72 J (d) 96 J
AP EAMCET-24.08.2021, Shift-I
76. Two systems are in thermal equilibrium. The
quantity which is common for them is Ans. (b) : Given,
(a) heat (b) momentum dQ = –30 J, dW = –18J , Ui = 60J
By 1st law of thermodynamics–
(c) specific heat (d) temperature
dQ = dU + dW
SRMJEEE - 2015 dU = dQ – dW
Ans. (d) : Two systems are said to be in thermal dU = –30 + 18 = –12J
equilibrium if they are at same temperature. It means U f − U i = −12J
there is no heat flow between them whatever the
amount of heat a system contains. Final internal energy, U f = −12 + 60
77. Prevost's theory of heat exchange is not Uf = 48 J
applicable at temperature 81. When the state of a gas is adiabatically changed
(a) 0º R (b) 0º C from an equilibrium state A to another
equilibrium state B, the amount of work done
(c) 0 K (d) 0º F
on the system is 35 J. If the gas is taken from
SRMJEEE - 2016 state A to B via a process in which the net heat
Ans. (c) : Emission of thermal energy stops only at absorbed by the system is 12 cal, the net work
absolute zero when the molecular motion is stoped. done by the system in ––––(1cal = 4.2 J)
Hence Prevost's theory of heat exchange is not (a) 13.2 J (b) 15.4 J
applicable at 0 K (absolute zero temperature). (c) 12.6 J (d) 16.8 J
78. Work done in an isothermal change of a gas AP EAMCET-06.09.2021, Shift-I
depends: Ans. (b) : In the first - case adiabatic change
(a) Only on temperature ∆Q = 0 ∆W = –35 J
(b) Only on volume expansion ratio From the first law of thermodynamics
(c) On both the temperature and volume ∆Q = ∆U + ∆W .....(i)
expansion ratio 0 = ∆U – 35
(d) Only on initial and final pressure ∴ ∆U = 35J
MP PET -2013 In the second case ∆Q = 12 cal
= 12 × 4.2 J
Ans. (c) : In isothermal process work done- = 50.4J
V  ∆Wnet = ∆Q – ∆U
W= n RT log  2 
 V1  = 50.4 – 35
∆Wnet = 15.4 J
W= n RT log (r)
82. If system A is in thermal equilibrium with B
and B is separately in thermal equilibrium with
V2 C, then A and C are in thermal equilibrium.
Where r = = Volume expansion ratio
V1 From which thermodynamics law, does this
follow?
So, work done depends on temperature and volume
(a) Zeroth (b) First
expansion ratio.
(c) Second (d) Third
79. Amount of work which can be obtained from AP EAMCET-06.09.2021, Shift-I
200 cal heat, will be Ans. (a) : Zeroth law of thermodynamics:- The law
(a) 280 J (b) 800 J states that if system A is in thermal equilibrium with B
(c) 420 J (d) 840 J and B is separately in thermal equilibrium with C, then
JIPMER-2004 A and C are in thermal equilibrium.
Ans. (d) : Given,
W = 200 cal
∵ 1 cal = 4.2 J
W = 200 × 4.2 J ⇒ 840 J
So, W = 840 J This law is known as zeroth law of thermodynamic.

590
83. In a given process, for an ideal gas, From question given that, P = AV2
∆W = 0and ∆Q < 0. Then for the gas. ______ 105 = A × 32
(a) The temperature will decrease 105
(b) The volume will increase A =
9
(c) The pressure will remain constant We know,
(d) The temperature will increase V2
AP EAMCET-23.08.2021, Shift-II W = ∫ Pdv
V1
Ans. (a) : From the first law of thermodynamics 5
dQ = dU + dW W = ∫ AV 2dv
3
Given dw = 0 5
5 10
dQ = dU = ∫ × V 2 ×dv
3 9
also given dQ < 0
so dU < 0 105 5 2
9 ∫3
= V dv
mCv dT < 0
Hence, the temperature will decrease. 5
105  V 3 
84. The heat absorbed by the system is going =  
through the cyclic process shown in figure is 9  3 3
105  53 33 
=  − 
9 3 3
105 125 − 27 
W =
9  3 
So, W = 3.6 × 105 J
86. A sample of gas expands from an initial
(a) 30.4 J (b) 31.4 J pressure and volume of 10 Pa and 1.0 m3 to a
(c) 32.4 J (d) 33.4 J final volume of 2.0 m3, the pressure and volume
AP EAMCET (Medical)-05.10.2021, Shift-I are related by the equation p = av2, where a =
AP EAMCET-07.10.2020, Shift-I 10 N/m8. Find the work done by the gas during
Ans. (b) : In cyclic process, change in internal energy is the expansion.
zero. (a) 23 J (b) 18 J
∆U = 0 (c) 9 J (d) 43 J
From first law of thermodynamics AMU-2011
∆Q = ∆U + ∆W Ans. (a) : Given that,
∴∆Q = ∆W ......(1) Initial pressure p1 = 10 Pa
Initial volume v1 = 1.0 m3
∆W = Area under the curve on P-V graph = πR2 Final volume (v2) = 2.0 m3
∆W = π (100 ) × 10 × 10
2 3 –6 Given, p = av2
10 = a(1)2
= 3.14 × 104 × 10–3
a = 10
= 31.4 J
We know, work done dW = pdv
∴ From equation (1)
Integrating both side, w.r.t. volume
Heat absorbed →∆Q = 31.4J v2
85. A quantity of a substance in a closed system is ∫ dw = ∫v1 pdv
made to undergo a reversible process from an
2
initial volume of 3 m3 and initial pressure 105 = ∫ 10v 2 dv
2 3
N/m to a final volume of 5 m . If the pressure 1

is proportional to the square of the volume (i.e. 2


v3
P = AV2), the work done by the substance will = 10
be 3 1
(a) 3.6 × 10 J2
(b) 7.4 × 10 J
3
8 1
(c) 2.2 × 104 J (d) 3.6 × 105 J = 10× − 
 3 3 
AMU-2016
10×7
Ans. (d) : Given, =
Initial volume (V1) = 3m3 3
5
Initial pressure (P) = 10 N/m 2 = 23.34
Final volume (V2) = 5 m3 W ≈ 23 J

591
87. A thermodynamic system undergoes cyclic PV3 = Constant .......(1)
process ABCDA as shown in fig. The work On comparing the equation (1) by PVn = Const.
done by the system in the cycle is- we get, n = 3
R(n − γ )
Molar heat capacity (C) =
(n − 1)( γ − 1)
(3 − 5 / 3)
=R
(3 − 1)(5 / 3 − 1)
(4 / 3)
=R
2(2 / 3)
(a) PO VO (b) 2PO VO C=R
PO VO Now, ∆Q = nC∆T
(c) (d) Zero
2 = 1× R × 400
CG PET-22.05.2022 ∆Q = 400 R
AIPMT-2014
∆W = ∆Q − ∆U
Ans. (d) : Area under the P-V diagram gives the work
done– = 400 R − 600 R
∆W = −200 R
89. The height of a waterfall is 50 m. If g = 9.8 ms–2,
the difference between the temperature at the
top and the bottom of the waterfall is
(a) 1.17°C (b) 2.17°C
(c) 0.117°C (d) 1.43°C
WB JEE 2009
From the figure, Ans. (c) : Given,
1 h = 50 m
Work done in ∆EBC = − × Po × Vo ( − CCW) g = 9.8 m/s2
2
1 Specific heat capacity of water (s) = 4200J/kgoC
Work done in ∆EAD = Po Vo (+ CW) ∵ When water falls from height of 50 m than potential
2
So Net work = 0 energy converts into heat energy.
88. One mole of an ideal monoatomic gas expands ∴ Potential energy = Heat energy
till its temperature doubles under the process mgh = ms∆T
V2T= constant. If the initial temperature is 400 gh 9.8 × 50
K, the work done by the gas is ∆T = =
s 4200
(a) 400 R (b) 200 R
∆T = 0.117oC
(c) −200 R (d) indeterminate
CG PET-2021 90. The temperature and pressure at which all the
three phases of a substance coexist is called:
Ans. (c) : Give, Ti = 400 K
(a) Sublimation point (b) Regulation
Tf = 2Ti = 800 K (c) Triple point (d) Fusion point
∆T = Tf − Ti = 400 K AP EAMCET-25.09.2020, Shift-I
∆U = nC v ∆T Ans. (c): Triple point of a pure substance is the
combination of temperature and pressure where three
3
∵ For monoatomic gas C v = R phases (i.e. solid, liquid and vapour) exist in
2 thermodynamic equilibrium.
3 
∆U = 1 R  × 400 91. A 5 mole bubble of Helium gas (monoatomic) is
2  submerged to a certain depth in water which
∆U = 600 R undergoes an increase of 10ºC in its
Given, V2T= Constant temperature. How much energy is added to the
helium bubble as heat during the increase in its
  temperature and its consequent expansion?
 PV = nRT 
 PV    (Take R =8.31 J/mol-K)
∵ V2   = Constant  PV = 1× RT  (a) 625 J (b) 1040 J
 R   
PV (c) 1250 J (d) 2325 J
T = 
 R  AMU-2019
592
Ans. (b) : Given that, W = 1 × 105 × (2.001 – 0.001)
Increase temperature = 10o C = 2 × 105 = 200×103 J
Gas constant, R = 8.31 J/mol-K = 200 kJ
Heat Q is related to the temperature change ∆T specific Heat, Q = M lv
heat of the gas = 1.00 × 2000
Q = nCp∆T = 2000 kJ
Q = n (Cv + R) ∆T [∵ Cp – Cv = R] Change in internal Energy (U) = Q – W
= 2000–200
Q = 5 × 2.5 × 8.31 × 10 = 1800 kJ
Q = 1038.75 Joule
94. An amount of 700 J of heat is transferred to a
Q ≃ 1040 Joule. diatomic gas allowing it to expand with the
92. An office room contains about 2000 moles of pressure held constant. The work done on the
air. The change in the internal energy of this gas is
much air when it is cooled from 34ºC to 24ºC at (a) 200 J (b) 100 J
a constant pressure of 1.0 atm is (c) 300 J (d) 500 J
[Use γ air = 1.4 and Universal gas constant TS EAMCET 20.07.2022, Shift-I
= 8.314 J/mol K] Ans. (a) : Given that,
(a) – 1.9×105 J (b) +1.9×105 J Amount of heat = 700 J
(c) – 4.2×105 J (d) + 0.7×105 J Specific gas constant, γ = 1.4
TS EAMCET (Engg.)-2017 Form first law of thermodynamics.
Ans. (c) : Given that,
∆Q = ∆U + ∆W
Number of moles = 200
Change in temperature = (24 – 34)ºC = 10ºC Cp − Cv = R 
= nC ∆T + P. ∆V  Cp 
2 v
 = γ 
γ =1+
f  Cv 
2 n.R∆T
1.4 = 1 + = + P.∆V
f γ –1
2 P.∆V
= 0.4 ⇒ f = 5 ∆Q = + P. ∆V [ P.∆V = nR.∆T ]
f γ –1
f
U = n R∆T  1 
700 =  + 1 P.∆V
1.4 – 1 
2
5
= × 2000 × 8.314 × (24 – 34) 1 
2 700 =  + 1 P.∆V
= 5 × 1000 × 8.314 × (–10)  .4 
= – 415700 700 = [ 2.5 + 1] P.∆V
= – 4.157 × 105 700
= –4.2 × 105 Joule P. ∆V =
3.5
93. 1.00 kg of liquid water at 100 °C undergoes a P. ∆V = 200 J
phase change into steam at 100 °C at 1.0 atm
Work-done = 200 Joule [W = P.∆V]
(take it to be 1.00 × 105 Pa). The initial volume
of the liquid water was 1.00 × 10 m which is
–3 3 95. Assertion A: The zeroth law of
3
changed to 2.001 m of steam. Find the change thermodynamics leads to the concept of
in the internal energy of the system. temperature.
[Use heat of vaporization = 2000 kJ / kg] Reason R: The zeroth law states that two
(a) 1800 kJ (b) 200 kJ system in thermal equilibrium with a third
system are in thermal equilibrium with each
(c) 2000 kJ (d) 180 kJ
other.
TS EAMCET 18.07.2022, Shift-I
The correct option among the following is
Ans. (a) : Given that, (a) A is true, R is true and R is the correct
Mass of liquid = 1.00 kg explanation for A
Temperature of liquid = 100 º C (b) A is true, R is true but r is not the correct
Temperature of steam = 100 º C explanation for A
Initial volume, V = 1 × 10–3 m3 (c) A is true but R is false
Final volume = 2.001 m3 (d) A is false but R is true
Work, W = P. ∆V TS EAMCET 20.07.2022, Shift-I
593
Ans. (a) : Zeroth law of thermodynamic leads to the 98. Hydrogen gas is undergoing a process given by
concept of temperature. Zeroth law of thermodynamic pv2 = constant. The ratio of work done by the
states. If system A is in thermal equilibrium with B and gas to the change in its internal energy is_____
B is separately in thermal equilibrium with C, then A 2
and C are in thermal equilibrium. (a) (b) 0.4
3
96. Starting with the same initial conditions, and (c) 1.5 (d) – 0.4
ideal gas expands form volume V1 to V2 in AP EAMCET-24.04.2018, Shift-II
three different ways. The work done by the gas 2
is W1 if the process is purely isothermal W2 if Ans. (d) : pv = C
the process is purely adiabatic and W3 if the p = C
process is purely isobaric then, choose the v2
correct option. v2
Work done, W = ∫ p dv
(a) W1 < W2 < W3 (b) W2 < W3 < W1 v1

(c) W3 < W1 < W2 (d) W2 < W1 < W3 v 2 1


= C ∫ 2 dv
JEE Main-29.06.2022, Shift-II v1 v

v2
Ans. (d) :  –1 
= C 
v v1

2
p2 v p v2
=– 2
+ 1 1
v2 v1
= – p2v2 + p1v1
Work done, W = – n R T2 + n RT1
= nR (T2 – T1)
Work done = Area under the curve = nR∆T
In the P –V diagram, Area under the isobaric process is Internal energy (∆U) = nCV ∆T
greater than isothermal and isothermal is greater than 5
area under adiabatic process. ∆U = n R ∆T
2
Work done = W2 < W1 < W3
W –2
97. Certain quantity of heat is supplied to a =
∆U 5
monoatomic ideal gas which expands at
= – 0.4
constant pressure. The percentage of heat that
is used to do work by the gas is 99. If an automobile engine is known to develop
(a) 80% (b) 20% 1472 W at 60 rpm, how much torque does it
(c) 40% (d) 60% exert on the crank shaft?
(a) 234 N.m (b) 500 N.m
AP EAMCET-25.04.2018, Shift-I
(c) 1000 N.m (d) 134 N.m
Ans. (c) : Quantity of heat is supplied to θ monoatomic AP EAMCET-25.09.2020, Shift-I
ideal gas which expands at constant, pressure from first
Law of thermodynamics. Ans. (a) : Given that,
Developed power, P = 1472 W
∆Q = ∆U + ∆W
Speed, N = 60
m cp ∆t = m cv ∆t + ∆W
2πN
dW = m cp ∆t – mcv ∆t ω=
60
W mc p ∆t – mc v ∆t
= 2π× 60
Q mc p ∆t =
60
5 3 ω = 2π

= 2 2 P
As we know that, τ =
5/ 2 ω
1 1472
= =
2.5 2π
The percentage of heat is used to do work by the gas is 1472 × 7
W 1 =
× 100 = ×100 2 × 22
Q 2.5 = 234.181
= 40% = 234N. m

594
100. An insulated cylindrical vessel filled with an Ans. (c) : Given that,
insulated piston of negligible weight and C = constant pressure
P
negligible thickness at the mid point of the
vessel. The cylinder contains a gas at 0ºC. U = internal energy
When the gas is heated to 100ºC, the piston T = absolute temperature
moves through a length of 5cm. Length of the Internal energy, dU = m CV∆T
cylindrical vessel in cm is dU
(a) 13.65 (b) 27.3 = nCV
(c) 38.6 (d) 64.6 dT
AP EAMCET -2011 dU
= CV
Ans. (b) : Given that, dT
Initial temperature, t1 = 0 dU
Final temperature, t2 =100ºC = CP – R [CP–CV = R]
dT
Moving length of piston = 5 cm dU
+ R = CP
dT
103. Figure below shows a cycle process abcda. If
∆Q be the heat supplied to the system. ∆U be
the change in internal energy and ∆W be the
The above given process is isobaric work done by the system. Then which of the
V1 V2 following relation is correct?
=
T1 T2
L / 2 ( L / 2 + 5)
=
273 373
L L 
  373 =  + 5  273
2 2  (a) ∆Q – ∆W = 0 (b) ∆Q – ∆U = 0
L L  (c) ∆Q + ∆Q = 0 (d) ∆V + ∆U = 0
  373 =  × 273 + 5 × 273  AP EAMCET-06.09.2021, Shift-II
2 2 
L Ans. (a) : Given that,
100 = 273 × 5 ∆Q = Heat supplied
2
50L = 273 × 5 ∆U = Change in internal Energy
10L = 273 ∆W = Work done by the system
L = 27.3
101. A gas does 4.5 J of external work during
adiabatic expansion. If its temperature falls by
2K, then its internal energy will be
(a) increased by 4.5 J (b) decreased by 4.5 J
(c) decreased by 2.25 J (d) increased by 9.0 J
AP EAMCET -2015 In a cyclic process, the system returns to its initial state.
Ans. (b) : Given that, External work = 4.5 Joule Hence the change in the internal energy (du) is zero.
Decreased temperature T = 2 K From the first law of thermodynamics
During adiabatic process, no heat transfer, ∂Q = 0 ∆Q = ∆U + ∆W
From first law of thermodynamics ∆Q = 0 + ∆W
∂Q = ∂u + ∂w ∆Q – ∆W = 0
0=∂u+∂w
∂u = –∂w  5
104. Two moles of helium gas  γ =  at 27°C is
∂u = –4.5 J  3
The temperature falls so internal energy will decrease. expanded at constant pressures until its volume
102. Molar specific heat at constant pressure CP is is doubled. Then it undergoes an adiabatic
related to internal energy U and absolute changes until the temperatures returns to its
temperature T as CP = ________ initial value. The work done during adiabatic
U dU process is _____
(a) (b)
T dT (universal gas constant = 8.3 j mol-1K–1)
dU (a) 7470 J (b) 7070 J
(c) +R (d) U × T
dT (c) 4770 J (d) 4077 J
AP EAMCET-25.08.2021, Shift-I AP EAMCET-26.04.2017, Shift-II
595
5 106. If Q, E and W denote respectively the heat
Ans. (a) : Helium gas constant γ= added, change in internal energy and the work
3
done in a closed cycle process, then
Temperature, T = 27ºC
(a) W = 0 (b) Q = W = 0
= 273+27 = 300k
(c) E = 0 (d) Q = 0
Initial Volume, V1 = V
AIPMT-2008
Final volume V2 = 2V
Use of gas equation Ans. (c) : Given that,
P1 V1 = nRT1 Q = Heat energy
= 2 × R × 300 E = Internal energy
W = Work done
= 30 × 8.3
E is a state function. E is depends on state of a process
= 249
At constant pressure, P = c E = m Cv ∆T
V1 V2 ∆T = 0
= E = m Cv ∆T
T1 T2
E=0
V1 2V1
= 107. When a system is taken through the process a b
T1 T2 c, 80 cal of heat is absorbed by the system and
T2 = 2T1 60 cal of work is done by it if the system is
T2 = 600 K taken through the process adc, 30 cal of heat is
nR(T2 – T1 ) absorbed, then work done by the system is
Work done,W =
γ –1
2 × 8.3(600 – 300)
=
5
–1 s
3
3 × 8.3 × 2 × 300 (a) 10 cal (b) 20 cal
= (c) 30 cal (d) 4 cal
2
AP EMCET(Medical)-2011
= 3 × 8.3 × 300
= 7470 Joule Ans. (a) : Given that,
In process a b c,
105. An ideal gas undergoes a thermodynamic
Absorbed heat = 80 cal
process ABCDA as shown in the diagram.
Total work done during the cycle ABCDA is Work done = 60
Internal energy (∆U) = ∆Q – ∆W
= 80 – 60
= 20 cal
In process a d c,
Absorbed heat = 30 cal
Internal energy is equal in both process because
Internal energy is a point function.
(a) 4 PV (b) Zero dQ = dU + dW
(c) PV (d) 0.5 PV 30 = 20 + dW
AMU-2004 dW = 10 cal.
Ans. (c) : 108. 10 g of water at 70°C is mixed with 5 g of water
at 30°C. Find the temperature of the mixture in
equilibrium.
(a) 33.33°C (b) 56.67°C
(c) 77.66°C (d) 30°C
AP EMCET(Medical)-2008
Ans. (b) : Given that,
Mass of water = 10 gm
Consider the P – V diagram Temperature of water = 70 ºC
Work done in the process ABCD = Area of Rectangle Mass of water = 3 gm
= AB × BC Temperature of ice = 30ºC
= (2P – P) × (2V – V) Given heat = taken heat
= PV (mC∆T)given = (mC∆T)taken

596
10 × 1 (70 – t) = 5 × (t – 30) Ans. (d) : The first law of thermodynamics states that,
140 – 2t = t – 30 when a system undergoes a thermodynamic cycle, the
3t = 140 + 30 net heat supplied to the system from the surrounding is
170 equal to the net work done by the system on its
t= surrounding.
3
Therefore, heat is form of energy.
t = 56.67 ºC
∆Q = ∆U + ∆W
109. Consider the following two statements and
choose the correct answer. ∆Q = ∆U + P ∆V
A: If heat is added to a system its temperature Where, ∆Q = Heat transfer
must always increase. ∆U = Internal energy
B: If positive work is done by a system a ∆W = external work done.
thermodynamic process, its volume must 112. Ice starts forming in a lake with water at 0°C,
increase. when the atmospheric temperature is –10°C, If
(a) Both A and B are correct the time taken for 1 cm of ice to be formed is 7
(b) A is correct, but B is wrong hours, the time thken for the thickness of ice of
(c) B is correct, but A is wrong change from 1 cm to 2 cm is (in hours)
(d) Both A and B are wrong (a) 3.5 (b) 7
AP EAMCET (Medical)-2007 (c) 14 (d) 21
Ans. (c) : (A) In cyclic and isothermal process energy AP EAMCET-28.04.2017, Shift-I
supplied to a system does not change temperature of the Ans. (d): Heat transfer–
system.
dQ KA ( ∆T )
∆T = 0, ∆E = 0 =
dt dx
d ( mL ) KA ( ∆T )
=
dt dx
d ( ρ.VL ) KA ( ∆T )
=
dt dx
d ( ρ.A.xL ) KA ( ∆T )
(B) If gas expands then work done by a system of =
dt dx
thermodynamics process it volume must increase.
W = P ∆V ( ρ AxL )
dt = dx
110. Consider the two statements A and B : KA ( ∆T )
A : The neutral temperature does not depend ρAL
on temperature of cold junction. = ( xdx )
KA∆T
B : The inversion temperature does not
ρL
depend on temperature of cold junction. = ( xdx )
(a) both A and B are correct K∆T
(b) A is correct but B is wrong t ρL l
(c) A is wrong but B is correct ∫0 dt = K∆T ∫0 xdx
(d) both A and B are wrong l
ρL  x 2 
AP EAMCET (Medical)-1998 t=  
Ans. (b) : The temperature Tn of Hot Junction at which K∆T  2 0
thermo emf becomes maximum is called neutral
ρL  l 2 
temperature and it does not depend on the temperature t= × 
of the cold Junction statement A is correct. The K∆T  2 
temperature Ti for which thermo emf becomes zero and
 ρL 
changes direction is called temperature of inversion, it t ∝ l2  2K∆T = constant 
depends upon the temperature of cold junction
Therefore B is wrong. t1  l1 
2

111. First law of thermodynamics states that : ∴ =  


t 2  l2 
(a) system can do work
(b) system has temperature t1 = 7 hr, l1 = 1 cm
(c) system has pressure l2 = 2 cm
2
(d) heat is a form of energy  l2 
EAMCET-1998 t2 = t 1 ×  
AP EAMCET(Medical)-1998  l1 
597
2
Temperature
2 (a) (b)
= 7× 
1 0
(0, -20)
= 7×4 Heat supplied

= 28 hour.
Time taken from 1 to 2 cm (c) (d)
= t2 – t1
= 28 – 7
= 21 hours
UPSEE - 2014
(B) Change of State, Type of Ans. (a) :
System
113. The degrees of freedom of a molecule of a tri-
atomic gas are:
(a) 2 (b) 4
(c) 6 (d) 8
BCECE-2005
Ans. (c) : Degree of freedom for triatomic gas is 6.Ice at -20°C will convert to ice at 0°C and will absorb
114. A gas in an air tight container is heated from heat T must be a straight line. Ice at 0°C (at constant
250C to 900C. The density of gas will: temperature) by absorbing a heat Q = mLice .
(a) increase slightly • Water at 0°C will convert to water at 100°C and will
(b) remain the same
again absorb heat in accordance with formula dQ =
(c) increase considerably (d) decrease slightly
mcwater.
BCECE-2003
Ans. (b) : As we know that, • Water at 100°C will vaporize to steam.
mass (At constant temperature) by absorbing heat Q= mLsteam.
Density =
volume 117. Two blocks of ice when pressed together join to
Mass and volume of the gas will remain same as density form one block because
will also remain the same. (a) of heat produced during pressing
115. 1 cc of water becomes 1681 cc of steam when (b) of cold produced during pressing
boiled at a pressure of 105 Nm–2. The (c) melting point of ice decreases with increase
increasing internal energy of the system is of pressure
(Latent heat of steam is 540 cal g–1, 1 cal = 4.2 J)(d) melting point of ice increases with increase in
(a) 300 cal (b) 500 cal pressure
(c) 225 cal (d) 600 cal AP EAMCET (22.09.2020) Shift-I
(e) 1000 cal Ans. (c) : Two blocks of ice when pressed together join
Kerala CEE 2012
to form one block because melting point of ice
Ans. (b) : Given that, decreases with increase of pressure.
V1= 1cc , V2 =1681 cc 118. Which of the following parameters does not
∆V = V2 – V1 =1680 cc cm3 ( ) characterize the thermodynamic state of
5
P = 10 N/m 2 matter?
We know that, (a) Temperature (b) Pressure
dW = Pdv =10 × 1680 × 10 =168 J
5 –6 (c) Work (d) Volume
dQ = mL AP EAMCET (18.09.2020) Shift-I
= 1×540×4.2 = 2268 J Karnataka CET 2009
dQ= dU + dW UP CPMT-2008, UPSEE - 2007
dU =dQ – dW AIEEE 2003, UPCPMT 2001
= 2268 –168 Ans. (c) : Those physical quantities whose value
= 2100 J depend only upon the state of the system and does not
dU = 2100J = 500 cal depend upon the path by which this state has been
116. A block of ice at temperature – 20ºC is slowly attained are said to be state function. Here, work is
heated and converted to steam at 100ºC. Which depending upon the path of the system. Thus, work is
of the following diagram is most appropriate? not a thermodynamic state of function.
598
119. A metal rod of length 10 cm and area of cross- 121. Liquid is filled in a flask upto a certain point.
section 2.8×10–4 m2 is covered with non- When, the flask is heated, the level of the liquid
conducting substance. One end of it is (a) immediately starts increasing
maintained at 80oC, while the other end is put in (b) initially falls and then rises
ice at 0oC. It is found that 20 gm of ice melts in 5 (c) rises abruptly
min. The thermal conductivity of the metal in (d) falls abruptly
Js–1 m–1 K–1 is (Latent heat of ice is 80 cal g–1). Manipal UGET-2014
(a) 70 (b) 80 Ans. (b) : The liquid and the flask undergoes volume
(c) 90 (d) 100 expansion and the flask expands first therefore the level
AP EAMCET (20.04.2019) Shift-1 of the liquid initially falls and then rises.
Ans. (d) : Given, 122. 1 g of steam at 100ºC and equal mass of ice at
l = 10 cm = 0.1m, A = 2.8×10–4 m2 0ºC are mixed. The temperature of the mixture
in steady state will be (latent heat of steam =
Temperature of one end = 80°C 540 cal/g, latent heat of ice = 80 cal/g) :
Temperature of other end = 0°C (a) 50ºC (b) 100ºC
m = 20g (c) 67ºC (d) 33ºC
Time taken to melt ice = 5 min = 300 sec
JCECE-2005
s = 80 cal g–1
Ans. (b) : Heat required by 1gm of ice to melt and 1gm
m × s × 4.184 water so formed to come to 100°C,
Rate of heat flow (Q) =
t C = 1 cal/kgºC
20 × 80 × 4.184 Q1= mLice + mc ∆T
= = (1×80) + (1×1×100)
300
= 180 cal
= 22.315 J/s
Heat released by 1 gm steam at 100°C to form water at
KA.∆T 100°C.
Rate of heat flow in the rod, (∆Q) =
l Q2 = Vb × L v
K × 2.8 ×10 –4 × 80 = 1×540
22.315 = = 540 cal
0.1
K = 99.62 ∵ Q2 > Q1
K ≈ 100 Js–1m–1K–1 ∴ Only part of steam will condense and temperature
120. 1g of water at atmospheric pressure has will still be 100ºC.
volume of 1cc and when boiled it becomes 123. 100 g ice is mixed with 100 g of water at 100º C.
1681cc of steam. The heat of vaporisation of What will be the final temperature of the
water is 540cal/g. Then the change in its mixture?
internal energy in this process is (a) 10º C (b) 27º C
(a) 540 cal (b) 500 cal (c) 14º C (d) none of these
(c) 1681 cal (d) None of above JCECE-2003
Manipal UGET-2019 Ans. (a) : Heat given by water,
Ans. (b) : Given that, Qwater = ms ∆T
= 100 × 1× (100–T)
Initial volume of water, V1 = 1cm3
= 100 × (100–T) cal
Volume of steam, V2 = 1681cm3
Heat taken by ice,
Change in volume dV = V2 – V1
Qice = mL + ms ∆T
= (1681 – 1)cm3
= 100×80 + 100 × 1 (T–0)
= 1680×10–6m3 =8000 + 100 T
Patm = 1.01× 105Pa According to principal of calorimetric,
∴ dW =PdV =1.01×105×1680×10–6 Heat given = Heat taken
dW = 169.68 J 100 × (100 – T) = 8000 + 100T
∵ dQ = dU + dW 10000 – 100T = 8000 + 100T
dU = dQ – dW 2000 = 200T
= 2268 – dW 2000
T=
=2268 – 169.68 200
U = 2098.32 J = 499.6 cal ≈ 500 cal = 10°C

599
124. The latent heat of vaporisation of water is 2250 For isobaric process,
J/kg. If the work done in the process of T 
vaporisation of 1 kg is 168 J, then increase in Change in entropy (∆S) = ncp ln  2 
internal energy will be:  T1 
(a) 1904 J (b) 1984 J 7  600 
(c) 3202 J (d) 2082 J = 1 × × 8.314 ln  
2  300 
JCECE-2003
7
Ans. (d) : Given that, = × 8.314ln ( 2 )
Latent heat, Lv = 2250 J/kg, m = 1kg 2
∆W = 168 J 7
= × 8.314 × 0.6931
∆Q = mLv = 1 × 2250 = 2250 J 2
∆U = ∆Q – ∆W ∆S = 20.169J / K ≈ 20J/K
= 2250 –168 127. An ideal gas is taken through a cycle ABCA as
=2082 J shown below in P-V diagram. What is the work
125. Assertion: In an isolated system the entropy done during the cycle?
increases.
Reason: The processes in an isolated system are
adiabatic.
(a) If both Assertion and Reason are correct and
the Reason is a correct explanation of the
Assertion.
(b) If both Assertion and Reason are correct but (a) PV/2 (b) PV
Reason is not a correct explanation of the (c) 2 PV (d) Zero
Assertion. SCRA-2012
(c) If the Assertion is correct but Reason is Ans. (b) : Work done = Area enclosed by triangle
incorrect. ABCA
(d) If both the Assertion and Reason are
1
incorrect. = ( 2P − P )( 3V − V )
(e) If the Assertion is incorrect but the Reason is 2
correct. 1
= ( P )( 2V )
AIIMS-2006 2
Ans. (b) : In non-isolated system, heat may enter into or ∴ Work done = PV
escape from the system due to which entropy may
128. Which of the following is correct?
increase or decrease but for isolated system we do not
(a) Heat and work are intrinsic properties of a
consider exchange of heat so, in this case entropy will
system
always increase as the process is spontaneous.
(b) Neither heat nor work is intrinsic property of
An adiabatic process involves no exchange of heat. We
a system
also define isolated system as having no exchange of
heat with the surrounding so process in an isolated (c) Heat is an intrinsic property while work is not
system are adiabatic. (d) Work is an intrinsic property while heat is not
The two statements are independently correct but not SCRA-2011
co-related. Ans. (b) : Heat and work unlike temperature, pressure,
126. N2 gas is heated from 300K temperature to and volume are not intrinsic properties (does not
600K through as isobaric process. Then find depends on mass) of a system.
the change in the entropy of the gas. (Taken n = Heat flow is a result of a temperature difference
1 mole) between two bodies.
(a) 10 J/K (b) 20 J/K Hence, it is not a intrinsic property of substance.
(c) 30 J/K (d) 40 J/K 129. Helium gas expands at a constant pressure
AIIMS-25.05.2019(E) Shift-2 when 15 kJ of heat is supplied. If Cp : Cv is
Ans. (b) : Given that, equal to 5 : 3, what is the increase in the
T1 = 300K, T2 = 600 K , R = 8.314 J/mol.K internal energy?
n = 1 mole (a) 3 kJ (b) 6 kJ
7R 5 (c) 9 kJ (d) 15 kJ
N2 gas is diatomic gas, Cp = ,Cv = R
2 2 SCRA-2010

600
Ans. (c) : Given that, Because a gas molecules are in a state of constant
CP 5 random motion. They posses kinetic energy. When a
Q = 15 kJ, = gas is heated, the average kinetic energy per molecule
CV 3 of the gas increases. As a result, its temperature rises.
According to the first law of thermodynamics – The average kinetic energy per molecule is proportional
∆Q = ∆U + ∆W to the absolute temperature of the gas.
133. Which of the following is unique function of
∆Q = nCp∆T initial and final states?
∆U = nCv ∆T (a) dQ (b) dW
∆U CV (c) dU (d) ∆Q and ∆W
= J&K CET- 2004
Q CP
Ans. (c) : Change in internal energy dU of a
CV 3 thermodynamic system is a unique function of initial
∆U = × Q = × 15kJ
CP 5 and final state of the system. It does not matter, how
that state has been obtained.
∆U = 9kJ
134. If for a gas R/CV = 0.67, this gas is made up of
130. 1 g of ice is mixed with 1 g of steam. At thermal molecules which are
equilibrium, the temperature of the mixture is : (a) monoatomic
(a) 50°C (b) 0°C (b) diatomic
°
(c) 55 C (d) 100°C (c) polyatomic
Karnataka CET-2015 (d) mixture of diatomic and polyatomic
molecules
Ans. (d) : According to principle of calorimeter state
J&K CET- 2003
that total heat given by a hotter body is equal to the total
Ans. (a) : Given that, R/CV = 0.67
heat received by colder body.
We know that,
i.e. Heat required to melt 1g of ice 0ºC into 1g of water
R = Cp – Cv
at 0ºC is 80cal.
R Cp − C v
And Heat required to convert 1g of ice 0ºC into 1g of ∴ = = 0.67
water at 100ºC = 1 × 1× 100 = 100 cal Cv Cv
Heat required to condense 1g of steam = 1 × 540 cal = Cp
or − 1 = 0.670
540 cal clearly, whole of steam is not condensed so, Cv
temperature of the mixture is 100ºC. Cp
131. A cycle tyre bursts suddenly. What is the type = γ = 1.67
Cv
of this process ?
So, the gas is monoatomic.
(a) Isothermal (b) Adiabatic
135. A copper disc with a central hole is heated. The
(c) Isochoric (d) Isobaric
diameter of the hole
Karnataka CET-2014 (a) increases
Ans. (b) : When a tyre bursts suddenly energy is not (b) decreases
immediately transferred between the system and the (c) first decreases then increases
surrounding. So, the process is adiabatic. There sudden (d) remain unchanged
expansion of its air into the atmosphere is adiabatic and J&K CET- 2002
the tyre is cooled. In adiabatic process, heat neither Ans. (a) : A copper disc with a central hole is heated,
enters the system nor leaves the system. the diameter of the hole will increases. The copper has a
132. When the temperature of a gas is increased positive thermal expansion coefficient so the size of the
hole in the copper disc will increase.
(a) its molecular kinetic energy increases
136. In a cyclic process, the change in the internal
(b) molecular potential energy decreases and energy of a system over one complete cycle
molecular kinetic energy also decreases; total (a) depends on the path (b) is always negative
energy remaining constant (c) is always zero (d) is always positive
(c) molecular potential energy increases and
J&K-CET-2015
molecular kinetic energy decreases; total
Ans. (c) : In cyclic process, the change in internal
energy remaining constant energy of a system over one complete cycle is always
(d) its molecular potential energy increases zero.
J&K CET- 2009 In a cyclic process (∆T = 0 so, ∆U = 0) system remains
Ans. (a) : When the temperature of gas increased its the same after each cycle. Hence, the change in internal
molecular kinetic energy increases. energy is zero.

601
137. During the melting of a slab of ice at 273 K at 140. Heat energy absorbed by a system in going
atmospheric pressure through a cyclic process shown in figure. The
(a) Positive work is done by the ice water system work done during the process is:
on the atmosphere
(b) positive work is done on the ice water system
by the atmosphere
(c) internal energy of ice water system decreases
(d) None of the above
UP CPMT-2006
Ans. (b) : There is a decrease in volume during melting (a) 107π J (b) 104π J
of an ice slab at 273K. Therefore, negative work is done
(c) 103π J (d) 102π J
on the by ice water system atmosphere or positive work
is done on the ice-water system by the atmosphere. Assam CEE-2017
138. An ideal gas undergoes through an isochoric Ans. (d): ∆U = 0
process whereby its internal energy is ∆W = Area of loop
increased by ∆U. If the heat expelled (thrown = π×10×10×10–3×103 = 100π
∆U ∆Q = ∆U + ∆W = 102π
out) by the gas is ∆Q, then the ratio is
∆Q 141. The Entropy (S) of a black hole can be written
(a) 1.0 (b) 0.5 as S = βk B A, , where kB is the Boltzmann
(c) −1.0 (d) −2.0 constant and A is the area of the black hole.
TS EAMCET 31.07.2022, Shift-II Then β has dimension of
(a) L2 (b) M L2T–1
Ans. (c) : Internal energy increases means temperature –2
increases and work done be zero as it is isochoric (c) L (d) dimensionless
process. WB JEE 2022
Given heat expelled by the gas so Ans. (c): Entorpy, S = βKBA ………(i)
∆Q must be negative. Dimensional unit of entropy
∆Q = ∆U + ∆W S = [ML2T–2 K–1]
Boltzmann constant, KB = [ML2T–2K–1]
∆Q = − ∆U
Area (A) = (L2)
∆Q
∴ =− 1 From equation. (i)
∆U [ML2T–2K–1] = β[ML2T–2K–1] [L2]
139. 50 g ice at 0°C in insulator vessel, 50 g water of
 ML2 T −2 K −1   1 
100ºC is mixed in it, then final temperature of β =  4 −2 −1  =  2 
the mixture is (neglect the heat loss)  ML T K   L 
(a) 10ºC (b) 0º <<< Tm < 20ºC
(c) 20ºC (d) above 20ºC β =  L−2 
JIPMER-2014 Hence option (c) is correct.
Ans. (a) : Let the final temperature be T. 142. Assertion (A) : Heat and work are modes of
Heat released at 100ºC of water Q1 = m1s∆t energy transfer to a system resulting in change in
its internal energy.
Heat gained by 0ºC ice to convert at TºC of water
Reason (R): Heat and work in
Q 2 = m 2 L + m 2 s∆t
thermodynamics are state variable.
According to calorimetric principle, The correct option among the following is
Heat lost by 100ºC of water = Heat gain by 0ºC of ice (a) (A) is true, (R) is true and (R) is the correct
Q1 = Q2 explanation for (A)
m1s∆t = m 2 s∆t + m 2 L (b) (A) is true, (R) is true but (R) is not the
50 × 1 × (100–T) = 50 × 1 × (T – 0) + 50 × 80 correct explanation for (A)
(c) (A) is true but (R) is false
 cal 
s = 1 gm for water 
(d) (A) is false but (R) is true
  TS EAMCET 19.07.2022, Shift-II
 
 L = 80 cal Ans. (c) : Properties like temperature, pressure, volume,
for ice 
 gm  internal energy, entropy enthalpy etc. can be described
as state variable because the difference in their Values
100 – T = T + 80 between any two equilibrium state does not depend on
100 – 80 = 2T the path connecting the states. The thermodynamic
20 = 2T quantities of heat and work are not state variables
T = 10°C because they depend on the path connecting the states.

602
143. Consider a ball of mass 100 g attached to one 145. A monoatomic gas at pressure P and volume V
end of a spring (k = 800 N/m) and immersed in is suddenly compressed to one eighth of its
0.5 kg water. Assume the complete system is in original volume. The final pressure at constant
thermal equilibrium. The spring is now entropy will be:
stretched to 20 cm and the mass is released so (a) P (b) 8 P
that it vibrates up and down. Estimate the (c) 32 P (d) 64 P
change in temperature of water before the JEE Main-26.07.2022, Shift-I
vibrations stop. Ans. (c) : Given that,
(Specific heat of the material of the ball = 400 Initial Pressure = P1
J/kg K specific heat of water = 4200 J/kg/K)
–4 –3 Initial volume = V1
(a) 8.2 × 10 K (b) 10 K
–3 –3 V
(c) 7.5 × 10 K (d) 10 K Compressed Volume (V2) = 1
TS EAMCET (Medical)-02.05.2018, Shift-I 8
As we know that
Ans. (c) : Given that,
Mass of ball (mb) = 100 gm PVγ = c
γ γ
spring constant (k) = 800 N/m P2  V1  P  V 
Stretched in spring (x) = 20cm =  ⇒ 2 = 1 
P1  V2  P1  V2 / 8 
Specific heat of the material of ball (c) = 400 J/kg/K
Specific heat of water (cv) = 4200 J/kg/K 5
γ = for monoatomic gas.
1 3
Energy, E = kx 2 5/3
2 P2  8 
= 
1 P1  1 
E = × 800 × ( 20 ×10 –2 )
2

2
= ( 23 )
P2 5/3
= 16 Joule P1
In water, Q = m cv ∆T + mbc∆T
= 0.5 × 4200 × ∆T + 0.1 × 400 × ∆T P2 / P1 = 25
16 = 0.5 × 4200 × ∆T + 0.1 × 400 × ∆T P2
= 32
16 = 2140 ∆T P1
16 P2 = 32 P1
∆T =
2140 146. In anomalous behaviour water exhibits
= 0.00747 (a) An expansion when temperature is raised
= 7.5 × 10–3K from 0ºC to 4ºC
144. When a gas filled in a closed vessel is heated by (b) An expansion when temperature is decreased
raising the temperature by 1 °C, its pressure from 4ºC to 0ºC
increases by 0.4%. The initial temperature of (c) An expansion when temperature is raised
the gas is __________ K. form 0ºC onwards
JEE Main-25.06.2022, Shift-II (d) Contraction when temp is raised from 0ºC to
Ans. (250) : By the Gay-Lussac Law, 4ºC
Volume, c = Constant Assam CEE-2021
Ans. (d) : Anomalous behavior water,
P1 P2 P T
= ⇒ 1 = 1 (i) Liquids expands on heating and contract on cooling
T1 T2 P2 T2 but water exhibits an exceptional behavior in
P T + 273 temperature range 0 ºC to 4 ºC.
= (ii) When water is heated from 0 ºC to 4 ºC its volume
P + 0.004P T + 274
decreases continuously.
P T + 273
= (iii) At 4 ºC, it Volume is minimum.
1.004P T + 274 (iv) Upon heating further, it expands and its volume
1 T + 273 increases.
=
1.004 T + 274 (v) This behavior of water between temperatures
T + 274 = 1.004 T + 273 × 1.004 internal energy of 0 ºC to 4 ºC is called anomalous
T + 273 = 1.004 T + 274.092 behavior of water.
–(0.004)T = 1.092
1.092
T= −
0.004
= – 23 ºC
T = (273 – 23)K
T = 250 K
603
147. The coefficient of volume expansion of 150. A 10 H inductor carries of 20 A. If the energy
glycerine is 49×10–5K.The fractional change in stored in this inductor melts ice at 0 °C, then
its density for a 30º C rise in temperature is. the mass of the ice melted is [Latent heat of ice
(a) 4.17 × 10–2 (b) 1.47 × 10–2 = 2.26 × 103 J kg–1]
(c) 7.14 × 10–2 (d) 1.74 × 10–2 (a) 0.6 kg (b) 0.88 kg
Assam CEE-2021 (c) 0.98 kg (d) 1.4 kg
Ans. (b) : Given that, AP EAMCET (Medical)-24.04.2019, Shift-I
Coefficient of volume expansion of glycerin Ans. (b) : Given that,
= 49 × 10–5/k Inductance = 10 Henry
Rise in temperature, ∆t = 30º
As we know that,
1  ∆v 
γ=  
∆T  v 
∆V
= γ∆T
v
= (49 × 10–5) × 30 Current = 20 A
= 0.0147 Temperature ∆t = 0 ºC
= 1.47 × 10–2 Latent heat of fusion = 2.26 × 103 J/kg
Fractional change in density = fraction change in As we know that,
Volume. 1 2
148. The change in entropy when 1 g of ice 0 °C is Energy stores = 2 Li
heated to form water at 40 °C is This stored energy is fully used in melting the ice
(a) 0.42 cal /°C (b) 1.411 cal /°C Q = mLf
(c) 0.28 cal /°C (d) None of the above 1 2
Assam CEE-2014 Hence, Li = mLf
2
Ans. (a) : In this case heat is required to melt ice and to
rise temperature Entropy. 1 L2i
Mass of melted ice (m) =
mL T  2 Lf
∆S = + mc log  2 
10 × ( 20 )
2
T  T1 
=
1× 80 313 2.(2.26 × 103 )
= + 1× log × 2.303
273 273 10 × 400
= 0.28 + 0.1366 = 0.42 cal/ºC m=
4.52 × 103
149. If x gram steam at 100oC is mixed y gram ice at 4
0oC, final product is (x + y) gram water at =
100oC. The ratio y /x will be 4.52
= 0.8849
(a) 1 (b) 2
= 0.88 kg
(c) 3 (d) 4
Tripura-2020 151. A small electric heater is used to heat 200 g of
water. The time required to bring all this water
Ans. (c) : Given that,
from 400C to 1000C is 200 s. If specific heat of
Mass of steam = x the water is 42 Jkg–1K–1 then the power
Temperature ∆t = 100 ºC supplied by the heater is–
Heat given out when x grams of steam at 100 ºC is (a) 155 W (b) 310 W
converted to water of 100 ºC (c) 88 W (d) 252 W
(Heat)1 = 540 x ......(i) AP EAMCET-08.07.2022, Shift-II
Heat gained by ice at 0ºC
Ans. (d) : Given that,
(Heat)2 = 80y + y (100 – 0)
Mass of water = 200 gm
(Heat)2 = 180 y .......(ii)
Initial temperature of water = 40 ºC
According to Law of calorimeter
Final temperature = 100 ºC
(Heat)1 = (Heat)2
Specific heat of water = 42 J/kg k
540x = 180y
Time = 200 Sec.
y 540
= Heat energy, Q = ms ∆t
x 180 = 20 × 42 × (100 – 40)
y Q = 840 × 60
=3
x Q = 50400

604
Q KA∆T
Power, P= Total heat, Q = × time
t b
50400 0.01× 24 ×10−2 × 50
=
200 = × (10 × 60 × 60 )
4 × 10−2
= 252 watt
1200 × 0.01
152. 5 g of ice at – 30 ºC and 20 g of water at 35 ºC = × 36 ×103
are mixed together in a calorimeter. The final 4
temperature of the mixture is = 3×36×103
(Neglect heat capacity of the calorimeter = 108×103
-1 –1
specific hear capacity of ice = 0.5 cal g ºC Melt ice mass,
and latent heat of fusion of ice = 80 cal g-1 and Heat, Q = mL
specific hear capacity of water = 1 cal g-1 ºC-1)
Q 108 × 103
(a) 0 ºC (b) 4 ºC mass (m) = = = 0.322 kg
(c) 5 ºC (d) 9 ºC L 335 × 103
AP EAMCET-05.07.2022, Shift-I Now, the mass of remaining part of ice = (4 – 0.322)
Ans. (d): Consider the final temperature of mixture be = 3.6776
T = 3.678 kg.
Given, 154. The resistance of the platinum wire of the
Water mass, m = 20 g at 35oC platinum resistance thermometer at ice point is
Ice mass m = 5g at 30oC 5 Ωand at steam point is 5.25 Ω. This
Heat lost by water, Q1 = 20×1×(35–T) ……(i) thermometer is inserted in a hot liquid then its
Heat taken by ice, Q2 = mCi∆T + mL + mCwT resistance is 5.75 Ω. Calculate the temperature
= (5×0.5×30)+(5×80)+5×1×T of the hot liquid (inºC).
= 75+400+5T ………(ii) (a) 300 (b) 250
equation (i) = equation (ii) (c) 200 (d) 150
20×(35– T) = 475+5T AMU-2019
20×35–20T = 475 + 5T
Ans. (a): Given,
700–20T = 475+5T
225 = 25T Resistance platinum wire at steam point R100 = 5.25Ω
T=9 C o Resistance at ice point Ro = 5Ω
o
153. If a 4 kg of ice is inside a closed cubical Resistance of platinum when insert in hot liquid at t C =
thermacol box of side length 20 cm and wall 5.75 Ω
thickness 4 cm then the mass of the ice R100 = Ro (1+ α × 100)
remaining after 10 hours is nearly 5.25 Ω = 5 Ω (1+ α × 100)
(The out side temperature = 50 °C 1.05 − 1 0.05
coefficient of thermal conductivity of thermacol α= = = 5 × 10−4 / o C
100 100
= 0.01 Js m °C
–1 –1 –1
Resistance of hot liquid, Rt = Ro (1+ αt)
Latent heat of fusion of ice = 335 × 103 J kg–1)
(a) 3.678 kg (b) 6.378 kg 5.75 = 5 (1+5×10–4 t)
(c) 2.87 kg (d) 1.87 kg 1.15 − 1 0.15
t= −4
= = 300 o C
AP EAMCET-25.04.2017, Shift-II 5 × 10 5 ×10−4
Ans. (a): Given, Hence, temperature of hot liquid, t = 300oC
thermal conductivity, 155. An ice-cream has a marked value of 350 cal.
K = 0.01 Js–1m–1°C–1 Energy delivered by this ice-cream to the body
Latent heat = 335×103 J/kg is (The calorie in this case is a kilocaloric)
Mass of ice mi = 4 kg (a) 0.407 kWh (b) 3.50 kWh
Thickness of box, b = 4cm (c) 1.750 kWh (d) 7.00 kWh
= 4×10–2 m AMU-2003
side length of box, l = 20×10–2 m
2 Ans. (a): Amount of energy delivered by given ice
Total surface area of box = 6l
cream to the body = 350 cal.
= 6×(20×10–2)2
–2
= 24×10 m 2 To find kilowatt-hour of energy deliver to the body
Change in temperature We know that,
∆T = (50 C – 0)
o 4200
1 kcal = 103 calorie = 4200J = kWh
= 50oC 3.6 ×106

605
Now according to the question, Choose the correct answer from the options
350 × 4200 given below :
350 kcal = kWh (a) A-I, B-II, C-IV, D-III
3.6 × 106
= 0.407 kWh (b) A-II, B-I, C-IV, D-III
(c) A-I, B-II, C-III, D-IV
156. Which of the following is not thermo dynamical
(d) A-II, B-I, C-III, D-IV
function ?
(a) Enthalpy (b) Work done JEE Main-29.01.2023, Shift-I
(c) Gibb's energy (d) Internal energy Ans. (b) :
CG PET- 2010, AIPMT-1993 (A) Isothermal Process Temperature constant (T = C)
Ans. (b): Basic function of thermodynamic potential is (II) No change in internal
a quantity used to represent the state of a system. energy
Internal energy U, enthalpy H, Helmholtz free energy F, (B) Adiabatic Process No heat transfer (∆Q = 0)
and free energy G, are four thermodynamic functions. (I) Work done by the gas
Work done is a path function that means that the system decreases internal energy
depends on the path. (C) Isochoric Process Volume constant (V = C)
157. Which of the following processes is reversible? (IV) No work is done on or
(a) Transfer of heat by radiation by the gas
(b) Electrical heating of nichrome wire (D) Isobaric Process Pressure constant (P = C) (III)
(c) Transfer of heat by conduction The heat absorbed goes partly
(d) Isothermal compression to increase internal energy
AIPMT-2005 and partly to do work
Ans. (d): Slow isothermal process of an ideal gas is 160. Assertion : In adiabatic process, change in
reversible process as the reaction can be brought back to internal energy is equal to work done on gas.
its initial state, while other option are irreversible Reason: In adiabatic process, no heat exchange
process. with surrounding.
158. Why do two ice blocks join to form one block (a) If both Assertion and Reason are correct and
when pressed together? Reason is the correct explanation of
(a) Melting point of ice is lowered with increase Assertion.
in pressure (b) If both Assertion and Reason are correct, but
(b) Melting point of ice increases with increase Reason is not the correct explanation of
in pressure Assertion.
(c) Melting point of ice remains unchanged with (c) If Assertion is correct but Reason is incorrect.
increase in pressure (d) If both the Assertion and Reason are incorrect.
(d) Melting point of ice is 0oC AIIMS-27.05.2018(E)
NDA (I) 2008 Ans. (a) : For adiabatic process,
Ans. (a): When we press the two ice block together, ∆Q = 0
then melting point of top layer of ice decreases. This top dQ = dU + dW
layer melts and as soon as the pressure is released and
dU = –dW
after it the molten ice freezes back into ice due to the
low temperature of surroundings. So, Assertion and Reason are correct & reason is the
correct explanation of Assertion.
(C) Thermodynamics Process 161. An ideal gas goes through a process
A → B → C → A cycle. The process A → B is
159. Match List I with List II adiabatic. Calculate the work done in the
process A → B
List-I List-II
A. Isothermal I. Work done by the gas
Process decreases internal
energy
B. Adiabatic Process II. No change in internal
energy
C. Isochoric Process III. The heat absorbed goes
partly to increase
internal energy and
partly to do work
D. Isobaric Process IV. No work is done on or
by the gas

606
P0 V0 ( 21 γ − 2 )
163. A gas expands with temperature according to
(a) P0 V0 (b) the relation V=kT2/3. Calculate the work done
(1 − γ ) when temperature changes by 60 K.
P0 V0 ( 21 γ − 1) (a) 10 R (b) 30 R
(c) P0 V0 In ( 2 ) (d) (c) 40 R (d) 20 R
( γ − 1)
AP EAMCET-07.09.2021, Shift-I
TS EAMCET 06.08.2021, Shift-II
Ans. (c) : Given that V=KT 2/3
Ans. (b) : Given,
Work done is given as
P1 = 2Po, V1 = Vo and P2 = Po, V2 = V1
RT
Work done in adiabatic process W = ∫ PdV = ∫ dV
V
P1V1 − P2 V2
W= Since V= KT2/3
γ −1 –1
2P0 V0 − P0 V1 dV=2/3K T 3 dT
=
γ −1 2
–1
dV= KT 3 dT
P0 ( 2V0 − V1 ) 3
=
γ −1 Eliminating K, be find
dV 2 dT
=
 V 3 T
V 
P0 V0  2 − 1  Hence,
 V0 
= T2
2 RT 2
γ −1
γ
W= ∫3
T1
T
dT = R(T2 − T1 )
3
P  V 
Since  2  =  1  2
 P1   V2  = × R × 60
3
γ
 P0   V0  = 40 R
 = 
 2P0   V1  164. Match the following?
1 Column I Column II
V1
= (2) γ (i) Adiabatic expansion (a) No work done
V0
(ii) Isobaric expansion (b) Constant
 1
 internal energy
P0 V0  2 − 2 γ 
 
=   (iii) Isothermal expansion (c) Increase in
γ −1 internal energy
 1  (iv) Isochoric expansion (d) Decrease in
P0 V0  2 γ − 2  internal energy
 
∴Work done =   (a) (i–a), (ii–d), (iii–b), (iv–c)
1− γ (b) (i–d), (ii–a), (iii–c), (iv–b)
162. Which of the following statement is incorrect (c) (i–d), (ii–c), (iii–b), (iv–a)
(a) In an adiabatic process the system is insulated (d) (i–a), (ii–b), (iii–d), (iv–c)
from surroundings and the heat absorbed or AP EAMCET-07.09.2021, Shift-I
released is zero.
Ans. (c) :
(b) In an isochoric process volume is variable
Colum (i) Colum (ii)
(c) In isobaric process pressure is constant
(i) Adiabatic expansion Decrease internal energy
(d) In a cyclic process the system returns to
initial state. (ii) Isobaric expansion Increase internal energy
TS EAMCET (Medical) 09.08.2021, Shift-I (iii) Isothermal expansion Constant internal energy
Ans. (b) : In an isochoric Process Volume is constant. (iv) Isochoric expansion No work is done
165. A gaseous mixture consists of 16 g of helium
and 16 g of oxygen. The ratio Cp/CV of the
mixture is
(a) 1.4 (b) 1.54
(c) 1.59 (d) 1.62
V1 = V2 COMEDK 2020
607
16 Ans. (c) : According first law of thermodynamics,
Ans. (d): For 16 g of helium, n1= =4
4 Q = ∆U + W
16 1 or ∆U = Q − W
For 16 g of oxygen n2= =
32 2 According to given conditions
For mixture of gases, I. Q > 0 and W= 0
n1C V1 + n 2 C V2 or ∆U = Q
CV = Here Q > 0, therefore ∆U will be increase.
n1 + n 2
II. Q < 0 and W = 0
f ∆U = Q
Where, C V = R or
2 Here , Q < 0, therefore ∆U will be decrease.
3 III. W > 0 and Q = 0
For Helium, f = 3 then C V1 = R
2 or ∆U = -W
5
For oxygen, f = 5 then C V2 = R Here , W > 0 therefore ∆U will be decrease.
2 IV. W < 0 and Q = 0
3 1 5 or ∆U =0 –(–W)
4× R + × R
CV = 2 2 2 ∆U = W
4+
1 Here , W < 0, therefore ∆U will be increase.
2 Hence, condition I and IV will lead to an
5 increase in the internal energy of the system.
6R + R
4 Therefore, option (c) is correct.
=
9 167. Match the following
2 I Isothermal Process 1 ∆Q = 0
29R × 2 29R II Isobaric process 2 ∆V = 0
= =
9× 4 18 III Isochoric process 3 ∆P = 0
We know, IV Adiabatic process 4 ∆T = 0
R (a) I-4, II-3, III-2, IV-1 (b) I-3, II-2, III-1, IV-4
Cv =
γ −1 (c) I-1, II-2, III-3, IV-4 (d) I-4, II-2, II-3, IV-1
(e) I-1, II-4, III-2, IV-3
R
γ −1 = CG PET-2019, Kerala CEE - 2017
CV
Ans. (a) : Isothermal process → ∆T = 0
Cp R Isobaric process → ∆P = 0
γ= = +1
Cv Cv Isochoric process → ∆V = 0
R Adiabatic Process → ∆Q = 0
= +1
29 168. The P-V diagram of a gas system undergoing
R
18 cyclic process is shown here. The work done
= 1.62 during isobaric compression is

166. In a thermodynamic system, Q represents the


energy transferred to or from a system by heat
and W represents the energy transferred to or
from a system by work
I. Q > 0 and W = 0 II. Q < 0and W =0
III. W > 0 and Q = 0 IV. W < 0 and Q = 0
Which of the above will lead to an increase in
the internal energy of the system?
(a) I only (b) II only
(a) 100 J (b) 200 J
(c) I and IV only (d) II and III only
(c) 600 J (d) 500 J
(e) II and IV only (e) 400 J
Kerala CEE -2018 Kerala CEE- 2013
608
Ans. (e) : 171. During the adiabatic expansion of 2 mol of an
ideal gas, the increase internal energy was
found to be equal to (-200 J). The work done by
the gas during the process will be equal to
(a) 0 (b) 400 J
(c) -200 J (d) 200 J
UPSEE - 2018
Ans. (d) : For Adiabatic expansion ,
dQ = 0
dQ = dU + dW
For isobaric compression, (D → A) 0 = dU + dW
Work done = –PdV (Anti clock wise) dW = – dU
= –2 × 102 × (1 – 3) Given, n = 2 moles, dU = –200 J
= (–2) × 102 × (–2) dW = – dU
= 4× 102 = 400 J = –(–200)
169. Identify the wrong statement dW = 200 J
(a) For isothermal process, ∆T = 0 172. A system undergoes a reversible adiabatic
(b) For isochoric process, ∆V = 0 process. The entropy of the system
(c) For isobaric process, ∆P = 0 (a) remains constant
(d) For adiabatic process, ∆Q = 0 (b) may increase or may decrease
(e) For cyclic process, ∆W = 0 (c) increases
Kerala CEE 2012 (d) decreases
Ans. (e) : UPSEE - 2016
Ans. (a) : For Adiabatic process,
∆Q = 0
We know,
∆Q
∆S = (∵ ∆Q = 0)
T
For cyclic process , ∆S = 0 ,
After undergoing multiple processes in one S = const
cycle system returns back to its initial state. Hence, entropy of the system remains constant .
n n

∑ Qi = ∑ Wi
i =1 i =1
173. If one mole of a monoatomic gas  γ 1 =  is


5
3
Qnet = Wnet mixed with one mole of a diatomic gas
170. The thermodynamic process in which no work  7
is done on or by the gas is  γ 2 = 5  , the value of γ for the mixture is
 
(a) isothermal process (b) adiabatic process
(a) 3.07 (b) 1.5
(c) cyclic process (d) isobaric process
(e) isochoric process (c) 1.40 (d) 1.53
Kerala CEE - 2011 UPSEE - 2015, 2009
Ans. (e) : Ans. (b) : For monoatomic gas,
5 3 5
γ1 = , CV= R and CP = R
3 2 2
For diatomic gas,
7 5 7
γ2 = , CV = R and CP = R
5 2 2
 3   5 
1×  R  + 1 R 
n1CV1 + n 2 CV2  2  2 
For isochoric process, (CV)mix= =
n1 + n 2 1+1
V = constant.
W= PdV 8R
= = 2R
W=0 2× 2

609
(CV)mix =2R  1.1V 
Similarly (Cp) = 3R = 1×8.314×100 ln  
 V 
C 3R = 831.4 ln (1.1)
γmixture = P = = 1.5
CV 2R = 831.4 × 0.095
γmixture = 1.5 W = 78.983 J ≈ 79 J
174. In the indicator diagram shown in figure the Work done on the system is taken as negative so
net amount of work done is W = – 79J
176. When a system is taken from state i to state f
along the path iaf, it is found that Q = 50 cal
and W = 20 cal. Along the path ibf Q = 36 cal.
W along the path ibf is

(a) negative (b) positive


(c) zero (d) infinity
UPSEE - 2013 (a) 6 cal (b) 16 cal
Ans. (a) : (c) 66 cal (d) 14 cal
VITEEE-2019, UPSEE - 2010
Ans. (a) :

From the figure


For Path iaf,
Figure 1, area is less and it is clock wise hence work is
positive. ∆ Q = 50 cal , ∆W = 20 cal
Figure 2, area is greater and it is anti clock wise hence ∆Q = ∆U + ∆W
work is negative. 50 = ∆U + 20
So, ∆U = 30 cal
Wnet = W1 – W2 For path ibf ,
175. In an isothermal expansion the work done in ∆Q = 36 , ∆U = 30 cal
increasing the volume of one mole of a gas by ∆Q = ∆U + ∆W ⇒ 36 = 30 + ∆W
10% at a temperature of 100 K is = 6 cal
(a) 79 J (b) Zero
177. Which of the following p-V diagrams best
(c) 8.31J (d) –79J
represents an isothermal process?
UPSEE - 2011
Ans. (d) : For isothermal expansion process,
p
(a) (b)
V

(c) (d)

UPSEE - 2009
V1 = V, V 2 = V + 10 % V Ans. (c) : For isothermal process,
= 1.1 V
n = 1 mole
T2= T1 = 100 K (T = constant)
R = 8.314 J/m.K
V 
W = nRT ln  2 
 V1 

610
PV = constant (T = constant) 181. Initial pressure and volume of gas are p and V
P 1V 1 = P 2V 2 respectively. First it's volume is expanded to 4V
isothermally and then again it's volume makes
Graph between P & V is a part of rectangular to be V adiabatically. Then it's final pressure is
hyperbola. (γ=1.5)
178. During an isothermal expansion of an ideal (a) 8P (b) 4P
gas: (c) P (d) 2P
(a) its internal energy decreases CG PET- 2013
(b) its internal energy does not change Ans. (d) : Given, P1 = P, V1 = V, V2 = 4V, γ = 1.5
(c) the work done by the gas is equal to the For isothermal expansion,
quantity of heat absorbed by it
P 1V 1 = P 2V 2
(d) both (b) and (c) are correct
UPSEE - 2005 P V PV
P2 = 1 1 =
Ans. (d) : For isothermal process, no change in V2 4V
temperature. P
T = constant P2 = ….(i)
4
dU = 0
For adiabatic process, V3 = V
Q = +W γ γ
∵ All the heat added to the system is used to do work. P2 V2 = P3 V3
γ
179. Which of the following is not true about the  V2 
process? P3 = P2   [from equation(i)]
(a) For isothermal process, dT=0  V3 
γ
(b) For isobaric process, dP= 0 P  4V 
(c) For isochoric process, dE=0 =  
4 V 
(d) For adiabatic process, dQ= 0
P 1.5
CG PET 2019 = ( 4)
Ans. (c): For an isothermal process temperature remains 4
constant, therefore, dT=0. P 3
= ( 4) 2
For an isobaric process pressure remains constant, 4
therefore, dP=0. For an adiabatic process, there is no P
exchange of heat between system and the surrounding, = × 8 = 2P
therefore, dQ=0. Volume will be constant in isochoric 4
process. P3 = 2P
180. The initial pressure and volume of a gas are P1 182. Figure shows four, PV diagram for the given
and V1, the gas after expansion attains final sample of gas. In which case no exchange of
volume V2. Let W1, W2 and W3 are the heat occurs with the sample?
corresponding work done by the gas under
isothermal, adiabatic and isobaric processes
respectively then
(a) W1=W2=W3 (b) W2>W1>W3
(c) W3>W1>W2 (d) W3>W2>W1
CG PET -2016
Ans. (c) :

(a) P (b) Q
(c) R (d) S
CG PET- 2006
Ans. (d) :

Work done= Area under the curve


∴ Wisobaric is largest and Wadiabatic is smallest.
∴ W3 > W1 > W2

611
No exchange of heat, Where, Pi is initial pressure.
For adiabatic process, there is no exchange of heat take For container B,
place. ( Pi )B ( V ) + (1.5∆P ) × V = V
PV γ = constant
( Pi )B = −1.5∆P .....(ii)
constant
P= From Equation (i) and (ii), we get

(P ) 2
When volume is increased then pressure is decreased, ( Pi )A = i B = ( Pi )B
For expansion process, 1.5 3
2
Slope of S > Slope of R m A = mB [∵ P ∝ m]
∴ S is for an adiabatic process for which no exchange 3
of heat takes place. 3m A = 2m B
183. When a gas closed in a closed vessel was heated 185. During an experiment, an ideal gas is found to
so as to increase its temperature by 5ºC, its obey an additional law VP2 = constant. The gas
pressure was seen to have increased by 1%. is initially at temperature T and volume V. The
The initial temperature of the gas was nearly temperature of the gas will be following, when
(a) 500ºC (b) 273ºC it expands to a volume 2V?
(c) 227ºC (d) 150ºC
(a) 2T (b) 4T
CG PET- 2004
Ans. (c) : Given, (c) 6T (d) 5T
For closed vessel, Manipal UGET -2020
V1 = V2, ∆T = 5oC, P1 = P, ∆T = (T2 – T1) = 5oC ⇒ T2= Ans. (a) : Given,
5+T1, P2 = 1.01 P T1 = T, V1 = V, V2 = 2V
From ideal gas equation, From Boyle's law -
PV = nRT PV = Constant
P∝T VP2 = constant
P1 T1 PT ∵ PV = nRT 
= ⇒ T1 = 1 2  
P2 T2 P2 V1P12 = V2 P22
∴ P = nRT 
P × ( 5 + T1 )
5 + T1  V 
T1 = = 2 2
1.01P 1.01  nRT1   nRT2 
1.01T1 = 5 + T1 ⇒ 0.01T1 = 5 ⇒ T1 = 500 K
V1   = V2  
 V1   V2 
T1 = 500K = 227º C 1 T2
× T12 = 2
184. Two identical containers A and B with V1 V2
frictionless pistons contain the same ideal gas 2 2
at the same temperature and the same volume V2  T2  2V  T2 
=  ⇒ = 
V. The mass of the gas in A is MA and that in B V1  T1  V T
is MB. The gas in each container is now allowed 2
to expand isothermally to the same final T 
2= 2 
volume 2V. The changes in the pressure in A T
and B are to be found ∆p and 1.5∆p T2
respectively, then relation for masses will be = 2
T
(a) 4mA = 9mB (b) 2mA = 3mB
(c) 3mA = 2mB (d) 9mA = 4mB T2 = 2 T
Manipal UGET -2020 186. Which equation is valid for adiabatic process?
CG PET- 2015, EAMCET-2004 (a) TV γ −1 = constant (b) pV γ −1 = constant
Ans. (c) : For isothermal expansion of an ideal gas,
p γ −1
PV = constant (c) T γ V γ −1 = constant (d) γ −1 = constant
P∆V + V∆P = 0 T
For container A, Manipal UGET -2020
( Pi )A (V) + V(∆P) = 0 Ans. (a) : For an adiabatic process,
∆Q = 0
V∆P
( i )A
P = − = −∆ P .....(i) ∵ PVγ = constant ….(i)
V
612
From ideal gas equation, Ans. (a) :
PV = nRT Given, T1 = 250K, T2 = 251 K
nRT ∆T = T2 –T1 = 251–250 = 1K
or P=
V If mass and volume (v) are constant.
Putting the value of P in equation (i), we get P∝Τ
nRT γ P
V = constant = constant
V T
or TVγ – 1 = constant P1 P2
=
187. Internal energy of a gas remains unchanged in T1 T2
I. an isothermal process II. an adiabatic process P2 T2
III. a reversible process IV. a cyclic process =
Which of these are true? P1 T1
(a) I and IV (b) I, III and IV P2 − P1 T2 − T1
=
(c) III and IV (d) II and III P1 T1
Manipal UGET -2020
CG PET- 2014 P2 − P1 1
=
Ans. (a) : The internal energy of the system remains P1 250
constant in isothermal process. For isothermal process, P2 − P1
dU = 0 So, % increase in pressure = × 100
P1
For cyclic process, dU = 0 i.e. internal energy remains
unchanged 1
= × 100
188. A thermodynamic system is taken through the 250
cycle PQRSP process. The net work done by = 0.4%
the system is 190. Work done by air when it expands from 50 L to
150 L at a constant pressure of 2 atmosphere is
(a) 2×104 J (b) 2×100 J
(c) 2×1011 J (d) 2×1015 J
Manipal UGET-2012
Ans. (a) : Given,
V1 = 50L, V2 = 150L, dV = 100L= 100×10–3 m3
(a) 20 J (b) –20 J
Constant pressure (p) = 2 atm = 2×1.01×105N/m2
(c) 400 J (d) –374 J
Workdone = P dV
Manipal UGET-2019
= 2 × 1.01 × 105 × 100 × 10–3
Ans. (b) :
W = 2.02 × 104J ≈2 × 104 J
191. A gas is suddenly expanded such that its final
volume becomes 3 times its initial volume. If
the specific heat at constant volume of the gas
is 2R, then the ratio of initial to final pressures
is nearly equal to
(a) 5 (b) 6.5
Net work done by the system = Area of PQRS (c) 7 (d) 3.5
= (300–100)×(200–100) Manipal UGET-2010
= 200×100 Ans. (a) : Given,
= 2× 104 kPa – cm3 V1 = V, V2 =3V, CV =2 R, CP – CV = R,
= 2×10 × 10 × 10 Pa–m
4 3 –6 3 CP=R+CV=R +2R = 3R
Wnet = 20J C 3R
γ= P = = 1.5 C P = 3R
Here, direction of process is anticlockwise so work C V 2R
done will be negative i.e. ∆W = – 20 J A gas is suddenly expanded, so adiabatic process
189. A gas at the temperature 250 K is contained in PVγ=const
a closed vessel. If the gas is heated through 1K,
then percentage increase in its pressure will be P1V1γ = P2 V2γ
1.5
(a) 0.4% (b) 0.2%  P1   V2   3V 
1.5

(c) 0.1% (d) 0.8%    = =  


P V  V 
Manipal UGET-2018  2   1 
613
P1 = P2 × ( 3) Ans. (d) : We know that,
1.5
= P2 × 5.196
N2 gas is the diatomic gas,
P1 = 5.19 P2
7
P1 γ = = 1.4
= 5.19 ≈ 5 5
P2
T0 = 300 K, P1 = P0, P2 = 10 × P0
192. An ideal gas expands along the path AB as For adiabatic process,
shown in the p-V diagram. The work done is P1–γ Tγ = const
P11– γ T1γ = P21– γ T2γ
γ γ –1
 T2   P2 
  = 
 T1   P1 
γ –1 1.4–1
T2  P2  γ  10.P0  1.4
=  = 
T1  P1   Po 
0.4
T2 1
= (10 ) 1.4 = (100 ) 7 = 1.9
T1
(a) 4 J (b) 1.2 J T2 =1.9 × T1 = 1.9 × 300 = 570 K
(c) 2.4 J (d) None of these T2 = 570 K
Manipal UGET-2010
∴ Final gas temperature after compression = 570 K
Ans. (b) :
195. A diesel engine has a compression ratio of 20 :
1. If the initial pressure is 1×105 Pa and the
initial volume of the cylinder is 1×10-3 m 3 , then
how much work does the gas do during the
compression?
(Assume the process as adiabatic)
(Cv = 20.8 J/mol K, γair = 1.4, (20)1.4 = 66.3)
(a) – 880 J (b) – 579 J
Net workdone = Area under P–V graph (c) 220 J (d) 485 J
= Area of ABCD
TS-EAMCET-07.05.2018, Shift-1
1
= ( AD + BC ) × CD Ans. (b) : Given,
2 Initial volume ( Vi ) 20
1 Compression ratio = =
= ( 4 + 8) × 0.2 Final volume ( Vf ) 1
2
= 12 × 0.1 = 1.2 Joule Vi = 20×Vf …(i)
5 –3
193. Which of the following is not a thermodynamic P i = 1×10 Pa, V i = 1×10 m3
coordinate? Putting Vi value is equation (i) we get,
(a) Gas constant (R) (b) Pressure (P) 1
(c) Volume (V) (d) Temperature (T) 1×10–3 = 20 Vf ⇒ Vf = × 10 –3 m3
20
Manipal UGET-2009
Karnataka CET-2009 = 0.05 × 10 –3 m 3
AIIMS-2001 For adiabatic process,
γ γ γ
Ans. (a) : Pressure (P), Volume (V) and temperature PV = const, Pi Vi = Pf Vf
(T) are the thermodynamic coordinates whereas R is a γ

universal gas constant valued at 8.314 J mol−1K −1  Vi 


Pf = Pi  
194. One mole of nitrogen gas being initially at a  Vf 
temperature of To = 300 K is adiabatically = Pi × (20)1.4 = 105 × 66.289
compressed to increase its pressure 10 times.
Pf = 66.3 × 105 Pa
The final gas temperature after compression is
Pi Vi – Pf Vf (10 × 10 ) – ( 66.3 × 10 × 0.05 × 10 )
5 –3 5 –3
(Assume, nitrogen gas molecules as rigid
1/7
diatomic and 100 = 1.9) W = =
γ –1 1.4 –1
(a) 120 K (b) 750 K
(c) 650 K (d) 570 K 10 – 3.31 × 10
2 2
=
TS-EAMCET-03.05.2019, Shift-1 0.4

614
–2.315 × 102 p0  V 2 
3V0
p0
W=
0.4
= –5.7875 × 102 =   =
V0  2  V 2V0
(9V02 − V02 ) = 4p0 V0
W = – 578.7 J ≈ –579 J 0

In process II, there is no work done as V = constant.


196. Certain amount of heat supplied to an ideal gas
In process III, work done is given as
under isothermal condition will result in vf V0
(a) an increase in the internal energy of the gas W = p 0 ∫ dV = p0 ∫ dV
vi 3V0
(b) external work done and a change in
temperature = p0 ( V0 − 3V0 ) = −2p0 V0
(c) a rise in temperature So, total work done in entire process is
(d) external work done by the system WTotal = 4p0 V0 + ( −2p 0 V0 ) = 2p0 V0
TS-EAMCET.11.09.2020, Shift-2
AP EAMCET(Medical)-1997 199. Work done on heating one mole of monoatomic
Ans. (d) : For isothermal process, gas adiabatically through 20oC is W. Then, the
work done on heating 6 moles of rigid diatomic
T = const and ∆U = 0 gas through the same change in temperature.
From first law of thermodynamic (a) 9 W (b) 10 W
∆Q = ∆U + ∆W (c) 12 W (d) 8 W
∆Q = ∆W TS-EAMCET-10.09.2020, Shift-1
Hence, Heat supplied to an ideal gas under isothermal Ans. (b) : For adiabatic process–
condition will result in external work done by the
system. ∆Q = 0
5
197. What is the name of ideal-gas process in which n = 1 mole, γ = ( for monoatomic gas )
no heat is transferred? 3
(a) Isochoric (b) Isothermal 3
(c) Isobaric (d) Adiabatic C V = R, C p = 5 R, ∆T = 20o C
2 2
TS-EAMCET-14.09.2020, Shift-1
PV –P V nRT1 – nRT2
Ans. (d) : The ideal-gas process in which no heat is W= 1 1 2 2 =
transferred between a thermodynamic system and its γ –1 γ –1
surroundings is called as adiabatic process. nR 1× R × 20
W= ( T1 – T2 ) = 5
198. Five moles of an ideal gas has pressure p0, γ –1 –1
volume V0 and temperature T0. The gas is 3
expanded to volume 3V0 along a path, so that
60R
the pressure p is changed as function of volume W= = 30R
V as p = p0 (V/ V0). The pressure is then 2
reduced to p0 maintaining the volume constant. W = 30R ……(i)
The gas undergoes an isobaric compression till 7
the volume and temperature become V0 and T0, n = 6 moles, γ = ( For diatomic gas )
respectively. The total work done by the gas 5
during the entire process is ∆T = 20o C
(a) p0V0/3 (b) 3p0V0 nR∆T 6 × R × 20
(c) 5p0V0/3 (d) 2p0V0 W' = =
γ –1 7
–1
TS-EAMCET-10.09.2020, Shift-2
5
Ans. (d) : The given situation is shown in the following
6 × 5 × R × 20
diagram = = 300R
2
'
W = 300R = 30 R × 10
From equation (i)
W' = 10W
So, work done on 6 moles of rigid diatomic gas is 10
times of work done on 1 mole of monoatomic gas.
200. The P-V diagram shown below indicates two
paths along which a sample of gas can be taken
from state A to state B. The energy equal to 5
Work done in expansion (process - I) PV in the form of heat is required to be
transferred, if the Path-1 is chosen. How much
vf 3V0 p V
= ∫ pdV = ∫ 0
.dV energy in the form of heat should be
vi V0 V0 transferred, if Path-2 is chosen?

615
201. One mole of ideal gas goes through a process
PV3 = constant, where P and V are pressure
and volume, respectively. Let W be the work
done by the gas as its temperature is increased
by ∆T. The value of |W| is (R is the universal
gas constant.)
1
11 (a) R∆T (b) R∆T
(a) pV (b) 6 pV 3
2
R
9 (c) R3∆T (d)∆T
(c) pV (d) 7 pV 2
2 TS-EAMCET-03.05.2019, Shift-2
TS-EAMCET-09.09.2020, Shift-1
Ans. (d) : Given,
Ans. (a) : For process (1) n = 1, PV3 = const
For adiabatic process, ∆Q = 0
PVγ = const compare, PV3 = PVγ
γ=3
PV –P V nR ∆T
W= 1 1 2 2 =
γ –1 γ –1
1 × R ∆T R ∆T
Given, W= =
∆Q1 = 5PV 3 –1 2
Workdone = Area under P–V graph R∆T
W=
W = P dV 2
W = P(3V–V)=2PV 202. The phenomenon of sound propagation in air is
∵ ∆Q1 = ∆U + W (a) Isothermal process (b) Adiabatic process
5PV = ∆U + 2PV (c) Isobaric process (d) Isochoric process
∆U = 3PV JCECE-2018
∵ Process (2) and process (1) has same end points and BCECE-2014
internal energy is point function therefore internal Ans. (b) : Laplace gave the theory which pointed out
energy will be same. that sound travels through air under adiabatic
So, internal energy will be same conditions, because of following reasons-
(∆U)1 = (∆U)2 = 3PV (i) Air is poor conductor of heat.
For process (2) (ii) When sound travels through air, then temperature
Work done (W)2 = Area under curve in P–V graph rises in the regions.
(iii) Since, compressions and rarefactions are formed so
= Area of  ABED + Area of ∆ ABC
rapidly heat produce during compressions does not get
1 3  time to pass into the region of rarefactions, so as to
=  P × ( 3V – V )  +  ( 3V – V ) ×  P – P  
 2  2  equalise the temperature.
1 1 γp
= P × 2V + × 2V × P v=
2 2 ρ
PV 5 203. Six moles of an ideal gas performs a cycle
= 2PV + = PV
2 2 shown in figure. The temperatures are TA =
(∆Q)2 = (∆U)2 + (∆W)2 600 K, TB = 800K, TC = 2200 K and TD =
1200K. The work done by the cycle ABCDA is
5 11PV
= 3PV + PV =
2 2
11PV
∆Q 2 =
2
11PV (a) 20 kJ (b) 30 kJ
Hence, energy in the form of heat should be
2 (c) 40 kJ (d) 60 kJ
transferred if path 2 will choose. AP EAMCET-2010, JCECE-2016
616
Ans. (c) : P2 V2γ = P3 V3γ
γ γ
 V3   V1  P2 2P
  =  = =
 V2   V2  P3 0.75P
2  V1 
( 2) =
γ
∵ = 2
0.75  V 
(A → B) → Isochoric process 2

(B → C) → Isobaric process 2 × 100


=
(C→ D) → Isochoric process 75
(D → A) → Isobaric process 2× 4 8
( 2) =
γ
=
• Process (A → B) and (C → D) is isochoric process 3 3
So, workdone is zero. Taking log both side we get,
W = PdV γ log (2) = log (8) – log (3)
V = const, W = 0 log ( 8 ) – log ( 3) 0.903 – 0.477
Process (B → C), γ= =
log ( 2 ) 0.301
WB→C = PdV = nR∆T
= 6×R (2200–800) = 6R×1400 γ = 1.42
WB→C = 8400R 205. A monoatomic gas is suddenly compressed to
• Process (D→A), (1/8)th of its initial volume adiabatically. The
WB→C = nR∆T = 6 × R(600–1200) ratio of its final pressure of the initial pressure
is (Given the ratio of the specific heats of the
= 6R (–600) = –3600R
given gas to be 5/3)
Total work done,
(a) 32 (b) 40/3
Wnet = W(A→C) + W(B→C) + W(C→D) + W(D→A)
(c) 24/5 (d) 8
= 0 + 8400R + 0 + (–3600R)
JCECE-2009
= 4800R = 4800× 8.314 = 39907.2 J
Ans. (a) : For adiabatic process, ∆Q = 0
Wnet = 39907.2 J ≈ 40 KJ
PVγ = constant
204. A given mass of gas is compressed isothermally
until its pressure is doubled. It is then allowed P1V1γ = P2 V2γ
γ
to expand adiabatically until its original P V 
volume is restored and its pressure is then So, 2 =  1 
found to be 0.75 of its initial pressure. The ratio P1  V2 
of the specific heats of the gas is approximately V
(a) 1.20 (b) 1.41 Given, V1 = V, V2 =
8
(c) 1.67 (d) 1.83 γ
EAMCET-2006, JCECE-2009 P  V  5
∴ 2 = = ( 8) 3 = 32
Ans. (b) : P1  V 
 8
P2
= 32
P1
206. If energy is supplied to a gas isochorically,
increase in internal energy is dU then :
Process (1→2), compressed isothermal process, (a) dQ = dU +dW (b) dQ = dU – dW
P 1 V1 = P 2 V2 (c) dQ = dU (d) dQ = – dU
P2 V1 JCECE-2006
= Ans. (c) : In isochoric process volume is constant work
P1 V2
done = PdV,
2P2 V1
= ( Given :P2 = 2P1 ) Where, P is pressure.
P1 V2 dV is the change in volume
V1 For an isochoric process dV = 0
=2 According to the first law of thermodynamics
V2
dQ = dU + dW [∵ dW = 0]
• Expansion adiabatic process, (2→3)
V3 = V1 ∴ dQ = dU

617
207. If the volume of gas is changed from V1 to V2 210. What happens to the internal energy of a gas
isothermally, then work done is : during isothermal expansion ?
V1 V2 (a) Internal energy will decrease
(a) RT ℓn (b) RT ℓn
V2 V1 (b) Internal energy will increase
V T (c) Internal energy will become zero
(c) R(T2 − T1 )ℓn 2 (d) R(V2 − V1 )ℓn 2 (d) Internal energy will remain same
V1 T1
JCECE-2003
JCECE-2006
Ans. (d) : In isothermal expansion temperature remains
Ans. (b) constant. Therefore, internal energy which is a function
of temperature will remain constant.
211. The work done in taking an ideal gas through
one cycle of operation as shown in the indicator
diagram below.

For isothermal process,


T = constant, dT = 0
V 
W = nRT ln  2 
 V1 
n=1
V  (a) 10–5 J (b) 10–3 J
W = RT ln  2  [∵ dT = 0, Q = W]
 V1  (c) 10–2 J (d) 10 J
COMEDK2012
As the heat added to the system is used to do work.
Ans (d) : Work done on the system, W
208. The work done by a gas is maximum when it
= Area under the P – V diagram
expands :
= (P2 – P1)(V2 – V1)
(a) isothermally (b) adiabatically
Here,
(c) isochorically (d) isobarically
P1= 2N m–2, V1 = 1m3
JCECE-2005 P2= 4 N m–2, V2=6 m3
Ans. (d) : Work done, W = PdV. W= (4–2)(6 –1) = 10 J
P-V diagram of different process is shown below: 212. An ideal gas is expanded from volume V1 to
volume V2, in three different ways:
isothermally, adiabatically and isobarically. If
W1, W2 and W3 are respectively the works done
in the three processes, then
(a) W3 > W2 > W1 (b) W1 < W2 < W3
(c) W3 > W1 > W2 (d) W1 < W2 > W3
As work done is the product of pressure and volume
changes, so work done is maximum for that process COMEDK 2011
whose curve surrounds maximum area. Ans. (a) : Adiabatic, isobaric and isothermal process
As shown, area of isobaric curve is maximum hence, between V1 and V2 are shown in the figure.
work done in isobaric expansion is maximum.
209. When you make ice cubes, entropy of water:
(a) does not change
(b) increases
(c) decreases
(d) may either increase or decrease depending on
the process used
JCECE-2007, UPSEE - 2006
JCECE-2003, UP CPMT-2006
Ans. (c) : The entropy function gives us a numerical
We can see in figure,
measure of the irreversibility of a given process i.e. it is
a measure of disorder of a system. During formation of Aisobaric > Aisothermal > Aadiabatic
ice cubes orderedness increase i.e. disorderness Hence, W3 > W2 > W1 [A= Area, W = Work done]
decrease hence, entropy decreases. A∝W

618
213. In an isochoric process 215. Which of the following relations does not give
(a) Work done is constant. the equation of an adiabatic process where
(b) Volume changes, work done remains same terms have their usual meaning?
(c) Volume remains constant and no work is (a) P1– γ T γ = constant (b) PV γ = constant
done by the system γ
(c) PV = constant
–1
(d) P γ T1– γ = constant
(d) Both volume and work done changes
COMEDK 2015 COMEDK 2013
Ans. (c&d) : We know in adiabatic process
Ans. (c) :
PVγ = constant ...(i)
From ideal gas equation,
PV = nRT
nRT
P=
V
Putting value of P in equation (i),
PVγ = constant
nRT γ
V = constant
V
For an isochoric process the area under the P-V curve is
nRTVγ–1 = constant
zero. Since, no work is done.
Thermodynamic process in which the volume remains ∴ TVγ–1 = constant
constant is called isochoric process. PV = nRT
V = constant, P = variable quantity. nRT
V=
214. A system is taken from state a to state c by two P
paths adc and abc as shown in the figure. The Putting value of V in equation (i),
γ
internal energy at a is U a = 10 J. Along the path  nRT 
adc the amount of heat absorbed
P  = constant
 P 
δQ1 = 50 J and the work obtained Hence, P1–γ Tγ = constant
δW1 = 20 J whereas along the path abc the heat So, C & D does not give adiabatic process.
absorbed δQ 2 = 36 J. The amount of work 216. Ideal gas undergoes an adiabatic change in its
along the path abc is state from (P1,V1,T1) to (P2,V2,T2). The work
done (W) in the process is ( µ = number of mole
CP, and CV, are molar specific heats of gas)
(a) W = µ(T1 – T2 )C p (b) W = µ(T1 – T2 )C V
(c) W = µ(T1 + T2 )C p (d) W = µ(T1 + T2 )C V
COMEDK 2017
J&K CET- 2008
(a) 10 J (b) 12 J
Ans. (b) : In adiabatic process,
(c) 36 J (d) 6 J
COMEDK 2013 PV −P V
W= 1 1 2 2
Ans. (d) : Given that, δQ1 = 50 J, δQ2 = 36, Ua = 10 J γ −1
µR(T1 − T2 )
W=
γ −1
µR(T1 − T2 )
W=
CP
−1
CV
µR(T1 − T2 )  CP CP − CV R 
W= ∵ γ = γ −1 = = 
CP − CV  CV CV CV 
CV
δQ = δU + δW
µR(T1 − T2 )
δUadc = δUabc ∴W =
R
Qabc – Wabc = Qadc – Wadc
36 – Wabc = 50 – 20 CV
Wabc = 6 J W = µ(T1 − T2 )C V

619
217. 1 mm3 of a gas is compressed at 1 atmospheric After placing hot milk in a room, the temperature of
pressure and temperature 27°C to 627°C. milk decreases. Hence entropy also decreases (–ve).
What is the final pressure under adiabatic 1
∴ ∆S ∝ , ∆S ∝ Q .
condition? ( γ for thegas = 1.5 ) T
Second law of thermodynamics states that entropy of
(a) 27 × 105 N / m 2 (b) 80 × 105 N / m 2
universe increases.
(c) 36 × 105 N / m 2 (d) 56 × 105 N / m 2 Hence, Assertion (A) and Reason (R) is true but Reason
COMEDK 2020 is not correct explanation of the Assertion.
Ans. (a) : Given that, P1 = 1 atm = 1 × 105 N/m2 219. Assertion: Thermodynamic process in nature
T1 = 27 °C = 300 K are irreversible.
Reason: Dissipative effects cannot be
T2 = 627 °C = 900 K
eliminated.
γgas = 1.5 (a) If both Assertion and Reason are correct and
P2 = ? the Reason is a correct explanation of the
We know in an adiabatic process, Assertion.
γ (b) If both Assertion and Reason are correct but
P2  T2  γ −1 Reason is not a correct explanation of the
= 
P1  T1  Assertion.
3
(c) If the Assertion is correct but Reason is
2 incorrect.
P2  900  3 −1 (d) If both the Assertion and Reason are
= 2
P1  300  incorrect.
P2 (e) If the Assertion is incorrect but the Reason is
= 33 = 27 correct.
P1
AIIMS-2004
P2 27 Ans. (a) : Thermodynamics process is irreversible, as
=
1 1 there always occur a loss of energy due to energy spent
in working against the dissipative force which is not
P2 = 27 × 1 = 27 atm = 27 ×105 N / m 2
recovered back.
218. Assertion: When a glass of hot milk is placed in In nature most of the phenomenon is irreversible. It
a room and allowed to cool, its entropy happen when some energy is converted into heat
decreases. energy. This is also called dissipative effect.
Reason: Allowing hot object to cool does not 220. Assertion: Reversible systems are difficult to
violate the second law of thermodynamics. find in real world.
(a) If both Assertion and Reason are correct and Reason: Most processes are dissipative in
the Reason is a correct explanation of the nature.
Assertion. (a) If both Assertion and Reason are correct and
(b) If both Assertion and Reason are correct but the Reason is a correct explanation of the
Reason is not a correct explanation of the Assertion.
Assertion. (b) If both Assertion and Reason are correct but
Reason is not a correct explanation of the
(c) If the Assertion is correct but Reason is
Assertion.
incorrect.
(c) If the Assertion is correct but Reason is
(d) If both the Assertion and Reason are incorrect.
incorrect. (d) If both the Assertion and Reason are
(e) If the Assertion is incorrect but the Reason is incorrect.
correct. (e) If the Assertion is incorrect but the Reason is
AIIMS-2006 correct.
Ans. (b) : AIIMS-26.05.2019(M) Shift-1
AIIMS-2005
Ans. (a) : During work done some heat is dissipated.
Loss of heat is due to acting some force on it.
But in a perfectly reversible system, there is no loss of
energy. It is impossible by any means to recover the
∆Q energy lost in doing work against dissipative forces.
Entropy, ∆S =
T Usually all process are dissipative in nature.

620
221. The temperature-entropy diagram of a
reversible engine cycle is given in the figure. Its
efficiency is

(i) What is W along path ibf?


(ii) If W = 13 cal for path fi, what is Q for the
path fi
(iii) Take Eint,i = 10 cal then what is Eint,f?
(a) 30, 20, 40 cal (b) 6, – 43, 40 cal
(c) 10, –20, 30 cal (d) 15, 35, 25 cal
1 1
(a) (b) AIIMS-2009
4 2
Ans. (b) :
2 1
(c) (d)
3 3
AIIMS-2008
Ans. (d) :

(i)
We know,
Q = ∆U + W
For path (iaf),
Q = 50 cal
W = 20 cal
Q = ∆U + W
We know that,
∆U = Q – W
∆Q = 50 – 20
Entropy ( ∆S ) =
T ∆U = 30 cal
1 3 For path (ibf) ,
Q1 = T0S0 + T0S0 = T0S0 Q = 36 cal, ∆U = 30 cal and W = ?
2 2
W = Q – dU
Q 2 = T0S0
= 36 – 30
Q3 will be zero. W = 6 cal
Q (ii) For path of (fi),
Efficiency η = 1 − 2 W = – 13 cal
Q1
∆U = – 30 cal
TS Qfi = ?
η =1− 0 0
3 Qfi = ∆U + W
T0S0
2 Qfi = – 43 cal
(iii) Ei = 10 cal
2
η = 1− Ef = ?
3
Ef = Ei + ∆U
3−2 = 10 + 30
η=
3 Ef = 40 cal
1 223. The volume of a gas is reduced adiabatically to
η=
3 (1/4) of its volume at 27ºC. If γ = 1.4. The new
temperature will be:
222. When a system is taken from a state i to f along (a) 300×(4)0.4 K (b) 150×(4)0.4 K
the path iaf (as shown in the figure). Q = 50 cal (c) 250×(4) K0.4
(d) none of these
and W = 20 cal; along path ibf, Q = 36 cal. AIIMS-2002
621
Ans. (a) : Given, Ans. (a) : When a system under goes a change under
V the condition that do not exchange of heat takes places
V2 = 1 , T1 = 27°C +273 = 300 K
4 between the system and surroundings. Then such a
γ = 1.4 process called adiabatic process. The leaking air of
T2 = ? balloon under goes adiabatic expansion. In this
expansion due to work done against external pressure,
TV γ−1 = C
γ−1
the internal energy of air reduces, Thus, it becomes
T1  V2  cooler.
= 
T2  V1  227. Assertion : In adiabatic process work is
γ−1 independent of path.
 V1  Reason: In adiabatic process work done is
300  4 
=  equal to negative of change in internal energy.
T2  V1  (a) If both Assertion and Reason are correct and
  Reason is the correct explanation of
γ−1
300  1  Assertion.
=  (b) If both Assertion and Reason are correct, but
T2  4 
γ −1
Reason is not the correct explanation of
T2 = 300 × ( 4 ) Assertion.
1.4 −1
T2 = 300 × ( 4 ) (c) If Assertion is correct Reason is incorrect.
(d) If both the Assertion and Reason are
T2 = 300 × ( 4 )
0.4
K incorrect.
224. In an adiabatic process the quantity which AIIMS-25.05.2019(M) Shift-1
remains constant is : Ans. (a) : We know that,
(a) total heat of system (b) temperature dQ = dU + dW
(c) volume (d) pressure dU = dW, (dQ = 0 for adiabatic)
AIIMS-1999
If system considered is adiabatic so there is no heat
Ans. (a) : In an adiabatic process no transfer of heat transfer i.e dU = –dW
take place from the system to the surrounding neither in
case of expansion nor during compression. Thus, we This expression shows that the adiabatic work done in
can say that in an adiabatic process, the quantity which taking a system between an initial and final state is path
remains constant is the total heat of the system. independent and work done is equal to negative of
225. During the adiabatic expansion of two moles of change in internal energy.
a gas the internal energy of a gas is found to 1
decrease by 2 joule. The work done on gas 228. Assertion : P versus V graph is straight line
during the process will be equal to
(a) –2J (b) 3J for adiabatic process.
(c) 1J (d) 2J Reason: PV = constant for adiabatic process.
AIIMS-2000 (a) If both Assertion and Reason are correct and
Ans. (a) : From first law of thermodynamic Reason is the correct explanation of
∆Q = ∆W + ∆U Assertion.
For adiabatic process, Q = 0 (b) If both Assertion and Reason are correct, but
Reason is not the correct explanation of
∆W = – ∆U (∵ ∆U = 2 Joule)
Assertion.
∆W = – 2 Joule (c) If Assertion is correct Reason is incorrect.
Hence, work done on gas is – 2 Joule. (d) If both the Assertion and Reason are
226. Assertion: Air quickly leaking out of a balloon incorrect.
becomes cooler.
AIIMS-25.05.2019(M) Shift-1
Reason: The leaking air undergoes adiabatic
expansion. Ans. (d) : In adiabatic process
(a) If both assertion and reason are true and ∆Q = 0
reason is the correct explanation of assertion. From ideal gas equation (for adiabatic process)
(b) If both assertion and reason are true but PVγ = C
reason is not the correct explanation of
1
assertion. P ∝ γ (straight line)
(c) If assertion is true but reason is false. V
(d) If both assertion and reason are false. PV = C
AIIMS-26.05.2019(E) Shift-2 T = C for isothermal process.
622
229. In an isobaric process, the work done by a di- 1
atomic gas is 10 J, the heat given to the gas will (a) T 2 (b)
T
be:
(c) T 3 (d) T
(a) 35 J (b) 30 J
AP EAMCET(medical)-2004
(c) 45 J (d) 60 J BCECE-2015
AIIMS-25.05.2019(E) Shift-2
Ans. (b) : Ideal gas equation
Ans. (a) : For constant pressure process PV = RT
W = 10J
R
W nR∆T nR∆T 1 V =  T
= = = P
Q nCp∆T f  f
+1
n  + 1 R∆T V, is directly proportional to T.
2  2
V∝T
W 1 f = Degree of freedom 
= f = 5 for dia atomic gas  V1 T1
=
Q 5 +1   V2 T2
2
V2 T
W 2 −1 = 2 −1
= V1 T1
Q 7
7W [By subtracting V1 from both side]
Q= V2 − V1 T2 − T1
2 = [T2–T1 = 1 K]
7 × 10 V1 T1
Q=
2 V2 − V1 1 1
= =
Q = 35 Joule V1 T1 T
230. A cycle process is shown on p-T diagram. 232. An ideal gas at pressure p is adiabatically
Which of the following curves shows the same compressed so that its density becomes n times
process on p-V digaram. the initial value. The final pressure of the gas
 Cp 
will be  γ = 
 CV 
(a) n γ p (b) n −γ p
(c) n ( γ −1) p (d) n (1−γ ) p
BCECE-2009
m ( mass )
Ans. (a) : Volume (V) =
d ( density )
in adiabatic process
pVγ = C
γ
p1  V2 
= 
p 2  V1 
γ
m 
BCECE-2016, BITSAT-2009 p1  d 2 
PV = 
Ans. (b) : = constant p2  m 
T d 
 1 
AB is isochoric process, V = c γ
BC is isobaric process, P = c p1  d1 
= 
When T is decreasing v is decreasing. p2  d2 
Hence, CA is isothermal process γ
 nd 
When, P is decreasing v is increasing. p 2 = p1  
Hence, diagram will rectangular hyperbola.  d 
231. At constant pressure, the ratio of increase in p 2 = pn γ
volume of an ideal gas per degree rise in kelvin
p2 = n γ p
temperature to its original volume is–
623
233. At constant temperature on increasing the Ans. (a) : Given,
pressure of a gas by 10%, its volume will T1 = 27°C = 27 + 273 = 300 K
decrease by–
P 5
(a) 9.09% (b) 10% P1 = P, P2 = , γ=
(c) 5% (d) 20% 8 3
BCECE-2018 We know in adiabatic process,
Ans. (a) : P1 = P, then P2 = P + 10 of P = 1.1P γ−1

According to Boyle's law PV = constant T1  P1  γ


= 
V2 P1 P 100 T2  P2 
= = =
V1 P2 1.1P 110 γ−1
  γ
Change in volume
T1  P 
∆V V2 − V1 100 − 110 −10 = 
= = = T2  P 
V V1 110 110 8 
Percentage change in volume 5 
 −1
∆V −10 300  8 
3 
× 100% = ×100 = 
5/3
V 110 T2  1 
= – 9.09%
Its volume decrease by 9.09% 300
= (8)0.4
234. Certain amount of an ideal gas of molecular T2
mass M is contained in a closed vessel. If the 300
vessel is moving with a constant velocity v, then T2 =
the rise in temperature of the gas when the (8)0.4
vessel is suddenly stopped will be 300
T2 =
 CP  2.297
Take γ =  T2 = 131K (near about)
 C V 
T2 = 131 − 273
Mv 2 Mv 2
(a) (b) o
2R( γ + 1) 2R( γ − 1) T2 = −142 C
Mv 2 ( γ − 1) Mv 2 ( γ + 1) PV
(c) (d) 236. The figure below shows the plot of versus
2R 2R nT
BCECE-2012 P for oxygen gas at two different temperatures
Ans. (c) : If m is the total mass of the gas, then its
1
Kinetic Energy mv2.
2
Given,
Molecular mass = M
Molecular velocity = v
When stopped, Kinetic Energy of gas increases. Read the following statements concerning the
1 above curves :
= mv = nCv∆T
2
2 (i) The dotted line corresponds to the ideal gas
m behaviour.
= C v ∆T (ii) T1 >T2
M
(iii) The value of PV/nT at the point where the
mR 1  R 
∆T = mv 2  As C v =  curves meet on the y-axis is the same for all
M( γ − 1) 2  γ −1  gases.
Mv ( γ − 1)
2 Which of the above statements is true ?
∆T = (a) (i) only (b) (i) and (ii) only
2R
ο (c) All of these (d) None of these
235. At 27 C a gas suddenly compressed such that
BCECE-2006
its pressure becomes 1/8th of original pressure,
The temperature of the gas will be Ans. (c): There is no deviation in the value of
(γ = 5/3) PV
in diagram.
(a) –142οC (b) 300K nT
(c) 327οC (d) 420K This is behaves like Ideal gas.
BCECE-2007 Dotted line behaves as an Ideal gas.
624
At high temperature→ Deviation less dV
W = RT ∫
At low temperature→ Deviation more- V
Thus, T1 > T2 V2
dV
Since, the two curves interact at dotted line so the value W = RT ∫
V1
V
PV
of at point on the y-axis is same for all gases.
W = RT [ log e V ]V2
V
nT
1
237. The following figure represents the
temperature versus time plot for a given W = RT [ log e V2 − log e V1 ]
amount of a substance when heat energy is V2
supplied to it at a fixed rate and at a constant W = RT log e
V1
pressure.
239. Which is not a path function?
(a) ∆Q (b) ∆Q + ∆W
(c) ∆W (d) ∆Q − ∆W
BCECE-2005
Ans. (d) : Heat and work are path function internal
energy is state function.
Which parts of the above plot represent a From 1st law of thermodynamics
phase change ? ∆Q = ∆W + ∆U
(a) a to b and e to f (b) b to c and c to d ∆U = ∆Q – ∆ W
(c) d to e and e to f (d) b to c and d to e
So, ∆Q – ∆W is not a path function.
BCECE-2006
240. The pressure and density of a diatomic gas
Ans. (d) : In given figure, phase change from b to c and
from d to e.  7
 γ = 5  change adiabatically from (P, d) to
At 0°C from b to c temperature of matter does not  
change but its state changes. d′ P′
Similarly, from d to e state of matter changes without (P' d ′ ). If = 32, then should be :
d P
changing temp,
1
238. The work done, W during an isothermal (a) (b) 32
process in which 1 mole of the gas expands 128
from an initial volume V1 to a final volume V2 (c) 128 (d) none of these
is given by : (R = gas constant, T = temperature) BCECE-2003
T  Ans. (c) : According to adiabatic process
(a) R(V2 − V1 )log  1 
P1v1γ = P2v2γ ...(i)
 T2 

γ d =
m
V  γ
m 
(b) R(T2 − T1 ) log e  2  m V
P1   = P2    
 V1   d1   d2  γ = m 
V   d 
(c) RT log e  2 
 V1  P1 P
= 2
V  (d1 ) γ (d 2 ) γ
(d) 2RT log e  1  γ
 V2  P2  d 2 
= 
BCECE-2005 P1  d1 
Ans. (c) : Work done is given by, γ
P'  d' ∵ (P1 = P) 
W = ∫ PdV ….(i) =   P = P ', d = d '
P d  2 2 
We know,
P' d' 7
PV = RT = (32)7 / 5  d = 32, γ = 5 
P  
RT
P= P'
V = 27
Putting value of P in equation (i) P
RT P'
W=∫ .dV = 128
V P

625
241. A gas at NTP is suddenly compressed to one- (a) 9E (b) 6E
fourth of its original volume. If γ is supposed to (c) 3E (d) E
be 3/2, then the final pressure is : MHT-CET 2020
3
(a) 4 atm (b) atm Ans. (a) :
2
E1 = σ × π R 12 × T14
1
(c) 8 atm (d) atm
4 E2 = σ × π R 22 × T24
BCECE-2003 2 4 2
E 2  R 2   T2   1  1
   =   ( 3) = × 81
4
V 3 ∴ =
Ans. (c) : V2 = , V1 = V, γ = E1  R1   T1   3  9
4 2
The process is adiabatic process. E2 = 9E1
γ
Pv = constant E2 = 9E
γ A black sphere has radius 'R' whose rate of radiation
P1  V2 
=  R R
P2  V1  and temperature 'T'. If radius is made and temp
3 3
γ
  '3T', the rate of radiation will be 9E.
P1  V  244. An electric kettle takes 4A current at 220V.
= 
P2  4  How much time will it take to boil 1 kg of water
 
V from temperature 20ºC? The temperature of
3/ 2
P1  1  boiling water is 100ºC
=  (a) 12.6 min (b) 4.2 min
P2  4 
(c) 6.3 min (d) 8.4 min
P2 = P1 (4)3/ 2
VITEEE-2010
P2 = P1 × 23
Ans. (c) : Given,
P2 = 8P1
m = 1000 gm
Hence, final pressure is 8 atmosphere. S = 1 cal/gºC
242. If α is the coefficient of performance of a We know that
refrigerator and ‘Q1’ is heat released to the hot
reservoir, then the heat extracted from the cold Q = mS ∆T = 1000 × 1 × (100 – 20)
reservoir ‘Q2’ is Q = 1000 × 80 cal ….(i)
αQ1 αQ1 Heat produced in time t is
(a) (b)
α -1 1+ α H = VIt
1+ α α -1 H = 220 × 4 × t Joule
(c) Q1 (d) Q1
α α 220 × 4 × t
MHT-CET 2009 H= cal …..(ii)
4.18
Ans. (b) : Coefficient of performance (COP) of a Equating (i) and (ii)
refrigerator,
200 × 4 × t
Q2 ∴ = 1000 × 80
α= 4.18
Q1 − Q 2
1000 × 80 × 4.18
1 Q1 − Q 2 Q1 t=
= = −1 220 × 4
α Q2 Q2
t = 380 sec = 6.3 min
Q1 1
= +1 245. In which of the processes, does the internal
Q2 α
energy of the system remain constant ?
α Q1 (a) Adiabatic (b) Isochoric
Q2 =
1+ α (c) Isobaric (d) Isothermal
243. A black sphere has radius ‘R’ whose rate of Karnataka CET-2008
radiation is ‘E’ at temperature ‘T’. If radius is Odisha JEE 2009
R
made ' ' and temperature ‘3T’, the rate of Ans. (d) : In isothermal process, the internal energy of
3
radiation will be the system remains constant.

626
246. An ideal gas is taken around ABCA as shown Ans. (a) : In taking a system from one state to another
in the PV diagram. The work done during a by different processes, the heat transferred Q and work
cycle is: done W are different, but their Q-W is same for all
processes. It gives the internal energy of the system.
∆U = Q – W
Thus, internal energy U of a thermodynamic system is a
characteristic property of the state of the system, it does
not matter how that state has been obtained.
249. Petrol engine does the work during
1 (a) suction stroke (b) exhaust stroke
(a) Zero (b) PV
2 (c) adiabatic expansion (d) combustion
(c) 2PV (d) PV J&K CET- 2005
Karnataka CET-2001 Ans. (c) : Petrol engine works during adiabatic
Ans. (d) : According to question- expansion.
The work done during a cycle is equal to the area under 250. The enthalpy change of a chemical reaction in
a P-V diagram. which 1 mole of a pure substance is formed
from the free elements in their most stable
states under standard state conditions is called
(a) Molar enthalpy of vaporization
(b) Standard enthalpy of fusion
(c) Standard enthalpy of vaporization
(d) Standard molar enthalpy of formation
J&K-CET-2018
1
W = × AC × BC Ans. (d) : The enthalpy change of a chemical reaction
2 in which 1 mole of a pure substance is formed from the
1 free elements in their most stable states under standard
W = × (3V − V) × (2P − P) state condition is called standard molar enthalpy of
2
formation.
1
W = × 2V × P Molar enthalpy of vaporization is the amount of energy
2 needed to change one mole of a substance from the
W = PV liquid phase to the gas phase at constant temperature
247. Temperature remains constant, the pressure of and pressure.
gas is decreased by 20%. The percentage 251. The processes 3, 2 and 1 represent the
change in volume is respective isobaric, isothermal and adiabatic
(a) increased by 20% (b) decreased by 20% expansion of one mole of a gas initially at a
(c) increased by 25% (d) decreased by 25% pressure P from volume V to 2V. The work
J&K CET- 2007 done is
Ans. (c) : Ideal gas equation PV = nRT
PV = constant = K {temp remain constant}
Pressure decrease by 20% P2 = 0.8P
So,
PV = (0.8P)V2
(a) Maximum in the process 1
PV V
V2 = = = 1.25V (b) Maximum in the process 2
0.8P 0.8 (c) Minimum in the process 1
So, volume of gas increases by 25% (d) Minimum in the process 3
248. Which of the following statement is true? J&K-CET-2016
(a) Internal energy of a gas depends only on the Ans. (c) : We know that work done = area under P-V
state of the gas. curve. Hence, from the given graph, we can infer that
(b) In an isothermal process change in internal the area under the process 3 curve will be maximum and
energy is maximum. under the process 1 curve will be minimum.
(c) Area under pressure, volume graph equals
heat supplied in any process.
(d) Work done is state dependent but not path
dependent.
J&K CET- 2006
627
252. A sample of ideal monoatomic gas is taken (a) 30 J (b) 40 J
round the cycle ABCA as shown in the figure. (c) 10 J (d) 90 J
The work done during the cycle is WB JEE 2020
Ans. (a) : Since we know that area under the curve on
P-V diagram gives as work transfer
So, for cyclic process,
∆Q = ∆W
∆Q = Qab + Qbc + Qca
(a) 3 pV (b) zero = –50 + 80 + 0
(c) 9 pV (d) 6 pV ∆Q = 30J
BITSAT-2012 Hence, ∆W = 30J
Karnataka CET-2003 255. Which of the following statements (s) is/are
Ans. (a) : According to figure- true?
∆W = Area under P - V curve "Internal energy of an ideal gas….."
1 (a) decreases in an isothermal process
= 3p × 2V (b) remains constant in an isothermal process
2
∆W = 3pV (c) increases in an isobaric process
(d) decreases in an isobaric expansion
253. WB JEE 2018
Ans. (b) : Internal energy of an ideal gas depends upon
the temperature of gas.
In isothermal process ∆U = 0
In isobaric expansion V ∝ T
So ∆U increases.
In the given figure, 1 represents isobaric, 2 Hence, internal energy of an ideal gas remains constant
represents isothermal and 3 represents in an isothermal process.
adiabatic processes of an ideal gas. If ∆U1, ∆U2, 256. One mole of a monoatomic ideal gas undergoes
∆U3 be the changes in internal energy in these a quasistatic process, which is depicted by a
processes respectively, then straight line joining points (V0, T0) and (2V0,
(a) ∆U1 < ∆U2 < ∆U3 (b) ∆U1 > ∆U3 < ∆U2 3T0) in a V-T diagram. What is the value of the
(c) ∆U1 = ∆U2 > ∆U3 (d) ∆U1 > ∆U2 > ∆U3 heat capacity of the gas at the point (V0, T0)?
WB JEE 2021 3
(a) R (b) R
Ans. (d) : Process 1 is isobaric expansion hence, 2
temperature of gas will increase. (c) 2R (d) 0
∵ ∆ U = mcv∆T WB JEE 2018
∴ ∆ U1 = positive Ans. (c) : dQ = dU + pdV
Process 2 is isothermal process dQ = nCVdT + pdV
∴ ∆ U2 = 0 dQ = CVdT + pdV ……(i) [ n = 1 mol]
Process 3 is an adiabatic expansion hence, temperature For monoatomic gas.
of gas will fall. 5
γ=
∴ ∆ U3 = negative 3
∴ ∆ U1 > ∆ U2 > ∆ U3 CP 5 5
254. An ideal gas undergoes the cyclic process abca = ⇒ CP = CV
CV 3 3
as shown in the given p-V diagram.
As we know,
CP – CV = R
5
CV − CV = R
3
3
It rejects 50 J of heat during ab and absorbs 80 CV = R
2
J of heat during ca. During bc, there is no
transfer of heat and 40 J of work is done by the
Putting value of Cv in equation (i), we get
gas. What should be the area of the closed 3
dQ = RdT + pdV
curve abca? 2
628
259. Choose the correct statement.
(a) The second law of thermodynamics disallows
some processes consistent with the first law
of thermodynamics
(b) Spontaneous processes are reversible
(c) In isothermal quasi static processes, the heat
3 RT
So, dQ = R × (3To − To ) + o × (2Vo − Vo ) cannot be absorbed or given out by the
2 Vo system.
3 RT (d) In quasi-static processes, the pressure and
dQ = R × 2To + o × V0
2 Vo temperature of the environment can differ
dQ = 4RTo from those of the system significantly.
CdT = 4RTo TS EAMCET 31.07.2022, Shift-II
C(3To – To) = 4RTo Ans. (a) : Correct statement as follows,
C × 2To = 4RTo (i) The second law of thermodynamics disallows some
C = 2R. processes consistent with the first law of
257. An ideal mono-atomic gas of given mass is thermodynamics.
heated at constant pressure. In this process, the (ii) 2nd law of thermodynamics states that all
fraction of supplied heat energy used for the spontaneous process are irreversible.
increase of the internal energy of the gas is (iii) In isothermal quasi-static process, the heat can be
(a) 3/8 (b) 3/5 absorbed or given out by the system.
(c) 3/4 (d) 2/5 (iv) In quasi-static process the system will be in
WB JEE 2013 equilibrium with the surroundings at every stage.
∆U Cv Hence, in quasi-static process the pressure and
Ans. (b) : Fraction = =
∆Q Cp temperature of the environment can not differ from
those of the system significantly.
We know that,
260. A diatomic gas is heated at constant pressure,
Cp
γ= what fraction of the heat energy is used to
Cv increase the internal energy?
5 3 3
For monatomic gas, γ = (a) (b)
3 5 7
Cv 1 3
So, = = (c)
5
(d)
5
Cp γ 5 7 9
258. A frictionless piston-cylinder based enclosure AP EAMCET-20.08.2021, Shift-II
contains some amount of gas at a pressure of Ans. (c) : Fraction of heat energy utilized in increasing
400 kPa. Then heat is transferred of the gas at
internal energy
constant pressure in a quasi-static process. The
piston moves up slowly through a height of 10 nCV ∆T
=
cm. If the piston has a cross-sectional area of nCp ∆T
0.3 m2, the work done by the gas in this process
We know that,
is
(a) 6 kJ (b) 12 kJ CP – CV = R
(c) 7.5 kJ (d) 24 kJ CP/CV = γ
WB JEE 2013 2
γ = 1+
Ans. (b) : Given that, f
P = 400 k Pa CV 1
h = 10 cm = 10–1 m =
2
CP γ
A = 0.3 m
W=F×h CV 1 f
= =
CP 1 + 2 / f 2 + f
∵ P = F/A ⇒ F = PA
Where f is the number of degrees of freedom of the gas
∴ W = PAh
For diatomic gas, f = 5
W = 400 × 10 × 0.3 × 10
3 –1

W = 12 kJ Thus, fraction = 5 / 7

629
261. Which one of the graphs below best illustrates Number of vibrational dof = 2 × 2 = 4
the relationship between internal energy U of Therefore, Total number of dof
an ideal gas and temperature T of the gas in K? f = 3+3+4 = 10
(a) (b) CP 2 2 12 6
= 1+ = 1+ = =
CV f 10 10 5
265. Three identical silver cups A, B and C contain
(c) (d) three liquids of same densities at same
temperature higher than the temperature of
the surrounding. If the ratio of their specific
AP EAMCET-19.08.2021, Shift-II heat capacities is 1 : 2 : 4, then
Ans. (a) : For Ideal gas, (a) A cools faster than B but slower than C
Total kinetic energy = Internal energy (b) B cools faster than C but slower than A
3  3 (c) A cools faster than B and C
U = n  KT  = nRT (d) C cools faster than B and A
2  2
(e) B cools faster than A and C
U ∝T
U = CT Kerala CEE 04.07.2022
Ans. (b, c) : Q = MCp∆T = constant
Cp , ∆T So, the rate of cooling will decreases.
(Cp)A : (Cp)B : (Cp)C = 1 : 2 : 4
(∆T)A : (∆T)B : (∆T)C = 4 : 2: 1
It means, A cools faster than B cools faster than C.
262. Which of the following is NOT a reversible So, B cools faster then C but slower than A.
process? So, A cools faster than B and C.
(a) Melting of ice 266. Temperature of cold junction in a
(b) Conduction of heat thermocouple is 10°C and neutral temperature
(c) Isothermal expansion of gas is 270°C, then the temperature of inversion is
(d) Adiabatic expansion of gas (a) 540°C (b) 530°C
AP EAMCET-19.08.2021, Shift-II (c) 280°C (d) 260°C
Ans. (b): Conduction of heat cannot be reversed. EAMCET-2007
263. Psychrometer is used to measure Ans. (b) : Given,
(a) relative humidity (b) pressure
Temperature at neutral junction (Tn)= 270°C
(c) temperature (d) density
Temperature at cold junction (Tc) = 10°C
JIPMER-2019
Temperature at inversion junction (Ti) = ?
Ans. (a) : A Psychrometer uses two thermometers to
measure relative humidity one measure dry-bulb We know that,
temperature and the other measures the wet-bulb Tn – Tc = Ti – Tn
temperature. 270 – 10 = Ti – 270
264. A poly atomic molecule has 3 translational, 3 270 + 270 – 10 = Ti
rotational degrees of freedom and 2 vibrational Ti = 530°C
Cp 267. The heat supplied to gas in the cyclic process
modes. The ratio of specific heats is
Cν ABCA (shown in figure) is
7 3
(a) (b)
5 5
5 5
(c) (d)
6 3
6
(e)
5
Kerala CEE 04.07.2022
Ans. (e) : Degree of freedom of polyatomic gas. (a) –2J (b) 4J
Number of translational dof = 3 (c) – 4J (d) 8J
Number of rotational dof = 3 UP CPMT-2001

630
Ans. (c): Work done = Area of ∆BCA 269. When a system is taken from state i to state f
1 along path ia ƒ in the figure, the heat absorbed
= − × ( 5 − 1) × ( 3 − 1) Q = 50 cal and the work done W = 20 cal. If W
2
= –13 cal for the return path ƒi, Q for this path
1
= − × 4× 2 is
2
= –4 J
(–)ve sign shows the process is in anticlockwise and
heat absorbing device.
268. The efficiency of an ideal gas with adiabatic
exponent γ for the shown cyclic process would
be
(a) 17 cal (b) -17 cal
(c) 43 cal (d) -43 cal
AMU-2008
Ans. (a):

(2ln 2 − 1) (1 − 1ln 2)
(a) (b)
γ /( γ − 1) γ /( γ − 1)
(2ln 2 + 1) (2ln 2 − 1)
(c) (d)
γ /( γ − 1) γ /( γ + 1) Qiaf = ∆Uiaf+ Wiaf
JIPMER-2018 50 =∆Uiaf + 20
Ans. (a): ∵Efficiency ∆Uiaf = 30 cal,
Qibf = W+∆U
= –13+ 30
Qibf = 17 cal (for return path)
270. The ratio of the slopes of isothermal and
adiabatic curves is
(a) 1 (b) γ
1 3
(c) (d)
Work done ( WD ) γ 2
Efficiency, η =
Heat input ( Q ) AP EAMCET (17.09.2020) Shift-I
WBC = PdV Ans. (c) : For isothermal process,
PV = Constant
= nR (dT) (∵PV = nRT)
PdV + VdP = 0
= nR (T0 – 2T0) PdV = –VdP
= – nRT0
 dP   P 
 2T    =  −  = slope of isothermal
WCA = 2nRTo ln  o   dV   V 
 To  For adiabatic process
= 2nRTo ln (2) PVγ = constant
∆QBC = nCp∆T P ( γ V γ−1dV ) + V γ dP = 0
nγR(2To − To ) nγRTo
= =  dP   P
γ −1 γ −1   = γ− 
 dV   V
WBC + WCA 2nRTo ln ( 2 ) − nRTo
Efficiency, η = = dP P
∆Q BC nRγTo = −γ   = slope of adiabatic
γ −1
dV V
nRTo ( 2ln 2 − 1)  dP   dP 
  = γ 
η=  dV adiabatic  dV isothermal
nRTo ( γ / γ − 1)
271. One mole of an diatomic ideal gas undergoes a
η=
( 2ln 2 − 1) process shown in P-V diagram. The total heat
( γ / γ − 1) given to the gas (ℓn 2 = 0.7) is

631
P
5
Q= P0V0+2P0V0 (0.7)
2
2P0 B 5 P V + 4P0 V0 (0.7)
Q= 0 0
2
Isothermal
5 P0 V0 + 2.8 P0 V0
P0 Q=
C 2
A
7.8 P0 V0
Q=
V 2
V0 2V0
Q = 3.9 P0 V0
(a) 2.5 P0V0 (b) 3.9 P0V0
(c) 1.1 P0V0 (d) 1.4 P0V0 272. The internal energy of an ideal gas depends on
WB JEE 2022 (a) Specific volume (b) Pressure
Ans. (b) : (c) Density (d) Temperature
TS EAMCET 30.07.2022, Shift-II
Ans. (d) : The internal energy of an ideal gas is the sum
of kinetic energy of the particle in the gas. In an ideal
gas, the intermolecular forces are assumed to be absent
and all the collisions are perfectly elastic. Thus the gas
possesses only translation kinetic energy and hence, the
internal energy of the ideal gas depends only on the
temperature.
273. A gas system is taken through the
thermodynamic cyclic process 1 → 2 → 3 → 1
as shown below. The amount of heat transfer
We know that ideal gas equation,
PV = nRT
 nR 
P= T
 V 
P∝T
P0 →T0 (A)
2P0→2T0 (B) V
(a) −P (b) PV
Heat supplied, 2
QAB = ∆UA→B + WA→B PV −3PV
(c) (d)
QA→B = nC v ∆T + P∆V 2 2
for (constant volume) ∆V= 0 TS EAMCET 20.07.2022, Shift-II
Ans. (b) : First law of thermodynamics
5 5R
QA→B = R [ TB − TA ] For diatomic CV = ∆Q = ∆U + ∆W …..(i)
2 2 For a cyclic process internal energy is zero
5 ∆U = 0
QA→B = RT0
2 Now, the first law of thermodynamics change into
Again PV = nRT Joule's law
PV = RT (For one mole gas) ∴ ∆Q = ∆W
P0V0 = RT0 Therefore, workdone under P –V diagram represent area
5 under the curve.
QA→B = P 0V 0 1
2 ∆Q = × P × V
Heat supplied → B to C 2
PV
 2V  ∆Q =
QB→C = 0 + (1) R(2T0)ln  0  2
 V0  274. A gas undergoes a thermodynamic cycle as
QB→C = 2RT0ln(2) shown in pressure-volume diagram below. The
QB→C = 2P0V0ln(2) heat transferred to the gas is 300J and 140J
along path 1→2 and 2→3 respectively. The
QB→C = 2P0V0 (0.7)
internal energy is changed by –260J along path
Total heat 3→1. The work done by gas along path
Q = QA→B + QB→C 1→2→3 is
632
Wcd = PdV
= 350×103 [200–400] × 10–6
= 350 × 103 (–200) × 10–6
= –70 Joule
For process da (volume at constant process)
Wda = 0
(a) –80 J (b) 140 J Total work done = Wab+Wbc+Wcd+Wda
(c) 160 J (d) 180 J 30+0–70+0
TS EAMCET 08.05.2019, Shift-II Wtotal = –40 Joule
Ans. (d) : Given that, For process abcda
Heat transfer to gas along path 1→2 is Q1 = 300 J Internal energy (∆U) = 0 (Because it is a cyclic process)
Heat transfer to gas along path 2→ 3 is Q2= 140 J The amount of heat
The change in internal energy along 3→1 is ∆U = –260J
dQ = dW+dU
According to first law of thermodynamic,
dQ = –40 Joule (–ve sign indicate heat is rejected)
W = Q1+Q2 – ∆U
= 300+140–260 276. 7 mole of certain monoatomic ideal gas
= 440–260 undergoes a temperature increase of 40K at
= 180 J constant pressure. The increase in the internal
energy of the gas in this process is
275. A gas system is taken through the
thermodynamic cycle (abcda) as shown in the (Given R = 8.3 JK–1 mol–1)
figure. The amount of heat change experienced (a) 5810 J (b) 3486 J
by the gas during the process is (c) 11620 J (d) 6972 J
JEE Main-26.07.2022, Shift-I
Ans. (b) : Given that,
n = 7 mole, ∆P = 0, ∆T = 40K
∆U = ?
For monoatomic internal energy,
3
(a) 40 J of heat is rejected ∆U = × n × R × ∆T
2
(b) 40 J of heat is absorbed
(c) 100 j of heat is rejected 3
= × 7 × 8.3 × 40
(d) 100 J of heat is absorbed 2
TS EAMCET 03.05.2018, Shift-I 3 × 7 × 8.3 × 20
Ans. (a) : ∆U = 3486 Joule
 7
277. Carbon monoxide  γ =  is carried around a
 5
closed cyclic process abc, in which 'bc' is an
isothermal process as shown in figure. The gas
absorbs 6000 J of heat as its temperature is
increased from 200 K to 800 K in going from
'a' to 'b'. The quantity of heat ejected by the
gas during the process 'ca' is
For process ab (constant pressure process)
Wab = PdV
= 150×103 (400–200) × 10–6
= 150×103 × 200 × 10–6
= 15×2×106×10–6
= 30×106×10–6
= 30 Joule
For process bc (constant volume process)
∆V = 0 (a) 6000 J (b) 2400 J
Therefore, Wbc = 0 (c) 8400 J (d) 4800 J
For process cd (constant pressure process) AP EAMCET-24.04.2019, Shift-II

633
Ans. (c) : Given, Heat absorbed a→b (a) (b)
(∆Q)ab = +6000 Jule = (at constant volume)
+ 6000 = mCV (∆T)
6000 = mCV (800–200)
6000 = mCV × 600 ....(i) (c) (d)
mCv = 10
For the process c to a
Ta = 200K Tb = Tc = 800 K
(∆Q)ca = mCP (∆T)
(∆Q)ca = mCP (200-800) BITSAT-2018
(∆Q)ca = – mCP × 600 AP EAMCET -2009
= –m (CV+R)×600 Ans. (a) :
For carbon monoxide
γ = 7/5
R R 5R
CV = = = ....(iii)
γ −1 75 −1 2 281. The relation between efficiency η of a heat
From equation (i) and (iii) engine and the coefficient of performance α of
.5R a refrigerator is
6000 = m × 600 1 1
2 (a) η = (b) η =
m5R 1− α 1+ α
10 = (c) η = 1 + α (d) η = 1 − α
2
mR = 4 AP EAMCET -2015
(Q)ca = –m (CV+R) × 600 Ans. (b) : We know that, Efficiency of a heat engine is
(Q)ca = (–mCV – mR)×600 given as,
= (–10–4)×600 T
η = 1− 2
= –14×600 T1
(Q)ca = –8400 (–ve sign indicates heat ejected)
T1 − T2
(Q)ca = 8400 Joule =
T1
278. In which of the following processes the internal
energy of the gas remains unaffected? 1
=
(a) Isobaric process (b) Adiabatic process T2
1+
(c) Isochoric process (d) Isothermal process T1 − T2
AP EAMCET-11.07.2022, Shift-II 1
Ans. (d) : As we know that, in an isothermal process η=
1+ α
the temperature is constant the change in internal energy When, η is the efficiency.
is zero because internal energy is the function of
temperature. T1 and T2 are temperature and α is the coefficient of
performance.
279. Heating of water at atmospheric pressure is an
……… . 282. A system is taken from state-A to state-B along
two different paths. The heat absorbed and
(a) isothermal process (b) isobaric process
work done by the system along these two paths
(c) adiabatic process (d) isochoric process are Q1, Q2 and W1, W2 respectively, then
AP EAMCET-23.09.2020, Shift-II (a) Q1 = Q2 (b) W1 = W2
Ans. (b) : Heating of water at atmospheric pressure is (c) Q1 − W1 = Q2 − W2 (d) Q1 + W1 = Q2 + W2
an isobaric process. An isobaric process is an process AP EAMCET-19.08.2021, Shift-I
occurring at constant pressure.
Ans. (c) :
280. An ideal gas is subjected to a cyclic process
ABCD as depicted in the p-V diagram given
below
According to the first law of thermodynamics,
Q1 = ∆U + W1
Q2 = ∆U + W2 (∆U = Uf–Ui independent of path)
Which of the following curves represents the Q 1 –Q 2 = W 1 –W 2

equivalent cyclic process? Q1 − W1 = Q 2 − W2

634
283. Match the following The total pressure P2 of the gas inside cylinder
List - I List - II P2 = Patm+∆P
(i) Isothermal process (a) 0 = 1×105+1×105
1 = 2×105 N/m
(ii) Isobaric process (b) [P2 V2 − P1V1 ] Volume of the gas
γ −1
∆V = A × ∆x
V  = 8×10–3 × 0.1
(iii) Isochoric process (c) µRT1n  2 
 V1  = 8×10–4 m3
(iv) Adiabadic process (d) P(V2–V1) The final volume of the gas
V2 = V1+∆V = 2.4×10–3+8×10–4
= 3.2×10–3 m–3
(a) i-c, ii-d, iii-a, iv-b (b) i-a, ii-d, iii-b, iv-c
Let T2 be the final temperature of gas then
(c) i-c, ii-b, iii-d, iv-a (d) i-b, ii-c, iii-a, iv-d
P1V1 P2 V2
AP EAMCET-25.04.2017, Shift-II =
AP EAMCET(Medical)-2013 T1 T2
Ans. (a) : PV 
T2 =  2 2  T1
V   P1V1 
(i) Isothermal Process = µRT ln  2 
 V1  300 × 2 ×105 × 3.2 ×10−3
(ii) Isobaric Process = P(V2–V1) = = 800K
(2.4 × 10−3 × 105 )
(iii) Isochoric Process = 0
285. Which of the following statement is wrong?
1
(iv) Adiabatic Process = [ P2 V2 − P1V1 ] (a) In an adiabatic process ∆Eint = –W
γ −1 (b) In a constant volume process ∆Eint =Q
284. An ideal monoatomic gas is confined in a (c) In a cyclic process ∆Eint =0
cylinder by a spring-loaded piston of area of (d) For adiabatic expansion of an ideal gas TVγ =
cross-section 8×10–3 m2. Initially the gas is at constant
300 K and occupies as volume of 2.4×10–3 m3 AMU-2007
and the spring is in its relaxed state as shown in Ans. (d) : For an adiabatic process,
the figure. The gas is heated by a small electric ∆Einput = –W
heater until the piston moves out slowly by
From first law of thermodynamics,
0.1m. The final temperature of the gas is
dQ = dU+PdV
_______.
(The force constant of the spring = 8000 Nm–1 ∆E = ∆U+PdV
Atmospheric pressure = 1×105 Nm–2 For constant volume ∆V = 0
The piston is massless and there is no friction ∴ ∆E= ∆U
between the piston and cylinder) For a cyclic, ∆Eint= 0
For adiabatic expansion of an ideal gas
T1V1γ–1 = T2V2γ–1
∴ TVγ–1 = constant
286. A sample of an ideal gas undergoes an
isothermal process as shown by the curve AB in
(a) 800 K (b) 400 K
the PV diagram. If ∆Q, ∆U and ∆W represent
(c) 600 K (d) 300 K the amount of heat absorbed the change in
AP EAMCET-28.04.2017, Shift-I internal energy and the work done respectively,
Ans. (a) : Given that, then which of the following statements is
P1 = 1×105 N/m2 K = 8000 N/m correct?
V1= 2.4 ×10–3 m3 ∆x = 0.1m
T1 = 300K A = 8×10–3 m2
When the heat is supplied by the heater the piston is
compressed by 0.1m. The reaction force of compression
of spring is equal.
f = K × ∆x (a) ∆Q = +ve, ∆U = 0, ∆W = –ve
= 8000 × 0.1 = 800 N (b) ∆Q = +ve, ∆U = 0, ∆W = +ve
Pressure exerted on the piston by the spring (c) ∆Q = +ve, ∆U = +ve, ∆W = 0
800 (d) ∆Q = +ve, ∆U = +ve, ∆W = +ve
∆P = F/A = = 1× 105 N / m 2
8 ×10−3 AMU-2018
635
Ans. (b) (a) (i) (b) (ii)
(c) (iii) (d) (iv)
AMU-2013
Ans. (b) :

For the graph of an ideal the process is an isothermal


expansion because volume of the ideal gas increases.
VB > VA & P B < P A 289. An ideal gas undergoing adiabatic change has
the following pressure-temperature
∵ ∆T = 0 (Isothermal Process) relationship
∴ Change in internal energy (a) Pγ–1 T γ = constant (b) PγTγ–1 = constant
∆U = 0 (∵ ∆U ∝ ∆T) (c) PγT1–γ = constant (d) P1–γT γ = constant
For an isothermal process- AIPMT-1996
 Vf  AP EAMCET(Medical)-1999
∆Q = ∆W = nRT ln   Ans. (d) : We know that, for reversible adiabatic
 Vi 
process,
V  PVγ = constant ….(i)
∆Q = ∆W = nRT ln  B 
V
 A Now the equation of an ideal gas
PV = nRT ….(ii)
Hence, ∆Q = +ve and ∆W = +ve
nRT
287. A thermodynamic state of a given sample of an V= ….(iii)
ideal gas is completely described, if its P
(a) Pressure, volume and internal energy are Put the value of V in equation (i), we get-
γ
known  nRT 
(b) Pressure, volume, temperature and internal P   = constant
 P 
energy are known γ
γ T
(c) Pressure, volume and temperature are known P ( nR ) γ = constant
(d) Pressure and volume are known P
AMU-2013
Ans. (c) : A thermodynamic state of a given sample of P1– γ T γ (nR)γ = constant
an ideal gas is completely described when its pressure P1– γ T γ = constant
volume and temperature are known. 290. One mole of an ideal gas at an initial
288. A cyclic process ABCA on the V-T diagram temperature of T K does 6R joules of work
(shown below) is performed with a constant adiabatically. If the ratio of specific heats of
mass of an ideal gas this gas at constant pressure and at constant
volume is 5/3, the final temperature of gas will
be
(a) (T + 2.4) K (b) (T – 2.4) K
(c) (T + 4) K (d) (T – 4) K
AIPMT-2004
Ans. (d) : Given, mass of an ideal gas = 1 mole, initial
temperature = TK, Work done = 6R Joules,
Which of the following figures corresponds to CP = γ = 5 / 3
the same process on a P-V diagram? CV
We know that,
(i) P1V1 – P2 V2
(ii) Work done for an adiabatic process (W) =
γ –1
nR ( T1 – T2 )
W= (∵PV = nRT)
γ –1
(iii) (iv) 1× R ( T – T2 )
6R =
γ –1

636
5  nR
6  – 1 = T – T2 Now, dU = dT
3  γ –1
4 = T – T2 nR 2
γ – 1 ∫1
dU = dT
T2 = (T – 4)K
nR
291. In thermodynamic processes which of the ∆U = [T2 – T1 ]
following statements is not true ? γ –1
(a) In an adiabatic process the system is insulated P V – PV
∆U = 2 2 1 1
from the surroundings γ –1
(b) In an isochoric process pressure remains 2 × 6 – 5 × 4 12 – 20
constant = =
7 2/5
(c) In an isothermal process the temperature –1
5
remains constant
−8 × 5
(d) In an adiabatic process PVγ = constant = = –20 kJ
2
AIPMT-2009
293. The volume (V) of a monoatomic gas varies
Ans. (b) :
with its temperature (T), as shown in the
• In an isochoric process volume remains constant. graph. The ratio of work done by the gas, to the
• In an isobaric process pressure remains constant. heat absorbed by it, when it undergoes a
292. One mole of an ideal diatomic gas undergoes a change from state A to state B, is
transition from A to B along a path AB as
shown in the figure.

1 2
(a) (b)
3 3
The change in internal energy of the gas during
2 2
the transition is (c) (d)
5 7
(a) 20 kJ (b) –20 kJ
(c) 20 J (d) –12 kJ NEET-2018
AIPMT-2015 Ans. (c) : Given, for monoatomic gas (γ) = 5/3
AP EAMCET (23.09.2020) Shift-I Volume (V) ∝ Temperature (T) (from figure)
Ans. (b) : Given, mass of an ideal diatomic gas = 1 We know that,
mole ∆W PdV nRdT R
= = = ….(i)
Qabsorbed nCP dT nCP dT CP
Now, CP – CV = R ….(ii)
CP

CV
CP
We know that, from 1st law of thermodynamics- CV = ….(iii)
γ
Q=U+W ….(i)
From equation (ii) and (iii), we get-
or, U=Q–W
 1
For an ideal gas, dU = nCVdT CP 1 –  =R
We know that,  γ
CP – CV = R ….(ii) R  1
= 1 −  ….(iv)
and
CP
=γ ….(iii) CP  γ 
CV Putting these values in equation (i), we get-
Form equation (ii) and (iii), we get ∆W 1 1 3 2
=1– =1– =1– =
R CP γ 5 5 5
CV =
γ –1 3

637
294. Thermodynamic processes are indicated in the 296. A sample of 0.1 g of water at 100°C and normal
following diagram pressure (1.013 × 105 Nm–2) requires 54 cal of
heat energy to convert to steam at 100°C. If the
volume of the steam produced is 167.1 cc, the
change in internal energy of the sample, is
(a) 42.2 J (b) 208.7 J
(c) 104.3 J (d) 84.5 J
NEET-2018
Ans. (b) : Given that, mass of water = 0.1g, volume of
water (V1) = 0.1 gm = 0.1 cc = 0.1 × 10–6 m3, volume of
steam (V2) = 167.1×10–6 m3
∴ Work done by the system (∆W) = PdV
= P [Vsteam – Vwater]
= 1.0013×105 [167.1×10–6 – 0.1×10–6]
= 1.013 × 10–1 [167.1–0.1]
(a)P→ a, Q→ c, R→ d, S→ b = 1.013 × 167 × 10–1
(b)P→ c, Q→ a, R→ d, S→ b = 169.171 × 10–1
(c)P→ c, Q→ d, R→ b, S→ a = 16.9171 J
(d)P→ d, Q→ b, R→ a, S→ c Heat supply to the system (Qs) = 54 cal = 54 × 4.18 =
225.72J
NEET-2017
Now, from 1st law of thermodynamics-
Ans. (b) : ∆Q = ∆U + ∆W
or ∆U = ∆Q – ∆W
∆U = 225.72 – 16.917
∆U = 208.8J
297. Two cylinders A and B fitted with pistons
We know that, PVn = constant contain equal amounts of an ideal diatomic gas
If n = 0, process is an isobaric at 300 K. Piston A is free to move and piston B
is fixed. Same amount of heat is given to the
n = ∞, process is an isochoric gases in the two cylinders. Temperature of the
n = 1, process is an isothermal gas in cylinder A increases by 30 K. Then,
n = 1.4, process is an adiabatic increase in temperature of the gas in the
1< n <1.4, process is polytropic. cylinder B is (γ=1.4 for diatomic gas
295. The P-V diagram for an ideal gas in piston (a) 24 K (b) 36 K
cylinder assembly undergoing a (c) 54 K (d) 42 K
thermodynamic process is shown in the figure. IIT 1998
The process is AP EAMCET(Medical)-2013
AP EAMCET-25.09.2020, Shift-II
Ans. (d) : Given,

(a) Adiabatic (b) Isochoric


(c) Isobaric (d) Isothermal
NEET OCT-2020
If piston is free to move then pressure is uniform in the
Ans. (c) :
cylinder therefore pressure remains constant therefore
process is isobaric.
So, ∆QA = nCP ∆TA ….(i)
If piston is stopped by stopper or fixed then the volume
of the system remains constant therefore process is
isochoric.
According to the P-V diagram given in question for an So, ∆QA = nCV ∆TB ….(ii)
ideal gas, the pressure remains constant during the According to the question same amount of heat is given
process, so it is isobaric process. to both the cylinder.

638
∆QA = ∆QB dW = 4.5 × 105 (2 – 0.5)
nCP ∆TA = nCV ∆TB = 4.5 × 1.5 × 105
= 6.75 × 105 = 6.75×102 kJ
C  Now from equation (i)
∆TB =  P  ∆TA
 CV  ∆U = dQ – dW
∆TB = 1.4×30 ∆U = 800 – 6.75 × 102
∆TB = 42 K = 800 – 675
= 125 kJ
298. An ideal gas is heated at constant pressure and
∴ ∆U = 1.25 × 105J
absorbs amount of heat Q. If the adiabatic
exponent is γ, then the fraction of heat 300. The gases A and B having same pressure P,
absorbed m raising the internal energy and volume V and temperature T are mixed. If the
performing the work, is: mixture has volume and temperature as V and
1 1 T respectively, the pressure of mixture is :
(a) 1− (b) 1+ (a) 2P (b) P
γ γ
P
2 2 (c) (d) 4P
(c) 1− (d) 1+ 2
γ γ AP EAMCET(Medical)-1998
AP EAMCET(Medical)-2004 Ans. (a) : Given,
Ans. (a) : Given that, For gas A and B
Heat added at constant pressure (Q) = mCP∆T Temperature = T
Change in Internal Energy ∆U = mCv∆T Pressure = P
From 1st Law of thermodynamics, Volume = V
∆Q = ∆U + ∆W Number of mole of A = number of mole of B = n
Now, After mixture ,
∆W = ∆Q – ∆U
Temperature of mixture = T
Performing the work Volume of mixture = V
Fraction =
heat absorbed ∴Number of mole of mixture = 2n
∆W We know that
= Ideal gas equation
∆Q
PV = nRT
∆Q – ∆U
= V nR
∆Q =
T P
∆U Now, V/T is same for gas A, gas B and mixture
= 1–
∆Q ( V / T )A = ( V / T )mixture
CV nR 2nR
= 1–
CP ∴ =
P P'
1 P' = 2P
Fraction = 1 –
γ 301. When water is heated from 0°C to 10°C, its
volume:
299. A gas under constant Pressure of 4.5×105Pa
when subjected to 800kg of heat, changes the (a) decreases continuously
volume from 0.5m3 to 2.0m3. The change in the (b) first decreases and then increases
internal energy of the gas is: (c) first increases and then decreases
(a) 6.75×105 (b) 5.25×105 (d) increases continuously
5
(c) 3.25×10 (d) 1.25×105 AP EAMCET(Medical)-1999
AP EAMCET(Medical)-2002 Karnataka CET-2015
Ans. (d) : Given, Ans. (b) : When water is heated from 0°C to 10°C there
Pressure = 4.5 × 105 Pa is a point in temperature scale of 4°C. Where density of
Heat (dQ) = 800 kJ water is maximum i.e. 1000 kg/m3 so the volume of
Volume (V1) = 0.5 m3 water first decrease up to 4°C and then increase.
Volume (V2) = 2.0 m3
We know that,
Form 1st law of thermodynamics
dQ = ∆U + dW ….(i)
∴ dW = P(V2 – V1) .....(ii)

639
302. A gas at temperature 27oC and pressure 30 atm From first law of thermodynamic,
is allowed to expand to 1 atm pressure. If the ∵ ∆Q = ∆U + W {W = 0}
volume becomes 10 times its initial volume, the
final temperature becomes: ∆Q = ∆U
(a) 1000C (b) 373 K 305. Heating of water at atmospheric pressure to
(c) 3730C (d) –1730C covered under the thermodynamic process
AP EAMCET(Medical)-1997 known as
(a) isobaric (b) isochoric
Ans. (d) : Given,
(c) isoentropic (d) isothermal
Initial temperature (T1) = 27°C = 300K
SRM JEE - 2014
Initial pressure (P1) = 30atm
Final pressure (P2) = 1atm Ans. (a) : Heating of water at atmospheric pressure to
covered under the thermodynamic process known as
Initial volume = V1
isobaric process because as heating of water is done at
Final volume = 10V1 uniform pressure through out the heating process:
We know that, QR=Heat rejected
Ideal gas equation, PV = nRT QS=Heat supply
PV
= Constant
T
For state 1 and 2
PV P V
∴ 1 1= 2 2
T1 T2
30 × V1 1× 10V1
=
300 T2 306. A monoatomic gas is suddenly compressed to
T2 = 100K 1
of its volume adiabatically. The pressure of
∴ T2 = (100 – 273)°C 8
T2 = –173°C the gas now to that of its original pressure is :
303. When water is heated from 0°C to 20° C, how (a) 8 times (b) 16 times
does its volume change? (c) 32 times (d) 128 times
(a) It shall increase AP EAMCET(Medical)-1998
(b) It shall decrease Ans. (c) : Given,
(c) It shall first increase and then decrease Initial volume of monoatomic gas = V1
(d) It shall first decrease and then increase V
Final volume of monoatomic gas = 1
NDA (I) 2008 8
Ans. : (d): When water is heated from 0oC, its volume Process is compressed adiabatically
decrease because its density increases and this effect We Know that,
lasts until the temperature reaches 4oC. This because the γ for monoatomic gas = 1.67
density of water is maximum at 4oC. After this point,
the density of water decreases and the volume increases.
304. In isochoric process :
(a) ∆U = ∆Q (b) ∆Q = ∆W
(c) ∆U = ∆W (d) None of these
BCECE-2005
Ans. (a) :

P1V1γ = P2 V2γ
γ
V 
P2 = P1  1 
 V2 
 V 
1.67
The process in which volume constant is called P2 = P1  1 
 1 
isochoric process. 
 8 V1 
We know,
W = PdV P2 = P1 (8)1.67
W=0 {∆V = 0} P2 = 32 × P1

640
γ−1
(D) Isobaric Process  16  T1 300
  = =
8 T2 200
307. In a cyclic process, the amount of heat given to (2)γ – 1 = 1.5 .....(ii)
a system is equal to From equation (i),
γ−1
(a) net increase in internal energy  VD  300
(b) net work done by the system   = = 1.5
 2  200
(c) net decrease in internal energy γ−1
 VD  γ−1 VD
(d) net change in volume   = (2) ⇒ =2
(e) net change in pressure  2  2
Kerala CEE- 2013 VD = 4 unit
Ans. (b) : Heat equation, 309. In the V-T diagram shown in adjoining figure,
∆Q = ∆U + ∆W what is the relation between P1and P2?
∆U = 0 [cyclic process]
∆Q = ∆W
The amount of heat given to a system is equal to net
work done by the system.
308. In the following P-V diagram of an ideal gas,
two adiabates cut two isotherms at T1 = 300K
and T2 = 200K. The value of VA = 2 unit, VB = 8
unit, VC = 16 unit. Find the value of VD. (a) P2 = P1 (b) P2 < P1
(c) P2 > P1 (d) insufficient data
VITEEE-2013
Ans. (b) : In an isobaric process

(a) 4 unit (b) < 4 unit


(c) > 5 unit (d) 5 unit
BITSAT-2018 P = constant
Ans. (a) : Given, VA = 2 unit, VB = 8 unit, VC = 16 unit V ∝T
TB = TA = T1 = 300 K  nR 
TC = TD = T2 = 200 K
V=  T
 P 
 nR 
∴ V – T graph is a straight line with slope  .
 P 
Since, (slope)2 >(slope)1
So, we can say that
P2 < P1
310. Two moles of a monoatomic gas undergoes an
isobaric process. If the gas temperature is
increased by 20 °C then the heat absorbed by
For adiabatic process,
the gas is (Take R = 8.3 J/K. mol)
TV(γ – 1) = Constant (a) 630 J (b) 430 J
Process (A – D) (c) 830 J (d) 960 J
TA VA( γ−1) = TD VD( γ−1) TS EAMCET 30.07.2022, Shift-I
γ−1 Ans. (c) : Given,
 VD  TA
V  = T .....(i) ∆T=20oC
 A D
R=8.3 J/K.mol
Process (B – C), n=2
TB VBγ−1 = TC VCγ−1  1
CP ×  1 –  = R
T1 (8) γ−1 = T2 (16) γ−1  γ

641
R From equation (i) and (ii)
CP = Now,
1– 1
γ w2 = w1 ln2
R  7 
= (∵ For monoatomic gas, γ = 5/3) 313. A diatomic gas  Cp = R  does 200 J of work
1– 3  2 
5
5R when it is expanded isobarically. The heat
CP = given to the gas in the process is
2
5R (a) 600 J (b) 800 J
Q = n CP ∆T = 2 × × 20 = 5 × 8.3 × 20 = 830J
2 (c) 900 J (d) 700 J
311. In which process the PV indicator diagram is a TS EAMCET 18.07.2022, Shift-II
straight line parallel to volume axis. Ans. (d) : Given,
(a) Isothermal (b) Isobaric
CP C
(c) Irreversible (d) Adiabatic = γ ⇒ CV = P
MP PMT-2009 C V γ
Ans. (b) : A isobaric process is a thermodynamic 7
process in which the pressure remains constant. This is CP = R , For diatomic
2
usually obtained by allowing the volume to expand or
contract in such a way to neutralize any pressure C = 7R = 7R
2γ 2 × 1.4
v
changes that would be caused by heat transfer.
We know that,
From 1st law of thermodynamics
dQ = dU + dW
∴ dQ = mCv ∆T + dW
7R
=m× ∆T + PdV
2 × 1.4
Given that the P - V indicates diagram shows a  70 
=   PdV + PdV
straight line parallel to volume axis  2 ×14 
The slope of the graph
7
∆P = PdV
=0 2
∆V
∆P = 0 7
dQ = × 200
Hence we can say that pressure remains constant. 2
312. Two moles of a gas is expanded to double its dQ = 700J
volume by two different processes. One is 314. A monoatomic gas does 100 J of work when it
isobaric and the other is isothermal. If w1 and
w2 are the works done respectively, then is expanded isobarically. How much of heat is
w given to the gas in the process
(a) w 2 = 1 (b) w 2 = w 2 (a) 150 J (b) 200 J
ln2
(c) w 2 = w1ln2 (d) V13/2 : V23 / 2 (c) 250 J (d) 300 J
TS EAMCET 18.07.2022, Shift-I
TS EAMCET (Engg.)-2016
Ans. (c) : We know that, Ans. (c) : Given,
For Isobaric process, Gas is mono atomic
Work done (w1) = PdV 5
= P(V2 – V1) ∴ γ=
3
= P(2V – V)
CP – CV = R [Mayer’s equation]
w1 = PV = nRT ….(i)
For Isothermal process, CP

CV
V 
Work done (w2) = nRT ln  2 
CP = γ CV
 V1 
 2V  CV (γ – 1) = R
= nRT ln   R
 V  CV =
w2 = nRT ln2 ….(ii) γ –1

642
Now from 1st Law of thermodynamics Ans. (b) : Given
dQ = dU + dW
= mCVdT + PdV
m × RdT
= + PdV
γ –1
PdV
dQ = + PdV
5
–1
3 We know that,
3 Area under the P– V diagram is equal to work done.
= PdV + PdV
2 1
∆W = × 300 × 3 = 450J
5 2
dQ = × PdV
2 317. In an isobaric process of an ideal gas. The ratio
5 of heat supplied and work done by the system
dQ = × 100
2   Q 
dQ = 250J i.e.  W   is
  
315. If the temperature of 50ml of gas at 27ºC raised
γ −1
to 57ºC at constant pressure, the final volume (a) (b) γ
of the gas is γ
(a) 23.7 ml (b) 25.0 ml γ
(c) 53.7 ml (d) 55.0 ml (c) (d) l
γ −1
TS EAMCET 03.05.2018, Shift-I
AMU-2001
Ans. (d) : Given,
Initial volume (V1) = 50 ml Ans. (c) : Given,
Initial temperature (T1) = 27°C = 300 K The process is Isobaric
Final Temperature (T2) = 57° C = 330 K Now, for Isobaric process
Ideal gas equation Q = mCPdT ….(i)
PV = nRT W = PdV ….(ii)
For constant pressure process We know that,
V CP – CV = R ….(iii)
= Constant
T CP
=γ ….(iv)
V V CV
∴ 1= 2
T1 T2 From equation (iii) and (iv), we get
50 V2 γR
= CP =
300 330 γ –1
V2 = 55 ml
Q mC P dT
316. A thermodynamic system is taken from an ∴ =
original state D to an intermediate state E by W PdV
linear process shown in the figure. Its volume is  γR 
then reduced to the original volume from E to m×  dT
 γ –1
F by an isobaric process. The total work done =
by the gas from D to E to F will be. PdV
 γ  mRdT
=  
 γ – 1  PdV
For Ideal gas
PV = mRT
Q γ  PdV 
∴ =  
W γ – 1  PdV 
Q γ
=
W γ –1
(a) – 450 J (b) 450 J 318. A sample of gas expands from volumeV1 to V2.
(c) 900 J (d) 1350 J The amount of work done by the gas is greatest
JEE Main-29.07.2022, Shift-II when the expansion is
643
(a) adiabatic 200
(b) isobaric T=
2
(c) isothermal T = 100K
(d) Equal in all above cases ∴Initial temperature of gas = 100 K.
BITSAT-2009 320. Work done by 0.1 mole of gas at 27ºC when it
AIPMT-1997 expands to double its volume at constant
AIIMS 1998 pressure is (assume R = 2 cal/mol-K)
JIPMER 2000 (a) 600 cal (b) 42 cal
Ans. (b) : (c) 60 cal (d) 546 cal
EAMCET-1994
Ans. (b) : Work done at constant pressure,
V 
W = 2.3026 nRT log10  2 
 V1 
 2V 
= 2.3026 × 0.1× 2 × 300log10  
 V 
= 2.3026 × 0.1 × 600 × 0.3010 ≃ 42 cal
For expansion process, P-V diagram gives work done. 321. When heat energy of 1500 J is supplied to a gas
Area is maximum under the isobaric curve. at constant pressure, 2.1 × 105 Nm–2, there was
an increase in its volume equal to 2.5 × 10–3 m3.
So, work done is maximum during an isobaric
The increase in its internal energy in joule is
expansion curve.
(a) 450 J (b) 525 J
Wisobaric > Wisothermal > Wadiabatic
(c) 975 J (d) 2025 J
319. The temperature of a gas contained in a closed EAMCET-1999
vessel increases by 2oC when the pressure is
increased by 2%. The initial temperature of the Ans. (c) : Given,
gas is: Heat energy supplied to a gas (Q) = 1500 J
(a) 200 K (b) 100 K Pressure (remains constant) = 2.1×105 Nm–2
(c) 200oC (d) 100oC Increase in its volume = 2.5×10–3 m3
AP EAMCET(Medical)-2006 We know that,
Ist law of thermodynamics,
Ans. (b) : Given,
∆Q = ∆U + ∆W
Temperature of a gas increased (∆T) = 2°C
∴ ∆W = PdV
2 = 2.1 × 105(2.5×10–3)
Pressure increased = 2% = = 0.02
100 = 21×25
We know that, = 525 J
Ideal gas equation ∴ ∆U = ∆Q – ∆W
PV = nRT ….(i) = 1500–525
According to the question ∆U = 975J
Let,
Initial temperature (T1) = T 322. When an ideal diatomic gas is heated at
Final temperature (T2) = T + 2 constant pressure, fraction of heat energy
Initial pressure (P1) = P supplied that increases the internal energy of
Final pressure (P2) = 1.02P the gas is
Now from Ideal gas equation 5 7
(a) (b)
P 7 5
= constant 3 5
T (c) (d)
P 1.02P 5 3
= 2
T T+2 (e)
T+2 3
= 1.02 Kerala CEE 2021
T
J&K CET 2006
2
1 + = 1.02 AMU-2002
T RPMT 2000
2 UPSEE 1998
= 0.02
T IIT 1990
644
Ans. (a) : Given, Ans. (e) According to the first law of thermodynamics,
For Ideal diatomic gas (γ)=1+2/f=1+2/5=7/5 ∆Q = ∆W + ∆U
R 5 In an isochoric process, the volume remains constant
∴ CV = = R (dV = 0)
γ −1 2
So work done (∆W) = PdV = 0
γR 7
CP = = R Hence, ∆Q = 0 + ∆U
γ −1 2 ∆Q = ∆U
Fraction of heat energy which increase internal energy Heat exchanged = change in internal energy.
326. The Volume of 1m3 of gas is doubled at
 dU  nC V dT C V 5 R 5 atmospheric pressure. The work done at
 dQ  = nC dT = C = =
2
constant pressure will be
 P 7
2 R 7
P P
(a) zero (b) 105 cal
323. In the Carnot engine when the heat is taken 5
(c) 10 J (d) 105 erg
from the source, then the temperature of the SRM JEE-2018
source Ans. (c) : Isobaric process– An isobaric process is a
(a) remains constant process occurring at constant pressure.
(b) does not remain constant For isobaric process we write ∆U = ∆Q – ∆W
(c) decreases = ∆Q – P (V2–V1)
(d) increases Work done in isobaric process is given as W
TS-EAMCET-09.09.2020, Shift-2 = P (Vf – Vi)
Ans. (a) : In the Carnot engine when the heat is taken = 1×105 (2–1)
from the source, then the temperature of the source = 105 J
remains constant because source and sink are 327. A cylinder of fixed capacity of 44.8 litres
thermostats maintained at particular constant contains helium gas at standard temperature
temperature. and pressure. The amount of heat needed to
raise the temperature of gas in the cylinder by
(E) Isochoric Process 20.0°C will be
(Given gas constant R = 8.3 JK–1-mol–1)
324. In isochoric process (a) 249 J (b) 415 J
(a) ∆W = 0 (b) ∆U = 0 (c) 498 J (d) 830 J
(c) ∆Q = 0 (d) ∆T = 0 JEE Main-29.06.2022, Shift-I
AP EAMCET-25.08.2021, Shift-I Ans. (c) : Given, Volume = 44.8 lit = 44.8×10–3m3
AP EAMCET-24.09.2020, Shift-I We know that,
UP CPMT-2005 One mole of any ideal gas at STP occupies a
AMU-2002 volume of 22.4 litre therefore, cylinder of fixed capacity
44.8 litre must contain 2 moles of helium at STP.
Ans. (a) :
As helium is monatomic gas.
• The work done by an ideal gas during an isochoric
process is zero. 3
For helium, C v = R
• In Isochoric process, volume remains constant. 2
Some energy with its environment, the process is Now, Heat needed to raise the temperature
known as an isochoric process. Q = nCvdT
So, When volume (V) = Constant 3
Q = 2 × R × 20
Then dV = 0 2
and = 3×8.3×20 = 498 J
work done (W) = ∫ PdV = 0 328. In an isochoric process, if heat is supplied to
gas the pressure will
It mean W = 0 (a) Increase
325. A process in which the amount of heat supplied (b) Decrease
to the system goes fully to change its internal (c) Remains same
energy and temperature is (d) Varies exponentially
(a) adiabatic process (b) cyclic process HP CET-2018
(c) isobaric process (d) isothermal process Ans. (a) : From ideal gas equation
(e) isochoric process PV = nRT
Kerala CEE 04.07.2022 ∴ For Isochoric process (V=C),
645
P∝T 831 × 72 × 105
⇒ P=
P1 P2 36
=
T1 T2 P = 831×2×105
For isochoric process temperature is directly P = 1662×105 Pa
proportional to pressure. If we supply heat, temperature P = 16.62 × 107 Pa
increases so pressure also increases.
331. The temperature of 5 moles of a gas at constant
329. A gas performs minimum work (in magnitude)
volume is changed from 100ºC to 120ºC. The
when the process is ……… .
change in internal energy is 80 J. The total heat
(a) Adiabatic (b) Isothermal
capacity of the gas at constant volume will be in
(c) Isobaric (d) Isochoric J/K
AP EAMCET-25.09.2020, Shift-I (a) 8 (b) 4
Ans. (d) : As we know that work done by a (c) 0.8 (d) 0.4
thermodynamic system is zero for isochoric process. EAMCET-2003
Ans. (b) : Given,
Number of moles (n) = 5
Change in temperature (∆T) =(120–100)= 20°C
Internal Energy (∆U)= 80J
We know that,
Ist Law of thermodynamics,
∆Q = ∆U + ∆W
Process OD is Isochoric process and area under curve in Now, for constant volume process work done is zero
P-V diagram is equal to work done.
∆W = 0
∴ Work done for isochoric process = P(V2–V1)
∴ V2 = V1 ∴ ∆Q = ∆U
But question is asked total heat capacity
Work done = 0
∆Q
∴ Total heat capacity =
330. A closed copper vessel contains water equal to ∆T
half of its volume. When the temperature of the
80
vessel is raised to 447ºC. The pressure of steam = = 4J/K
in the vessel is: (Treat steam as an ideal gas. 20
Universal gas constant = 8310 J/mol-K, mole-
density of water is 1000 kg/m3, molecular (F) Isothermal Process
weight of water = 18)
(a) 33.24 × 107 Pa (b) 16.62 × 107 Pa 332. Heat is given to an ideal gas in an isothermal
7
(c) 10.31 × 10 Pa (d) 8.31 × 107 Pa process.
AP EAMCET(Medical)-2000 A. Internal energy of the gas will decrease.
B. Internal energy of the gas will increase.
Ans. (b) : Given,
Temperature raised (T) = 447°C = 447+273 = 720K C. Internal energy of the gas will not change.
Universal gas constant (R) = 8310 J/mol-K D. The gas will do positive work.
E. The gas will do negative work.
Density of water (ρ) = 1000 kg/m3
Choose the correct answer from the options
Molecular weight of water = 18
given below:
Mass of water in the vessel = ρ × V/2
(a) C and D only (b) B and D only
Now, from ideal gas equation
(c) A and E only (d) C and E only
PV = nRT
JEE Main-30.01.2023, Shift-I
m
∴ PV = RT Ans. (a) : ∆Q = ∆U + ∆W
M
ρ× V ∵ In isothermal process
PV = × RT
18 × 2 ∆Τ = 0
1000 × 8310 × 720 ∴ ∆U = 0
P=
36 ∆Q = ∆W

646
333. A closed gas cylinder is divided into two parts
by a piston held tight. The pressure and volume
of gas in two parts respectively are (P, 5V) and
(10P, V). If now the piston is left free and the
system undergoes isothermal process, then the
volume of the gas in two parts respectively are
(a) 2V, 4V (b) 3V, 3V
(c) 5V, V (d) 4V, 2V (a) A (b) B
(e) 2.5V, 3.5V (c) C (d) D
Kerala CEE 2007 (e) E
Ans. (a) : Kerala CEE 2004
Ans. (b) : Best representation of an isothermal process
be like hyperbola. Hence option (b) is right.

For isothermal process


T = constant
PV = nRT
PV = constant
Piston moves towards lower pressure until both side 335. The condition dQ = dW holds good in which of
pressure will same. the following?
P × 5V + 10P × V = P' 6V (a) Adiabatic process (b) Isothermal process
5PV + 10PV = 6 P' V (c) Isochoric process (d) Isobaric process
15PV = 6P'V AP EAMCET (21.09.2020) Shift-II
15P Ans. (b) : In isothermal process there is no change in
P' = (where, P' = common pressure) internal energy (∆U = 0).
6
For Ist Part- First law of thermodynamics,
P × 5V = P' × V1 dQ = dW + dU
15P dU = 0
5V × P = × V1 dQ = dW
6
V1 = 2V (for Part-I) 336. The first operation involved in a carnot cycle is
(a) isothermal expansion
For IInd Part-
(b) adiabatic expansion
Boyle's law
(c) isothermal compression
5 (d) adiabatic compression
10P × V = P × V2
2 BITSAT-2005
V2 = 4V (for Part-II) Ans. (a) : Carnot cycle-
334. Which of the accompanying P-V diagrams best
represents an isothermal process?

Hence, isothermal expansion is first process in carnot


cycle.
337. In the equation PVγ = constant, the value of γ is
unity. Then the process is
(a) isothermal (b) adiabatic
(c) isobaric (d) irreversible
BITSAT-2010
647
Ans. (a) : General process: PV γ = constant Ans. (d) : In isothermal process:
γ Process Isothermal curves never intersect each other because
if they intersect then at the point of intersection for
0 Isobaric process
two different temperature the volume and pressure of
1 Isothermal process an ideal gas will be same. This is not possible.
1.4 Adiabatic process Compression or expansion takes place slowly not
∞ Isochoric process rapidly so given reason is incorrect.
338. For an isothermal expansion of a perfect gas, Hence, both assertion reason are incorrect.
∆P 341. A diatomic gas which has initial volume of 10
the value of is equal to liter is isothermally compressed to 1/15th of its
P
original volume where initial pressure is 105
∆V ∆V
(a) −γ1/ 2 (b) − Pascal. If temperature is 27ºC them find the
V V work done by gas.
∆V ∆V (a) –2.70×103 J (b) 2.70×103 J
(c) −γ (d) −γ 2
V V (c) –1.35×103 J (d) 1.35×103 J
BITSAT-2012 AIIMS-26.05.2018(E)
Ans. (b) : Differentiate PV = constant V 
Ans. (a) : W = P1V1 ln  2 
P∆V + V∆P = 0
 V1 
∆P – ∆V
= 1
P V =105 × 10 × 10–3ln  
 15 
339. One mole of O2 gas having a volume equal to
22.4 Litres at 0°C and 1 atmospheric pressure W = −2.70 × 10 J3

in compressed isothermally so that its volume 342. The change in the entropy of a 1 mole of an
reduces to 11.2 litres. The work done in this ideal gas which went through an isothermal
process is- process from an initial state (P1, V1, T) to the
final state (P2, V2, T) is equal to
(a) 1672.5 J (b) 1728 J
(a) zero (b) R ln T
(c) –1728 J (d) –1572.5 J
BITSAT-2014, 2009 V1 V
(c) R ln (d) R ln 2
Ans. (d) : V1 = 22.4 L, V2 = 11.2 L V2 V1
Mole of O2 n = 1, R = 8.314 J/mol K AIIMS-2010
P = 1 atm Ans. (d) : Work done by n moles of a gas when its
volume changes from V1 to V2 is.
V 
W = 2.303nRT log10  2  V
 V1  W = nRT ln 2
V1
 11.2  For n = 1
= 2.303 × 1× 8.314 × (273) log10  
 22.4  V
W = RT ln 2
W = −1572.5 J V1
340. Assertion: The isothermal curves intersect each For an isothermal process, ∆U = 0
other at a certain point. V
∴ ∆Q = ∆U+W = 0 +RT ln 2
Reason: The isothermal changes takes place V1
rapidly, so the isothermal curves have very
V
little slope. ∆Q = RT ln 2
(a) If both Assertion and Reason are correct and V1
the Reason is a correct explanation of the dQ V
Assertion. ∵ Entropy = = R ln 2
T V1
(b) If both Assertion and Reason are correct but
343. An ideal gas is subjected to an isothermal
Reason is not a correct explanation of the expansion such that its volume changes from Vi
Assertion. to Vf and pressure from Pi to Pf the work done
(c) If the Assertion is correct but Reason is on the gas is:
incorrect. V V
(d) If both the Assertion and Reason are (a) W = + nRT log f (b) W = –nRT log f
Vi Vi
incorrect.
(e) If the Assertion is incorrect but the Reason is P P
(c) W = nRT log f (d) W = –nRT log f
correct. Pi Pi
AIIMS-2008 AIIMS-2011
648
Ans. (d) : The work done in expansion of gas 345. A gas is compressed isothermally the r.m.s.
V P velocity of its molecules
W = nRT ln f = nRT ln i (a) increases
Vi Pf
(b) decreases
The work done on the gas will be negative.
(c) first increased and then decreases
 pf  (d) remains the same
∴ W = – nRT ln  
 pi  MHT-CET 2020
344. A monoatomic gas of pressure ‘P’ having MHT-CET 2009
volume ‘V’ expands isothermally to a volume Ans. (d) : R.M.S. velocity of gas,
‘2V’ and then adiabatically to a volume '16V'.
The final pressure of the gas is (ratio of specific γRT
Vrms =
5 M
heats = )
3 Vrms ∝ T
P P
(a) (b) If a gas is compressed isothermally then the r.m.s
16 8 velocity of the molecules remain the same.
P P
(c) (d) 346. An ideal gas undergoes an isothermal change in
32 64 volume with pressure, then
MHT-CET 2020
Assam CEE-2019 (a) PV = constant (b) (PV)γ = constant
Ans. (d) : Given, (c) PVγ = constant (d) PγV = constant
J&K CET- 2001
Ans. (a) : For an ideal gas, the product of pressure and
volume (P-V) is a constant, if the gas is kept in
isothermal condition.
In other words,
From ideal gas law, PV = nRT.
PV = nRT = constant
∴ PV = constant
347. Which of the following is a slow process?
Case-I:- (a) Isothermal (b) Adiabatic
P1 = P, V1 = V, V2 = 2V (c) Isobaric (d) None of these
We know that, J&K CET- 2000
P 1V 1 = P 2V 2 Ans. (a) : Isothermal processes are necessarily slow
PV = P2 (2V) because they require heat transfer to remain at the same
P temperature as being in thermal equilibrium with a
P2 = reservoir.
2
Case-II:- 348. Name the process in which Boyle’s law is
V2 = 2V, V3 = 16V applicable?
We know that, (a) Adiabatic process (b) Isochoric process
P2 V2γ = P3V3γ (c) Isobaric process (d) Isothermal process
P
  ( 2V ) = P3 (16V )
γ γ J&K-CET-2019
2 Ans. (d) : Boyle's law is applicable to an isothermal
γ γ process where temperature remains constant.
P  2V  P  1 
P3 =   =   Boyle's law, Temperature = constant
2  16V  28
PV = nRT
∵ For monoatomic gas, γ = 5/3
PV = constant
5/3
P
P3 =    1  P 1 V 1 = P 2V 2
2  23 
5
P1
=  
22
P
P3 =
64
649
349. The volume of an ideal diatomic gas is doubled 352. The pressure of a gas kept in an isothermal
isothermally. The internal energy container is 200 kPa. If half the gas is removed
(a) is doubled from it, the pressure will be:
(b) is halved (a) 100 kPa (b) 200 kPa
(c) is increases four times (c) 400 kPa (d) 800 kPa
(d) is remains unchanged MP PET-2012
UPSEE - 2009 Ans. (a) Let the number of mole n and then become n/2.
Ans. (d) : No change in the internal energy of an ideal n
n1=n, n2= and P1 = 200 kPa
gas, but for real gas internal energy increases, because 2
work is done against intermolecular forces. We know that,
In is isothermal change temperature remain constant. PV = nRT ….(i)
Hence internal energy will not change ∴ P∝n
350. Work done per mol in an isothermal change is P2 n 2
=
V V P1 n1
(a) RT log10 2 (b) RT log10 1
V1 V2 n
V2 V1 P2 2 1
(c) RT loge (d) RT loge s = =
V1 V2 P1 n 2
UP CPMT-2010 1
P2 = P1
UP CPMT-2009 2
1
Ans. (c) : Work done = ∫ P.dV …... (i) = × 200
2
PV= nRT = 100kPa
n = 1 mole
353. In isothermal process, which of the following is
RT not true?
P=
V (a) Temperature remains constant
Work done (b) Internal energy does not change
dV
V2 (c) No heat enters or leaves the system
= RT ∫ (d) none of the above
V
V1

= RT[ln V]VV12 JIPMER-2005


Ans. (c) :We know that,
= RT (lnV2 – lnV1) In an isothermal process, Temperature is constant
V i.e. PV = constant
= RT ln 2
V1 According to joule’s law, internal energy remains
constant for isothermal process.
{ln = log e } In isothermal process, heat can enters or leaves the
V  system, to keep the temperature constant. So, statement
∴ Work done = RT loge  2  (c) is wrong.
 V1  354. The volume of an ideal gas is doubled in an
351. In an isothermal process if heat is supplied to isothermal process. Then, which of the
an ideal gas, then following is true ?
(a) The internal energy of the gas will decrease (a) Work done by the gas is positive
(b) The internal energy of the gas will increase (b) Work done by the gas is negative
(c) The gas will do positive work (c) Internal energy of the system decreases
(d) The gas will do negative work (d) Internal energy of the system increases
TS EAMCET (Medical) 09.08.2021, Shift-I JIPMER-2016
Ans. (c) : In Isothermal process the temperature is Ans. (a) : Given that,
constant. V1 = V, V2 = 2V
The internal energy is a state function dependent on For isothermal ∆U = 0
temperature. Hence, the internal energy change is zero. dW = P∆V
For an ideal gas, in an isothermal Process – dW = P ( V2 − V1 )
∆U = 0 = Q – W dW = P ( 2V − V )
So, W = Q dW = PV
Hence, gas will do positive work. Hence, the work done by the gas is positive.
650
355. The slope of isothermal and adiabatic curves
are related as
(a) Isothermal curve slope = adiabatic curve
slope
(b) Isothermal curve slope = γ × adiabatic curve
slope
(c) Adiabatic curve slope = γ × Isothermal curve (a) Nothing can be predicted
slope (b) m1 < m2
1 (c) m1 = m2
(d) Adiabatic curve slope = × Isothermal
2 (d) m1 > m2
curve slope JIPMER-2016
UP CPMT 1971 Ans. (b) :
BHU 1996
MHT CET 1999
UPSEE 2000
RPMT 2003
JIPMER-2016
BITSAT-2008
Ans. (c) :In the slope of isothermal curve,
T = constant
PV = nRT From the ideal gas equation for the slope m1 and m2
PV = constant PV = nRT
m
PdV VdP PV = RT
+ =0 M
dV dV
For slope (I) :-
dP  –P 
=  m RT
dV  V  P= 1 ….. (i)
M V1
Slope of isothermal curve on P-V diagram = −P( ) V For slope (II):-
m RT
Slope of adiabatic curve on P-V diagram P= 2 …… (ii)
M V2
PV γ = C
From equation (i) and (ii),
Differentiating with respect to V,
m1 RT m
d (Vγ ) dP M V1 MV2
= 2 RT
P + Vγ =0
dV dV m1 m 2
=
γ dP
d ( Vγ ) V1 V2
V = –P
dV dV V∝m
γ dP γ –1 From the figure V2 > V1
V = –PγV
dV ∴ m2 > m1
dP –PγV γ –1 357. Slope of an isothermal curve is always
= γ
= –PγV γ –1– γ (a) equal to adiabatic curve
dV V
(b) greater than adiabatic curve
dP
= –PγV –1 (c) less than adiabatic curve
dV (d) cannot be determined
dP P
∴ = –γ WB JEE-2007
dV V Ans. (c) : We know that, for Isothermal process
Hence, adiabatic slope = γ times the slope of isothermal PV = constant
curve. Differentiating both side
356. Two different isotherms representing the PdV + VdP = 0
relationship between pressure p and volume V VdP = –PdV
at a given temperature of the same ideal gas dP − P
=
are shown for masses m1 and m2 , then dV V
651
For adiabatic Process 360. A graph of pressure (P) against volume (V) of
PVγ = Constant an ideal gas undergoing an isothermal process
is:
Differentiating both side
(a) a straight line passing through the origin
PγVγ–1dV+VγdP = 0 (b) a parabola
VγdP = –γPVγ–1dV (c) a rectangular hyperbola
dP −γPV γ−1 (d) a straight line parallel to pressure axis
=
dV Vγ AP EAMCET-23.09.2020, Shift-II
dP −γP J&K-CET-2017
=
dV V Ans. (c) : We know that
 dP   dP 
∴   < 
 dV isothermal  dV  Adiabatic
∴ Slope of an Isothermal curve is always less than
adiabatic curve.
358. When 50l of air at STP (Standard Temperature
and Pressure) is compressed isothermally to
10l, the amount and direction of heat exchange
of air is
[Use Patm = 100kPa and ln (0.2) = –1.61] Ideal gas equation
(a) 8.05 kJ flows out (b) 8.05 kJ flows in PV = nRT .....(i)
(c) 5 kJ flows out (d) 5 kJ flows in For isothermal process
TS EAMCET 02.05.2018, Shift-II PV = Constant
Ans. (a) : Given, Now, Differentiating both side
Initial volume of air at STP=50l=50×10–3 m3 PdV+VdP = 0 .....(ii)
Final Volume of air = 10l = 10×10–3 m3 dP V
=−
Patm = 100 kPa dV P
Now, Process is Isothermally compressed Note: differential equation of rectangular hyperbola.
∴ From 1st law of thermodynamics 361. An ideal gas expands isothermally from volume
∆Q = ∆U + ∆W ....(i) V1 to volume V2. It is then compressed to the
∴ ∆Q = ∆W original volume V1 adiabatically. If p1, p2 and
Now, for Isothermal process W represent the initial pressure, final pressure
and the net work done by the gas respectively
V  during the entire process, then
∆W = Pi Vi ln  f 
 Vi  (a) p1 > p2, W = 0 (b) p1 > p2, W > 0
(c) p2 > p1, W > 0 (d) p2 > p1, W < 0
 10 × 10−3 
= 100 × 50 × 10−3 × ln  −3 
AP EAMCET -2010
 50 × 10  Ans. (d) : Given,
= 5 × ln(0.2) I II
= –5×1.61
∆Q = −8.05kJ
Negative sign shows that heat rejected from the system
∴ ∆Q = 8.05kJ flows out
359. Which of the following processes is reversible?
(a) Transfer of heat by radiation
(b) Electrical heating of a Nichrome wire Isothermal Adiabatic
(c) Transfer of heat by conduction Expansion Compression
(d) Isothermal compression
TA = TB
JIPMER- 2007
Ans. (d) : For process to be reversible it must be quasi- Now,
static. For quasi static process all change take place As shown in the above figure
infinitely slowly. Isothermal process occur very slowly Process AB = Isothermal Expansion
so it is quasi- static and hence it is reversible. Process BC = Adiabatic Compression
652
From figure it can be said that, area under the curve for ∵ Isothermal compression,
isothermal expansion is lesser than area under the curve
for adiabatic compression. P 1V 1 = P 2V 2
∴ Work done is negative (W < 0). Also pressure V
1 × V = P2 ×
(P2 > P1). 2
362. Isothermal process is the graph between P2 = 2 atm
(a) Pressure and Temperature 365. Which of the following, in general, is a slow
(b) Pressure and Volume process?
(c) Volume and Temperature (a) Isothermal (b) Adiabatic
(d) P,V and Temperature (c) Isobaric (d) Isochoric
AP EAMCET-20.08.2021, Shift-I AP EAMCET-25.08.2021, Shift-I
Ans. (b) : Isothermal Process is the graph between Ans. (a) : Slow process is generally prefer to isothermal
Pressure and volume process. The transfer of heat into or out of the system
happens so slowly that thermal equilibrium is
maintained.
i.e. ∆T = 0
366. An ideal gas A and a real gas B have their
volumes increased from V to 2V under
isothermal conditions. The increase in internal
energy
(a) will be same in both A and B
363. Match the following? (b) will be zero in both the gases
Column-I Column-II (c) of B will be more than that of A
A. Isothermal process (i) No heat exchange (d) of A will be more than that of B
B. Adiabatic process (ii) Constant AIPMT-1993
temperature JIPMER 2001,2002
C. Isochoric process (iii) Constant pressure Ans. (c) : Ideal Gas (A) – Internal energy of an ideal
D. Isobaric process (iv) Constant volume gas, is the function of temperature
(a) ( A − iv ) , ( B − ii ) , ( C − i ) , ( D − iii ) dU = mcv∆T
If ideal gas compressed during isothermal process. The
(b) ( A − iv ) , ( B − i ) , ( C − ii ) , ( D − iii ) temperature remains constant
(c) ( A − ii ) , ( B − iii ) , ( C − iv ) , ( D − i ) ∴ dU = 0
(d) ( A − ii ) , ( B − i ) , ( C − iv ) , ( D − iii ) So, there will be zero increment in internal energy.
Real gas (B) – Real gas as Vander Waals gas, the
AP EAMCET-03.09.2021, Shift-II
internal energy of a Vander Waals gas is dependent on
Ans. (d) : the volume as well as the temperature.
Isothermal process → Constant temperature Thus for the real gas, internal energy will increases.
Adiabatic process → Νο Ηeat exchange Hence, Increase in internal energy of real gas B will be
Isochoric process → Constant volume more than the of ideal gas A.
Isobaric process → Constant pressure 367. A gas is taken through the cycle A→B→C→A,
364. Assume that you have an ideal gas for which γ as shown. What is the net work done by the
= 1.50, initially at 1.0 atm pressure When the gas?
gas is compressed to half its original volume.
then the final pressure. if the compression is
isothermals. is ______
(a) 4.0 atm (b) 2.8 atm
(c) 2.0 atm (d) 6.0 atm

AP EAMCET-03.09.2021, Shift-II
Ans. (c) : Given,
Ideal gas, γ = 1.50
Initial pressure, P1 = 1 atm
Initial volume, V1 = V (a) 2000 J (b) 1000 J
V V (c) Zero (d) –2000 J
Final volume, V2 = 1 =
2 2 NEET-2013
653
Ans. (b) : Net work done by the gas

(a) (b)

Work done = Area enclosed by curve


(c) (d)
1
= × (7 – 2) × 10–3 × (6 – 2) ×105
2
AIPMT-2012
1
= × 5 × 10–3 × 4 × 105 Ans. (d) : Given,
2 Isothermal process from A to B volume from V to 3V
= 10 × 10–3 × 105 And when, constant pressure process,
= 10 × 100 P = C, volume become from 3V to V
= 1000 J Thus, PV diagram,
368. A thermodynamic system is taken through the
cycle ABCD as shown in figure. Heat rejected
by the gas during the cycle is

370. During an isothermal expansion, a confined


ideal gas does –150 J of work against its
surroundings. This implies that
(a) 300 J of heat has been added to the gas
(a) 2pV (b) no heat is transferred because the process is
isothermal
(b) 4pV
(c) 150 J of heat has been added to the gas
1 (d) 150 of heat has been removed from the gas
(c) pV
2 AIPMT-2011
(d) pV Ans. (c): From the first law of thermodynamics –
AIPMT-2012 ∆Q = ∆U + ∆W
Ans. (a) : Work done in cyclic process is equal to the ∵ ∆U = mc∆T
area under the cycle also work done is positive, if cycle In case of isothermal condition
is clockwise and negative if cycle is anticlockwise. ∆U = 0
Work done (W) = area under P–V graph Therefore, ∆Q = ∆W
∆W = – (2P–P) (3V–V) Given, work against surrounding –
∆W = – 2PV ∆W = – 150 J
∴ ∆Q = – (–150) J
∆Q = ∆U + ∆W
∆Q = + 150 J
∆Q = 0 – 2PV Thus 150 J of heat has been added to the gas.
Hence, heat rejected is 2PV. 371. One mole of an ideal gas undergoes an
369. One mole of an ideal gas goes from an initial isothermal change at temperature T, so that its,
state A to final state B via two processes. If first volume V is doubled, R is the molar gas
undergoes isothermal expansion from volume constant. Work done by the gas during this
changes is
V to 3V and then its volume is reduced from3V
(a) RT log 4 (b) RT log 2
to V at constant pressure. The correct p-V (c) RT log1 (d) RT log 3
diagram representing the two processes is AP EMCET(Medical)-2008
654
Ans. (b) : Given, Ans. (a) : γ is dimensionless quantity, it is ratio of
Vi = V, Vf = 2V specific heats, hence it will not depend on temperature
So, under isothermal process work is given by the So γ for given gas remains constant.
V  5
relation W = RT log  f  374. 800 cc volume of a gas having γ = is suddenly
 Vi  3
 2V  compressed adiabatically to 100 cc. If the initial
W = RT log   pressure is P, then the final pressure will be:
 V 
W = RT log(2) P  24 
(a) (b)   P
32  5 
(G) Adiabatic Process (c) 8 P (d) 32 P
(e) 16 P
372. A hypothetical gas expands adiabatically such Kerala CEE 2004
that its volume changes from 08 litres to 27
liters. If the ratio of final pressure of the gas to AIPMT-2014
16 Ans. (d) : Given,
initial pressure of the gas is . Then the ratio
81 5
P1 = P, γ = , V1 = 800cc, V2 = 100cc
cp 3
of will be.
cv We know, PVγ = Constant
γ
3 3 P2  V1   800 
5/3
(a) (b) = =
1 2    
P1  V2   100 
1 4
(c) (d) P2 = 32P
2 3
JEE Main-31.01.2023, Shift-II 375. The ratio of the adiabatic to isothermal
elasticities of a triatomic (non-linear) gas is
cp
Ans. (d) : We know, =γ (a) 3 : 4 (b) 1 : 2
cv (c) 4 : 3 (d) 5 : 3
For an adiabatic process, AP EAMCET (21.09.2020) Shift-II
PV γ = C Ans. (c) : From elasticity of adiabatic process
P1V1γ = P2 V2 γ Eadi = γP ….(i)
γ From elasticity of isothermal process
P1  V2 
=  Eiso = P …..(ii)
P2  V1 
Ratio of the adiabatic to isothermal elasticity,
P 16 E adi γP
Given, 2 = = = γ = 1+
2
[ For triatomic, f = 6 ]
P1 81 E iso P f
γ
81  27  2 4
=  = 1+ =
16  8  6 3
4 3γ
3 3 376. Five moles of Hydrogen gas initially at STP is
  =  compressed adiabatically so that its
2 2
By comparing, temperature becomes 673 K. The increase in
4 = 3γ internal energy of the gas is
cp (R = 8.3 J mol–1 K–1, γ = 1.4 for diatomic gas)
4
= γ= (a) 80.5 kJ (b) 21.55 kJ
cv 3
(c) 41.50 kJ (d) 65.55 kJ
cp AP EAMCET (20.04.2019) Shift-II
373. The correct relation between γ = and
cv Ans. (c) : Final temperature = 673 K = T2
temperature T is : Initial temperature = 273 K = T1
° 1 Number of moles of hydrogen, n = 5
(a) γαT (b) γα
T Universal gas constant (R) = 8.3 J mol–1 K–1
1 For diatomic gas (γ) = 1.4
(c) γαT (d) γα
T Now, change in internal energy due to adiabatic process
JEE Main-31.01.2023, Shift-I is

655
∴ ∆U = ncv∆T From equation (i) and (ii)–
 ∵ Specific heat,  T γ
R     V = constant
∆U = 5 × ( T2 – T1 )  C = R  V
γ −1  γ – 1  T.Vγ–1 = constant
v

8.3 379. In adiabatic expansion of gas, the quantity
= 5× ( 673 – 273) which remains constant is
1.4 − 1
(a) amount of heat
8.3
= 5× × 400 (b) temperature
0.4 (c) both the amount of heat and temperature
∆U = 41.5kJ (d) pressure and temperature of gas
So, the increase in internal energy of the gas is CG PET- 2013
∆U = 41.5 kJ Ans. (a) : In adiabatic expansion of gas have no heat is
377. In an adiabatic process, the pressure is allowed to enter into or escape from the gas so, amount
2 2 of heat remains constant in an adiabatic expansion of
increased by %.If γ = , then the volume
3 3 gas.
decreases by nearly For a adiabatic condition–
4 2 PVγ = constant
(a) % (b) %
9 3 C
The ratio of specific heat (γ) = P of the gas.
9 CV
(c) 1% (d) %
4 380. When gas in a vessel expands its internal
BITSAT-2020 energy decrease. The process involved is
Ans. (c) : For the adiabatic process. (a) isothermal (b) isobaric
PVγ = Constant (c) adiabatic (d) isochoric
PVγ = C CG PET- 2008
Taking log on both sides. MP PET-2008
log P + γ log V = log C Ans. (c) : Gas in a vessel expands its internal energy
Differentiating, decrease. The process involved is adiabatic
∆P ∆V dU + dW = dQ
+γ =0
P V ∴ dU = – dW
∆V −1  ∆P  Hence, the process is adiabatic.
=  
V γ  P  381. Assertion: Adiabatic expansion is always
∆V − 1  ∆P  accompanied by fall in temperature.
× 100 =  × 100  Reason: In adiabatic process, volume is
V γ  P 
inversely proportional to temperature.
3 2 (a) If both Assertion and Reason are correct and
=– ×
2 3 Reason is the correct explanation of
= –1% Assertion.
So, volume decrease by 1%. (b) If both Assertion and Reason are correct, but
378. Which equation is valid for adiabatic process? Reason is not the correct explanation of
γ −1
(a) TV = constant γ −1
(b) pV = constant Assertion.
(c) If Assertion is correct but Reason is incorrect.
p γ −1
(c) T γ V γ −1 = constant (d) γ −1 = constant (d) If both the Assertion and Reason are
T incorrect.
CG PET- 2014 AIIMS-2011, 2013, 2014
Ans. (a) : For an adiabatic process– Ans. (c) : According to option the equation of state for
PVγ = Constant (i) an adiabatic process is–
For an ideal gas
PV
= nR = C TVγ–1 = constant
T constant
T=
P=
CT V γ –1
V So, if the gas is expanded then temperature falls. But the
T temperature for the process is inversely proportional to
P= ….(ii)
V Vγ–1.

656
382. Assertion: Air quickly leaking out of a balloon Ans. (d) : Given,
becomes cooler. T1 = 27°C = 300 K
Reason: The leaking air undergoes adiabatic V 1= V
expansion.
(a) If both Assertion and Reason are correct and 8
V2 = V
the Reason is a correct explanation of the 27
Assertion. 5
(b) If both Assertion and Reason are correct but γ=
3
Reason is not a correct explanation of the
For adiabatic process,
Assertion.
(c) If the Assertion is correct but Reason is TV γ –1 = constant
incorrect. So,
(d) If both the Assertion and Reason are incorrect. γ –1
T1  V2 
(e) If the Assertion is incorrect but the Reason is =  ….(i)
correct. T2  V1 
AIIMS-2005 Put the given value in equation (i)
Ans. (a) : The condition that on exchange of heat takes 5
−1
place between the system and surroundings then such   8  3
300   27  
V
process is called adiabatic process, leaking air of
= 
balloon undergoes adiabatic due to work done against T2  V 
external pressure the internal molecules into or out of
the system by heat the air. Molecule escaping from the 9
T2 = 300 ×
balloon work on the surrounding air molecule push 4
them out the way. T2 = 675 K
383. Assertion: When a bottle of cold carbonated drink Rise in temperature (∆T) = 675 – 300
is opened, a slight fog forms around the opening. Rise in temperature (∆T) = 375 K
Reason: Adiabatic expansion of the gas causes
lowering of temperature and condensation of 385. Ideal gas for which γ = 1.5 is suddenly
water vapours. 1
compressed to th of its initial volume. The
(a) If both Assertion and Reason are correct and 4
the Reason is a correct explanation of the ratio of the final pressure to the initial pressure
Assertion.  C 
(b) If both Assertion and Reason are correct but is  γ = P 
Reason is not a correct explanation of the  C V 

Assertion. (a) 4 : 1 (b) 8 : 1


(c) If the Assertion is correct but Reason is (c) 1 : 16 (d) 1 : 8
incorrect. MHT-CET 2020
(d) If both the Assertion and Reason are incorrect.
Ans. (b) : In an adiabatic process, the energy transfer
(e) If the Assertion is incorrect but the Reason is
from the thermodynamic system to surrounding is only
correct.
in the form of work i.e. there is no transfer of heat and
AIIMS-2003
matter in an adiabatic process. For adiabatic process,
Ans. (a) : When a bottle of cold carbonated drink is PV is constant.
opened, then adiabatic expansion of gas evolved due
temperature of gas decreases. Adiabatic expansion P1 V1γ = P2 V2γ
causes cooling. γ 1.5
P1  V2   V / 4  1 1
384. An ideal gas at 27ºC is compressed =  =  = 3/ 2
=
adiabatically to (8/27) of its original volume. If P2  V1   V  4 8
ratio of specific heats, γ = 5/3 then the rise in P2 8
=
temperature of the gas is 375 K P1 1
(a) 500 K (b) 125 K
386. A gas expands adiabatically at constant
(c) 250 K (d) 375 K
1
UPCPMT 1984, AIPMT 1999 pressure, such that its temperature T ∝ .
DPMT 2000, BHU 2001 V
Punjab PET 2001 The value of Cp/Cv of the gas is
UPSEE 2002,2003 (a) 1.30 (b) 1.50
Karnataka CET 2003 (c) 1.67 (d) 2.00
DCE 2004, MHT-CET 2020 MHT-CET 2004
657
Ans. (b) : 388. In an adiabatic expansion of an ideal gas the
TVγ-1 = Constant …. (i) product of pressure and volume :
K (a) decreases
= Constant (b) increases
R
(c) remains constant
1
T∝ (d) at first increases and then decreases
V
Karnataka CET-2020
1
T=K ( ∵ K = proportionality constant) Ans. (a) : Ideal gas equation is given by,
V PV = nRT
1
∵ T is decreasing, PV also decreases.
TV 2 = K
If T decreases PV also decreases because in an
1
TV = constant
2
…. (ii) adiabatic expansion. Internal energy decreases and
temperature also decreases.
By comparing equation (i) and (ii) we get
389. A graph of pressure versus volume for an ideal
1
γ –1 = gas for different processes is as shown. In the
2 graph curve OC represents :
1
γ = +1
2
3
γ=
2
=1.5
387. A sample of gas with γ = 1.5 is taken through
an adiabatic process in which the volume is
(a) isochoric process (b) isothermal process
compressed from 1600 cm3 to 400 cm3 If the
initial pressure is 150 kPa, then work done in (c) isobaric process (d) adiabatic process
the process is Karnataka CET-2009
(a) - 120 J (b) + 120 J Manipal UGET-2009
(c) - 480 J (d) + 480 J Ans. (d) :
SCRA-2013
Ans. (c) : Given that,
P1 =150 kPa, V1 = 1600 cm3, V2 = 400 cm3 and γ = 1.5
As the process is adiabatic,
So, by using adiabatic equation, i.e.
P1V1γ = P2 V2γ
γ
V 
P2 = P1  1 
 V2  Isochoric Process– It is a thermodynamic process in a
1.5 system, during which the volume remains constant is
 600  called the isochoric process.
= 150  
 400  ∆V = 0
1.5
= 150 × 4 Isobaric Process– The thermodynamic process in a
= 1200 kPa system during which the pressure remains Constant
Work done in adiabatic process ∆P = 0
P V – P2 V2 Adiabatic Process– The thermodynamic process in a
W= 1 1
γ –1 system during which no heat or mass transfer occurs
between thermodynamic system is called an adiabatic
150 × 1600 – 1200 × 400
W= process.
1.5 – 1
• Curve OA, P = constant, means Isobaric process.
W=
240 – 480 • Curve OD, V = constant, means isochoric process.
0.5 • Curve OB represents isothermal process.
W = – 480J • Curve OC represent adiabatic process.

658
390. An ideal monoatomic gas at 27°C is compressed 392. The temperature of the system decreases in the
8 process of
adiabatically to times of its present volume. (a) free expansion
27
(b) adiabatic expansion
The increase in temperature of the gas is :
(c) isothermal expansion
(a) 375°C (b) 402°C (d) isothermal compression
°
(c) 175 C (d) 475°C J&K CET- 2007, EAMCET-2007
Karnataka CET-2003 Ans. (b) : In an adiabatic expansion since no heat is
Ans. (a) : As we know that for adiabatic process supplied from outside, therefore the energy for the
PVγ = Constant expansion of the gas is taken from the gas itself, it
means the internal energy of an ideal gas undergoing in
TVγ-1 = Constant an adiabatic expansion decreases, and because the
5 internal energy of an ideal gas depends only on the
For monoatomic gas γ = temperature, Therefore its temperature must decreases.
3
That is why the temperature of the system decreases in
T1 = 27°C = 27+273 = 300K the process of adiabatic expansion.
Using TVγ–1 = Constant 393. A certain mass of gas at NTP is expanded to
T1 V1γ–1 = T2V2γ–1 three times its volume under adiabatic
γ –1 conditions. The resulting temperature of gas
T2  V1  will be
= 
T1  V2  (γ for gas is 1.40)
1.4
 27 
2/3
1
(b) 273 × ( 3)
0.4
T2 =   × T1 (a) 273 ×  
 8   3
0.4
2 1
3 (d) 273 × ( 3)
1.4
T2 =   × 300 (c) 273 ×  
2 3
J&K CET- 2003
T2 = 675 – 273 = 402°C
Ans. (c) : As we know that, according to adiabatic law
∆T= T2 – T1 TVγ–1 = Constant
= 402°C – 27°C T1V1γ–1 = T2 V2γ–1
∆T = 375°C Given that γ = 1.40
γ –1
Where ∆T is the increase in temperature of the gas. T2  V1 
= 
391. A thermodynamic process in which the system T1  V2 
is insulated from the surroundings and no heat or,
flows between the system and the surroundings γ –1
V 
is an T2 = T1  1 
(a) isothermal process (b) adiabatic process  V2 
(c) isochoric process (d) isobaric process V 
1.4 – 1

J&K CET- 2011 T2 = 273  1 


 3 
Ans. (b) : The thermodynamic process in which there is 0.4
1
no exchange of heat from the system to its surrounding ∴ T2 = 273  
neither during expansion nor during compression is 3
called adiabatic process. 394. Bursting of cycle tube, propagation of sound
In an adiabatic process ∆Q = 0 waves in air, sudden blowing off of safety valve
of a pressure cooker are examples of
• Isothermal process is a type of thermodynamic (a) isothermal process (b) adiabatic process
process in which the temperature T of a system (c) cyclic process (d) isobaric process
remains constant J&K CET- 1998
∆T = 0 Ans. (b) : Bursting of cycle tube, propagation of sound
• A thermodynamic process in which the volume of the wave in air, sudden blowing off of safety valve of a
system is kept constant is called isochoric Process. pressure cooker are examples of adiabatic process.
∆V = 0 395. The process in which no heat is exchanged with
the surroundings of a system is described as
• Isobaric process is a type of thermodynamic process
(a) isothermal (b) isobaric
in which the pressure of the system remains constant.
(c) isometric (d) adiabatic
∆P = 0 J&K CET- 1997
659
Ans. (d) : The process in which no heat is exchanged R
W= Ti − T f  ....(i)
with the surrounding of a system is described as γ −1 
adiabatic process. Where,
396. In an adiabatic change, the pressure P and R = universal gas constant
temperature T of a monoatomic gas are related γ = adiabatic constant
by the relation P ∝ Tc, where c is equal to Now, universal gas constant is given by
5 2 R = Cp − Cv
(a) (b)
3 5 Hence after putting the value of R in equation (i)
3 5 we get,
(c) (d)
5 2 C − Cv
W= p  Ti − T f 
UP CPMT-2010, SCRA-2012, AIIMS-2007 γ −1 
Ans. (d) : We know that,
Cp
For adiabatic change – ∵ γ= , so we have
γ Cv
T
= constant Cp − C v
P γ−1 W=  Ti − Tf 
T γ ∝ P γ−1 Cp
−1
γ Cv
T γ−1 ∝ P Cp − C v
W= Ti − T f 
(Cp − C v ) / C v 
γ
γ−1
P ∝T
W = C v ( Ti − T f )
γ 5/3 Now from this result we can say that the work done by
c= = gas is proportional to specific heat at constant volume
γ −1 5 / 3 −1
and difference between initial and final temperature.
5
c= 399. During adiabatic expansion, the increase in the
2 volume is associated with
397. In an adiabatic process (a) increase in pressure and decrease in T
(a) PγV = constant (b) TVγ–1 = constant (b) decrease in pressure and increase in T
(c) PV = constant (d) All of these (c) increase in pressure and T
UP CPMT-2003 (d) decrease in pressure and T
Ans. (b) : For an adiabatic process, PVγ = constant. TS EAMCET 30.07.2022, Shift-I
nRT Ans. (d) : PVγ = constant
Using the ideal gas equation and substituting V = And, TVγ–1 = constant
P
In the above equation gives TVγ–1 = constant From the above equation increasing in volume pressure
and temperature both are decreases.
Where γ is the specific heat ratio
398. Consider n moles of an ideal gas. The gas
expands adiabatically from initial temperature
Ti to final lower temperature Tf. The work
done by gas is proportional to
(a) Specific heat at constant pressure and
difference between initial and final
temperature
(b) Specific heat at constant volume and 400. An ideal gas undergoes an adiabatic process. If
difference between initial and final the pressure of the gas is reduced by 0.1% then
temperature  C 
the volume is changed by  Given γ = p = 5/3 
(c) Only specific heat  Cv 
(d) Only difference between initial and final (a) 0.1 % (b) 0.05%
temperature
(c) 0.06% (d) –0.05%
TS EAMCET 31.07.2022, Shift-II
TS EAMCET 05.08.2021, Shift-I
Ans. (b) : Given,
Ans. (c) : From the relation, PVγ = K
Initial temperature = Ti
Now differentiating we get, PVγ–1 dV + dP.Vγ = 0
final temperature = Tf
−dV dP
as we know work done in an adiabatic expansion is By making proper rearrangements we get, =
given by – V γP

660
In adiabatic process on a gas with γ = 1.4, the pressure  2 
1.4
of the is reduced by 0.1 %. = 2 × 105  
The volume (decreases) changes by about,  0.5 
P2 = (2×105) (4)1.4 [(4)1.4 = 6.96]
−dV 0.1 5 2
= × 100 = 0.06% = 2×10 ×6.96 N/m
V 1.4 P2 = 13.92×105 N/m2
401. An ideal monoatomic gas of volume V is Now work done in adiabatic process,
adiabatically expanded to a volume 3V at 27°C. P V – P2 V2
The final temperature in Kelvins is: W= 1 1
γ −1
CP
(use = 5/3) 2 ×105 × 2 − 13.92 × 105 × 0.5
CV =
(a) 144.2 (b) 170.3 1.4 − 1
5
(c) 50.4 (d) 100.2 4 × 10 − 6.96 × 105
=
TS EAMCET 05.08.2021, Shift-II 0.4
Ans. (a) : In an adiabatic process,
=
( 4 − 6.96 )
× 105
PVγ = constant 0.4
TVγ–1 = constant ( −2.96 ) ×105
γ−1 =
T2  V1  0.4
=  W= –7.4×105 J
T1  V2 
403. When there is no heat change from
For Monoatomic’gas
surroundings in a system, then the process
5 taking place is
γ=
3 (a) isobaric (b) isochoric
T1 = 27°C = 27 + 273 = 300 K (c) adiabatic (d) isothermal
5
−1 2/3 SRM JEE - 2011
 V 3  V  Ans. (c) : The process in which no heat is transferred to
T2 = 300   = 300  3V 
 3V    or from the system, during the whole process, is called
2/3 adiabatic process.
1
= 300   5
3 404. One liter of a gas (with γ = ) at NTP is
2 3
×3
1 3 compressed adiabatically to one cubic
T23 = (300)3  3  centimeter, then the resulting pressure is :
(a) 10 Atm (b) 103 Atm
3 1 (c) 105 Atm (d) 100 Atm
T2 = 27000000   MP PMT-2013
9
3 Ans. (c) :Given that,
T2 = 3000000
P1=1atm,
T2 = 144.2 K
V1= 1L =1000 cm3
402. A diatomic gas of volume 2 m3 at a pressure 2 × We know that,
105 N/m2 is compressed adiabatically to a For adiabatic process
volume 0.5 m3. The work done in this process
is. P1V1γ = P2 V γ2
[Use 41.4 = 6.96] 5
(a) 2.96 × 105 J (b) –2.96 × 105 J P2 = ?, V2 = 1 cm3, γ =
5
3
(c) –7.4 × 10 J (d) 7.4 × 105 J (1) × (1000)5/3 = P2 × (1)5/3
TS EAMCET 04.08.2021, Shift-I (1000) 5/3 = P2
Ans. (c) : For adiabatic condition, P2 = 105 atm
PVγ = constant 405. For a gas of non rigid diatomic molecules, the
Given, V1 = 2 m3 Cp
P1 = 2×105 N/m2 value of γ = is :
CV
V2 = 0.5 m3
By adiabatic process, 9 7
(a) (b)
P 1V 1γ = P 2V 2γ 7 5
γ 5 11
V  (c) (d)
P2 = P1  1  [for diatomic gas γ = 1.4] 3 9
 V2  MP PMT-2013
661
Ans. (b) : The value of γ for diatomic γ
P ∝ ( T ) γ −1 ----- (i)
2
γ = 1+ And according to question
f
Degree of freedom for diatomic (f) = 5 P ∝ T 3 .......(ii)
2 Comparing equation (i) from equation (ii) we, get-
Hence, γ = 1+ γ
f =3
2 γ −1
γ = 1+
5 γ = 3γ – 3
γ = 7/5 2γ=3
3
406. P-V plots for gases during adiabatic process as γ =
shown in figure plot 1 and 2 should correspond 2
respectively to C
∴ We know that p = γ
Cv
Cp 3
=
Cv 2
408. The work of 146 kJ is performed in order to
compress one kilo mole of a gas adiabatically
(a) He and O2 (b) O2 and He and in this process the temperature of the gas
(c) He and Ar (d) O2 and N2 increases by 7oC. The gas is (R = 8.3J mol−1
JIPMER-2017 K−1)
JIPMER-2015 (a) diatomic
Ans. (b) : The slope of adiabatic process of P–V (b) triatomic
(c) a mixture of monoatomic and diatomic
 dP  P
diagram  =γ (d) monoatomic
 dV  V
UP CPMT-2008
From graph (slope)2 > (slope)1 JIPMER-2009
⇒ γ 2 > γ1 UPSEE - 2007
γ for monoatomic gas (Ar, He) is greater than γ for UPSEE - 2015
diatomic gas (O2, N2). JCECE-2008
407. During an adiabatic process, the pressure of a Ans. (a) : W = – 146 kJ
gas is found to be proportional to the cube of n = 1000 moles.
Cp T 2 – T 1 = 7 0C
absolute temperature. The ratio for the We know that,
Cv
gas is nR ( T1 − T2 )
W=
4 γ −1
(a) (b) 2
3 1000 × 8.3 × 7
146000 =
(c)
5
(d)
3 γ −1
3 2 γ – 1 = .4
TS EAMCET 09.05.2019, Shift-I γ = 1.4
JIPMER-2014, CG PET- 2013 Hence, the gas is diatomic gas.
NEET 2013, BCECE-2008 409. A perfect gas is found to obey the relation
Karnataka CET 2007 PV3/2 = constant during an adiabatic process. If
UP CPMT-2006 such a gas initially at a temperature T, is
AIEEE 2003 compressed to half of its initial volume, then its
Karnataka CET-2001 final temperature will be:
AP EAMCET -2011, 2000 (a) 2T (b) 4T
1/2
Ans. (d) : For an adiabatic process relation between (c) (2) T (d) 2(2)1/2T
pressure and temperature is given as- JIPMER-2005
γ Ans. (c) : Given, Ti = T, Vi = V
 P1   T1  γ −1
=
    Tf = ?, Vf =
V
 P2   T2  2
662
For adiabatic process, 412. Two containers A and B contain equal volumes
TVγ – 1 = constant of an identical gas at the same pressure and
temperature. The gas in container A is
Ti Viγ−1 = Tf Vfγ−1 compressed to half its original volume
3
3
−1 isothermally, while the gas in container B is
−1  V 2 compressed to half its original volume
T(V) = Tf  
2
2 adiabatically. The ratio of the final pressure of
Tf = 21/2T gas in container B to that of gas in container A
410. For an adiabatic expansion of a monoatomic is ______
γ −1
perfect gas, the volume increases by 24%. 1
(a) (2)γ–1 (b)  
What is the percentage decrease in pressure ? 2
(a) 24% (b) 40% 2 2
 1   1 
(c) 48% (d) 71% (c)   (d)  
JIPMER-2015  1− γ   γ −1 
AP EAMCET-23.08.2021, Shift-I
Ans. (b) : For adiabatic process
γ Ans. (a) : Given,
PV = constant
Two containers A and B contain equal volumes of an
P α V–γ identical gas at the same pressure and temperature.
dP  dV 
= –γ  
P  V 
for mono atomic (γ) = 5/3
dP  dV 
× 100 = – γ   ×100
P  V 
dP  5 24 
× 100 = –  ×  ×100
P  3 100 
dP 40 The P–V diagram shown for the container A and
× 100 = – × 100
P 100 container B.
decreases in pressure = 40% When the compression is isothermal for gas in container
411. A sound wave passing through an ideal gas at A,
NTP produces a pressure change of 0.001 P 1 V1 = P 2 V2
dyne/cm2 during adiabatic compression. The P1 = P0
corresponding change in temperature (γ = 1.5 V1 = 2V0
for the gas and atmospheric pressure is 1.013 × V2 = V0
106 dyne/cm2) is Then,
(a) 8.97 × 10–4 K (b) 8.97 × 10–6 K P0.(2V0) = P2.V0
(c) 8.97 × 10–8 K (d) 8.97 × 10–9 K P ( 2V0 )
⇒ P2 = 0 = 2P0
AP EAMCET -2012 V0
Ans. (c) : For adiabatic process When the compression is adiabatic for gas in container
Tγ P1–γ = constant B,
γ
T =P γ–1 P1V1γ = P2 V2 γ
⇒ P0 ( 2V0 ) = P2 ( V0 )
γ γ
γ log T = (γ – 1) log P
Differentiating both side –
P0 ( 2V0 )
γ

∆T ∆P ⇒ P2 =
γ = ( γ – 1)
( V0 )
γ
T P
P0 ( V0 ) ( 2 )
γ γ
 γ − 1  ∆P
∆T =   T ⇒ P2 =
 γ  P ( V0 )
γ

1.5 −1 0.001 ⇒ P2 = ( 2 ) P0


γ
=  × × 20
 1.5  1.013×106
The Ratio of the final pressure of gas in container B to
= 8.97 × 10–8 K that of gas in container A.

663
( P2 )B 2 γ P0 mCp dT −VdP
= =
( P2 )A 2P0 mCv dT PdV

=  CP 
2 −VdP
γ–1 γ=  = γ
(P2)B : (P2)A = (2) PdV  CV 
413. A diatomic gas initially at 18oC is compressed dV dP
adiabatically to one eighth of its original γ + =0
volume. The temperature after compression V P
will be γ ln V + ln P = ln C
o
(a) 18 C o
(b) 395.4 C PVγ = C
o o
(c) 144 C (d) 887.4 C 415. If a monatomic gas is compressed adiabatically
Punjab PET 1995 to (1/27)th of its initial volume, then its pressure
AIPMT 1996,1999 becomes
DPMT 2000 (a) 27 times (b) 125 times
CG PET-2021 (c) 243 times (d) 81 times
Ans. (b) : Given, T1 = 18 oC (e) 64 times
1 V1 Kerala CEE 2020
V2 = V1 , =8
8 V2 Ans. (c) : For adiabatic process,
We know that for adiabatic compression, P1V1γ = P2 V2γ
γ−1
TV = constant. 1
Given, V2 = V1
T1V1γ −1 = T2 V2γ −1 27
γ −1 γ
V  V
T2 = T1  1  PV γ = P2  
 V2   27 

= (18 + 273) (8)(1.4−1) P2 V γ


PV γ =
27 γ
= 291× (8)(0.4)
= 668.5 K P2 = 27 γ P
= 395.5 o C For monoatomic energy,
414. Which of the following is adiabatic gas 5
γ=
equation? 3
(a) PV = Const (b) PVγ = Const P2 = 33γ P = 35 P = 243P
γ–1
(c) PV = Const (d) Pγ = 1/V
416. Which of the two, (i) compressing a gas
SRM JEE-2018
isothermally until its volume is reduced by half
SRM JEE-2017
(ii) compressing the same gas adiabatically
Ans. (b) : PVγ = Constant until its volume is reduced by half, will require
From 1st law of thermodynamics, more work to be done?
δQ = dU + δW {δW = PdV} (a) (i)
δ Q = dU + PdV …… (1) (b) (ii)
H = U + PV (c) Both will require the same amount of work
dH = dU + d(PV) (d) It will depend upon the nature of the gas
dH = dU + PdV + VdP
J&K-CET-2012
dH = δ Q + VdP …… (2)
For adiabatic, Ans. (b) : Case I:
δ Q = dU + PdV Compressing a gas isothermally will its volume is
dU = –PdV ….. (3) 1
reduced to of initial volume
From equation (2), 2
dH = δQ + VdP Case II:
dH = VdP ….. (4) Compressing same gas adiabatically (PVγ = C) until its
From equation (4) & (3) volume reduced by half.
dH −VdP Hence, graph plotted for case (i) and case (ii) So, PV
=
dU PdV diagram is –

664
 R  γ−1
= n  ( 2 − 1) To
 γ −1 
∆Q =
γ −1
( 2 − 1)
nRTo γ −1

Therefore compression of gas compressed adiabatically ∵ PV = nRT


will require more work than when compressed Po Vo γ−1
isothermally. ∵ ∆Q = (2 − 1)
( γ − 1)
417. The initial pressure and volume of a given mass
418. An ideal gas having initial pressure P, volume
 Cp  V and temperature T is allowed to expand
of an ideal gas  with = γ  , taken in a
 Cv  adiabatically until its volume becomes 4V,
cylinder fitted with a piston, are P0 and V0 T
while its temperature falls to . If the work
respectively. At this stage the gas has the same 2
temperature as that of the surrounding done by the gas the during the expansion is
medium which is T0. It is adiabatically αPV, the value of α is
V (a) 1.25 (b) 1.0
compressed to a volume equal to 0 . (c) 1.50 (d) 2.0
2
Subsequently the gas is allowed to come to TS EAMCET 19.07.2022, Shift-I
thermal equilibrium with the surroundings. Ans. (b) : Given,
What is the heat released to the surrounding? Initial pressure = P
PV Volume = V
(a) 0 (b) (2γ−1 − 1) 0 0 Temperature = T
γ −1
After adiabatically expansion,
P0 V0
(c) γP0V0ln2 (d) T
2( γ − 1) Temperature =
2
WB JEE 2019 Volume = 4V
Ans. (b) : Given, TVγ–1 = Constant
CP
=γ T1V1γ – 1 = T2V2γ –1
CV T
T(V)γ– 1 =   (4V) γ−1
Initial pressure and volume are P0 and Vo 2
Gas has same temperature of surrounding 2 = (4)γ – 1
V 21 = (2)2(γ–1)
Adiabatically compressed to a volume = 0
2 2(γ – 1) = 1
For adiabatic process 1
γ−1 γ −1 =
γ−1 2
T2  P2  γ  V1 
=  =  1 3
T1  P1   V2  γ = +1 =
2 2
γ−1
T2  V1  γ = 1.5
∴ = 
T1  V2  ∵ PVγ = constant
γ−1 ∴ TVγ –1 = Constant
 V 
T2 = T0  0  [T1 = T0] PV γ
 V0 / 2  = Constant
TV γ−1
T2 = T0(2) γ –1 PV
∆Q = nCVdT = Constant
T
 R   T1   T2 
= n  (T2 − T1 )
 γ −1   VP  = V P 
 1 1  2 2
 R  T T/2
∆Q = n   ( T2 − To ) =
 γ −1  VP (4V).P2
 R  γ−1 1
= n  [To (2 ) − To ] P2 = × P
 γ −1  8

665
1 421. Given below are two statements
∵ Workdone = (P1V1 − P2 V2 )
γ −1 Statement-I: When µ amount of an ideal gas
= α PV (given) undergoes adiabatic change from state (P1, V1,
T1) to state (P2, V2, T2), then work done is
1  P 
αPV =  PV − 4V  IR ( T2 − T1 ) Cp
1.5 − 1  8  W= , where γ = and R =
1− γ Cv
1 1
= × PV   universal gas constant.
0.5 2
Statement-II : In the above case, when work is
PV
αPV = × 0.5 done on the gas, the temperature of the gas
0.5 would rise.
α=1 Choose the correct answer from the options
419. When a gas expands adiabatically, its volume is given below
doubled while its absolute temperature is (a) Both statement-I and statement-II are true
decreased by a factor of 2. The value of the
(b) Both statement-I and statement-II are false
adiabatic constant is
(a) 1 (b) 5/3 (c) statement-I is true but statement-II is false
(c) 2 (d) 7/5 (d) statement-I is false but statement-II is true
TS EAMCET 20.07.2022, Shift-I JEE Main-28.06.2022, Shift-I
Ans. (c) : Gas expand adiabatically, Ans. (a) : Given,
Volume V1 = V Statement (i): Work done for adiabatic
V2 = 2V nR(T2 − T1 )
Absolute temperature is decrease by factor = 2 WD =
1− γ
T1
i.e, =2 C
T2 γ= P
C
TVγ – 1 = constant
V

γ−1
Work done on the gas = – ve work
T1  V2  Q = W + ∆U
= 
T2  V1  0 = W + ∆U
γ−1 ∆U =–W
 2V 
2=  ∴ ∆U = Positive
 V 
Hence, temperature will increase.
2 = (2)γ –1
γ–1=1 422. Two moles of a monoatomic gas at 27 ºC and
γ=2 three moles of a diatomic gas at the same
temperature expand adiabatically. If the work
Hence the value of adiabatic constant γ = 2
done by each gas during the expansion is
420. An ideal gas undergoes four different processes 4157J, The ratio of the final temperatures of
from the same initial state as shown in the
figure below. Those processes are adiabatic, the monoatomic gas to that of the diatomic gas
isothermal, isobaric and isochoric. The curve is
which represents the adiabatic process among (Universal gas constant = 8.314 Jmol–1 K–1)
1, 2, 3 and 4 is (a) 3 : 5 (b) 4 : 7
(c) 2 : 3 (d) 3 : 7
AP EAMCET-24.04.2019, Shift-II
nR(T2 − T1 )
Ans. (b) : Work, done WD =
1− γ
C
γ= P
CV
For monoatomic, n = 2, γ = 1.6
(a) 3 (b) 4 For diatomic , n = 3, γ = 1.4
(c) 1 (d) 2
W = 4157 J
Ans. (d) : For monoatomic
Curve (4) ⇒ Isobaric process. (P = C)
2 × 8.14(273 − T2 )
Curve (3) ⇒ isothermal process (T = C) 4157 =
Curve (2) ⇒ Adiabatic process (PVγ = C) 1.6 − 1
Curve (1) ⇒ Isochoric Process (V = C) T2 = 135 K

666
For diatomic As we know,
3 × 8.314(273 − T21 ) PV = nRT
4157 = T=C
1.4 − 1
PV = Constant
T'2 = 233.33K
1
Ratio of monoatomic final temperature to diatomic final P∝
T V
temperature = 2 Volume decreases, when compressed isothermally then
T2' pressure will increase.
135 Hence, it is said that when ideal gas is compressed
= isothermally the pressure will increase.
233.33
= 0.578 426. A given system undergoes a change in which
T 4 the work done by the system equals the
= 2 ≃ decrease in its internal energy. The system
T2' 7 must have undergone an _______
423. A gas is expanded from an initial state to a final (a) Isothermal change (b) Adiabatic change
along a path on a P-V diagram. The path (c) Isobaric change (d) Isochoric change
consists of (i) an isothermal expansion of work AP EAMCET-25.08.2021, Shift-I
50 J. If the internal energy of gas is changed by J&K CET- 1999
–30 J, then the work done by gas during
Ans. (b) : We know that,
adiabatic expansion is
(a) 40 J (b) 100 J ∆W + ∆U = 0
(c) 30 J (d) 20 J ∆W = – ∆U
AP EAMCET-04.07.2022, Shift-II According to first law of thermodynamics
Ans. (c) : In case of isothermal condition, ∆Q = ∆U + ∆W
Work done = 50 J ∆Q = 0 (∵ adiabatic process)
If internal energy of gas, ∆U = – 30 J ∆U = –∆W
Work done in case, of adiabatic ∆W = –∆U
Process, Hence, process is adiabatic.
dQ = dU + dW 427. The mole of an ideal gas expands adiabatically
dU = – dW from 200 K to 250 K. If the specific heat of the
dW = – dU gas at constant volume is 0.8 kJ kg–1, then the
dW = + 30 J work done by the gas is
424. during adiabatic expansion of 2 moles of a gas, (a) 20 J (b) 20 kJ
the change in internal energy was found to be (c) 40 J (d) 40 kJ
equal to 100 J. The work done during the AP EAMCET-24.04.2017, Shift-II
process will be equal to____ Ans. (d) :Given,
(a) Zero (b) 200 J Temperature change = 250 K – 200K = 50 K
(c) –100 J (d) 100 J Specific heat = 0.8 kJ/kg
AP EAMCET-25.09.2020, Shift-II dU = ms∆T
Ans. (d) : according to first law dU = 1 × 0.8 × 50
∆Q = W + ∆U dU = 40 kJ
during adiabatic expansion {∆Q = 0} As we know that,
W + ∆U = 0 dQ = dU + dW
W = –∆U dW = –dU [dQ = 0 for adiabatic process]
W = – ( –100) dW = – 40
W = 100J Work done by the gas = 40 kJ.
425. When the ideal gas is compressed isothermally, 428. A mixture of two non-reactive ideal gases is
then its pressure_____ enclosed in a vessel consisting of one mole of a
(a) Increases monoatomic gas ‘A’ and ‘n’ moles of diatomic
(b) Decreases gas ‘B’ at a temperature ‘T’. If the adiabatic
(c) remains the same 13
constant of the gaseous mixture is , then the
(d) first increases and then decreases 9
AP EAMCET-24.08.2021, Shift-II value of ‘n’ is.
Ans. (a) : When the ideal gas is compressed (a) 5 (b) 2
isothermally (T = C) (c) 4 (d) 3
Then the volume of gas gets decreases AP EAMCET-25.04.2017, Shift-I
667
Ans. (d) : TVγ–1 = c
monoatomic diatomic T1 (V1)γ–1 = T2 (V2)γ–1
n1 = 1 n2 = n T1 (5.6)2/3 = T2 (0.7)2/3
3 5 5
C v1 = R C v2 = R γ= for monoatomic
2 2 3
T2 = ( 8 )
2/3
n1CV1 + n 2 CV2 T1
(Cv)mix =
n1 + n 2 T2 = (23)2/3T1
3 5 T2 = 4 T1
R+n R nR∆T
(Cv)mix = 2 2 Work done, W =
1+ n ( γ – 1)
R ( 3 + 5n ) 1
= R.( T – 4T )
2 (1 + n ) = 4
5
(CP)mix = (Cv)mix + R –1
3
R  3 + 5n 
=  +R =
–3RT / 4
2  1+ n  2/3
9RT
= –
R  5 + 7n  8
(CP)mix =  
2  1+ n  430. Starting with the same initial conditions, an
ideal gas expands from volume V1 to V2 in
( C P )mix
(γmix) = three different ways. The work done by the gas
( Cv )mix is W1 if the process is purely isothermal, W2 if
R  5 + 7n  purely isobaric and W3 if purely adiabatic. The
  (a) W1 > W2 > W3 (b) W2 > W1 > W3
2 1+ n 
= 
R  3 + 5n  (c) W2 > W3 > W1 (d) W1 > W3 > W2
 
2  1+ n  AMU-2008
13 5 + 7n MHT-CET 2005
= Ans. (b) : Work done area under the P–V diagram
9 3 + 5n
39 + 65n = 45 + 63n Work done for isobaric = P (V2 – V1)
65n – 63n = 6 V
Work done for isothermal = P1V1 log 2
2n = 6 V1
n=3 P2 V2 – P1V1
Work done for adiabatic =
429. 5.6 litre of helium gas at STP is adiabatically γ –1
compressed to 0.7 litre. If the initial
temperature of the gas is T K, work done in the
process is (R is universal gas constant in SI
units)
9 9 
(a) RT (b) −  RT 
8  8  Since, area under the curve is maximum for isobaric
4  3 process in expansion.
(c) −  RT  (d) RT
 3  4 431. An ideal gas is compressed to half its initial
AP EAMCET-25.04.2017, Shift-II volume by means of several process. Which of
the process results in the maximum work done
Ans. (b) :Given that,
on the gas?
Initial volume = 5.6l (a) Adiabatic (b) Isobaric
Final volume = 0.7 l (c) Isochoric (d) Isothermal
Initial temperature = T K AIPMT-2015
668
Ans. (a) : (a) adiabatic (b) isochoric
(c) isobaric (d) isothermal
NEET National-2019
Ans. (a) : Free expansion i.e. expansion against vacuum
is adiabatic in nature for all type of gases. It should be
noted that temperature final temp is equal to initial temp
for ideal gases.
Free expansion dW = 0
dQ = dU + dW
434. In which of the following processes, heat is
neither absorbed nor released by a system?
Work done on the gas = area under P – V curve.
(a) Adiabatic (b) Isobaric
As area under the P –V curve is maximum for adiabatic
(c) Isochoric (d) Isothermal
process, so work done on the gas is maximum for
adiabatic process. NEET National-2019
(Slope)adia > (slope)isothermal Ans. (a) : Adiabatic process:
(Area)adia > (Area)iso > (Area)isobaric In adiabatic process exchange of heat ∆Q = 0
An adiabatic process is a reversible process with
432. A gas is compressed isothermally to half its
constant entropy for an ideal gas.
initial volume. The same gas is compressed
separately through an adiabatic process until Isothermal process:
its volume is again reduced to half. Then It is a process in which temperature remains constant.
(a) compressing the gas through adiabatic Isobaric process – pressure is constant
process will require more work to be done. Isochoric process – Volume is constant
(b) compressing the gas isothermally or 435. If ∆U and ∆W represent the increase in
adiabatically will require the same amount of internal energy and work done by the system
work. respectively in a thermodynamical process,
(c) which of the case (whether compression which of the following is true?
through isothermal or through adiabatic (a) ∆U = –∆W, in an adiabatic process
process) requires more work will depend (b) ∆U = ∆W, in an isothermal process
upon the atomicity of the gas. (c) ∆U = ∆W, in an adiabatic process
(d) compressing the gas isothermally will require (d) ∆U = – ∆W, in an isothermal process
more work to be done. AIPMT-2010
NEET-2016 Ans. (a) : From the first law of thermodynamics
Ans. (a) : ∆Q = ∆U+ ∆W
∆Q = 0 for adiabatic process
So, 0 = ∆U + ∆W
∆U = – ∆W
436. For an adiabatic process, the relation between
V and T is given by
(a) TVγ = constant (b) TγV = constant
1–γ
(c) TV = constant (d) TVγ–1 = constant
AP EAMCET(Medical)-2007
Ans. (d) : For an adiabatic process
Let us consider that the gas undergoes compression PVγ = Constants …..(i)
V We know that,
from V to . PV = RT ( For one mole of gas)
2
From P – V diagram, when we compare the area under RT
P=
the isothermal process curve and the adiabatic process V
curve, the work done for adiabatic process is higher Substituting the value of P in equation (i),
than that of work done by isothermal process. PVγ = C
433. Two cylinders A and B of equal capacity are RT γ
connected to each other via a stop cock. A .V = C
V
contains an ideal gas at standard temperature C
and pressure. B is completely evacuated. The TV γ−1 =
entire system is thermally insulated. The stop R
cock is suddenly opened. The process is TV γ –1 = Constant.

669
437. One mole of an ideal gas with γ = 1.4 is 439. In the adiabatic compression, the decrease in
adiabatically compressed so that, its volume is associated with
temperature rises from 27°C to 35°C. The (a) increase in temperature and decrease in
change in the internal energy of gas is: pressure
(R = 8.3 J/mol - K) (b) decrease in temperature and increase in
(a) –166 J (b) 166 J pressure
(c) decrease in temperature and decrease in
(c) –168 J (d) 168 J
pressure
AP EAMCET(Medical)-2001
(d) increase in temperature and increase in
Ans. (b) : Given that, pressure
γ = 1.4 EAMCET-2008
∆ T = (35° – 27°) = 8 Ans. (d) : According to first law of thermodynamics
R = 8.3 J/mol-K dQ = dU + dW
We know that, dQ = 0 from an adiabatic process
The change in internal energy, dU = – dW
γ –1
n.R.∆t γ –1
∆U = T1  P1  γ  V2 
γ –1 =  = 
T2  P2   V1 
1× 8.3 ( 8 ) This relation suggest that an decrease in volume is
∆U =
1.4 – 1 associated with increase in temperature and pressure.
∆U = 166 J 440. Which of the following is true in the case of
438. An ideal gas at a pressure of 1 atm and adiabatic process, where γ = CP/CV?
temperature of 27ºC is compressed (a) P1-γTγ = constant (b) PγT1–γ = constant
γ
adiabatically until its pressure becomes 8 times (c) PT = constant (d) PγT = constant
the initial pressure, then final temperature is (γ EAMCET-2008
= 3/2) Ans. (a) : Gas equation for an adiabatic process
(a) 627ºC (b) 527ºC PVγ = constant …..(i)
(c) 427ºC (d) 327ºC Standard gas equation
EAMCET-2000 PV = RT (For one mole of gas)
Ans. (d) : Given that, P1 = 1 atm, T2= 27°C = 27 + 273 RT
V= ….(ii)
= 300K P
3 From equation (i) and (ii),
P2 = 8 P1 = 8 atm, γ = = 1.5 γ
2 T
P   = constant
In an adiabatic process, P
γ γ−1
 T2   P2   Tγ 
  =  P. γ  = constant
 T1   P1  P 
 γ−1  P1–γ T γ = constant
 
T2  P2  γ  P1–γ T γ = constant is true for adiabatic process.
= 
T1  P1  441. A process in which there is no flow of heat
between the system and surroundings is a/an
 γ−1 
  (a) adiabatic process (b) cyclic process
 P  γ 
T2 = T1 ×  2  (c) isobaric process (d) isochoric process
 P1  (e) isothermal process
 1.5−1 
  Kerala CEE 2021
 8  1.5 
= 300 ×   Ans. (a) : Adiabatic Process– A thermodynamic
1
process where no flow of heat between the system and
0.5 /1.5
8 surrounding.
= 300 ×  
1 Thermodynamics equations of heat
T2 = 600K dQ = dU + dW
dQ = 0 (For adiabatic process)
T2 = ( 600 – 273)
dU = – dW
= 327°C The system is thermally isolated from surroundings.

670
442. A given mass of gas at a pressure 'P' and ∆T
absolute temperature 'T' obeys the law P ∝ T3 T × 100
during an adiabatic process. The adiabatic 1
bulk modulus of the gas at a pressure 'P' is = × 100 = 0.4%
250
2P P ∆P P
(a) (b) P ∴ = constant ⇒ =
3 T ∆T T
3P ∆P ∆T ∆P
(c) (d) 2 P ×100 = × 100 ⇒ × 100 = 0.4%
2 P T P
( ) ( ) ( )
AP EAMCET-23.04.2019, Shift-II α = 32 γ –1 = 32 1.4 –1 = 32 0.4
Ans. (c) : Given that, 2
α = ( 32 ) 5 = ( 2 ) 5 = 22 = 4
2 5×
P ∝ T3
PV = nRT α=4
PV
T=
nR (H) Polytropic and Other Process
3
 PV 
P∝   444. During an experiment, an ideal gas is found to
 nR  obey an additional law VP2 = constant. The gas
P2 V3 = constant .....(i) is initially at temperature T and volume. V.
Taking square root on both side of equation (i), The temperature of the gas will be following,
PV3/2 = c1/2 when it expands to a volume 2V?
PV3/2 = c .....(ii) (a) 2T (b) 4T
Differentiating both side of equation (ii),
(c) 6T (d) 5T
dV 3/ 2 dP CG PET- 2014
P + V3 / 2 =0
dV dV Ans. (a) : Given,
3 1/ 2 dP VP2 = constant …..(i)
PV = –V3 / 2
2 dV We know PV = nRT
3 –dP nRT
P= Again P=
2 dV / V V
−dP Put in equation (i), We get,
Bulk modulus, K =
(dV / V) (nRT) 2
∴ = constant
3 V
∴ Bulk modulus, K = P.
2 T2
= constant
443. A diatomic ideal gas is compressed V
1
adiabatically to of its initial volume. If the Thus volume V when expanded to 2V, temperature T2
32 T22 V2 2V
initial temperature of the gas is T1 (in Kelvin) = =
T12 V1 V
and the final temperature is α Ti, the value of α
is T2 = 2 T1
(a) 1 (b) 2 445. Two containers of equal volume contain the
(c) 3 (d) 4 same gas at pressure P1 and P2 and absolute
CG PET 2019 temperature T1 and T2 respectively. On joining
Ans. (d) : For adiabatic process, the vessels the gas reaches a common pressure
P and common temperature T. The ratio P/T is
Given,
equal to
V PT + P T
V1 = V, V2 = P P
(a) 1 + 2 (b) 1 1 2 22
32 T T (T + T )
1 2 1 2
T1 = Ti, T2 = αTi
P1T2 + P2 T1 P1 P
γ = 1.4 (for diatomic gas) (c) (d) + 2
(T1 + T2 ) 2 2T1 2T2
T1V1γ –1 = T2 V2γ –1 CG PET- 2006

671
Ans. (d) : From ideal gas– 9
= P0 V0
PV = mRT 2
PV Q=U+W
∴ m= [ according to first law of thermodynamic ]
RT
∴ Number of moles of gas in first container– 9 3
⇒ Q = P0 V0 + P0 V0
PV 2 2
m1 = 1 Q = 6 P 0V 0
RT1
447. Expansion during heating:
Number of moles of gas in second container– (a) Increases the weight of a material
PV (b) Decreases density of material
m2 = 2
RT2 (c) Occurs only in solid
(d) Occurs at same rate of all liquids and solids
Number of moles in containers when joined with each
other. AP EAMCET-19.08.2021, Shift-II
Ans. (b): Density is mass divided by volume As you
PV
m= heat something up the volume increases so the density
RT decreases.
But, 448. If the coefficient of superficial expansion is x
m = m1 + m2 times the coefficient of cubical expansion, then
P ( 2V ) P1V P2 V the value of x is
= + 3
RT RT1 RT2 (a) 2 (b)
2
2P P1 P2
= + 2 1
T T1 T2 (c) (d)
3 2
P P1 P SCRA-2015
= + 2
T 2T1 2T2 Ans. (b) : Given that,
446. 2 moles of an ideal mono-atomic gas is carried Coefficient of superficial expansion = γ
from a state (P0, V0) to state (2P0, 2V0) along a As we know that coefficient of cubical expansion is
straight line path in a P-V diagram. The related to coefficient of superficial expansion.
amount of heat absorbed by the gas in the The relation between cubical expansion and superficial
process is given by expansion β is given as
β γ
9 =
(a) 3 P0V0 (b) P0V0 2 3
2

3 or γ= .......(i)
(c) 6 P0V0 (d) P0V0 2
2
It is given that in question coefficient of the cubical
WB JEE 2017 expansion is increased by the x times of the coefficient
Ans. (c) : of superficial expansion i.e.
γ = βx
From equation (i) and (ii),
3
x=
2
x = 1.5
449. ‘m’ grams of a gas of molecular weight M is
flowing in an isolated tube velocity v. If the gas
Area under the curve given work done in P-v graph flow is suddenly stopped the rise in the
VP temperature is : (γ = ratio of specific heats, R=
W = V0 P0 + 0 0 universal gas constant, J= mechanical
2
equivalent of heat).
3
W = P0 V0 Mv 2 ( γ − 1) m v 2 ( γ − 1)
2 (a) (b)
2RJ M 2RJ
∆u = nC v (TB − TA )
mv 2 γ Mv 2 γ
3nR  2P0 × 2V0 P0 V0  (c) (d)
=  –  2RJ 2RJ
2  nR nR  AP EAMCET(Medical)-2006
672
Ans. (a) : Given that,
Mass of gas = m (I) Free Expansion Process
Molecular weight = M
Velocity = v 451. There is no change in internal energy of an
ideal gas when it undergoes.
Mechanical equivalent of heat = J (a) Isothermal expansion
Since the gas flow is suddenly stopped, we will consider (b) Adiabatic expansion
it to be an adiabatic process. (c) Free expansion
1 1 (d) Isobaric expansion
Change in energy ∆ K.E. = mv – M.0 2

2 2 AP EAMCET (18.09.2020) Shift-I


1 2 Ans. (c) Free expansion :- The expansion of a gas
= mv against vacuum (zero resistance) is known as free
2
Work done for adiabatic process, expansion.
nR∆T PV – P V
W= W= 1 1 2 2
γ –1 γ –1
∆U = ∆W [for adiabatic process]
1 nR∆T
mv 2 = J
2 γ –1
m W = 0 , dQ = 0
n=J [ no.of moles.] dQ = dU + W
M
0 = dU + 0
1 mR∆T
mv 2 = J Uf = Ui = 0
2 M ( γ – 1) Uf = Ui
v 2
JR∆T ∵ For free expansion there is no change in internal
= energy of an ideal gas when it undergoes.
2 M ( γ – 1)
452. Assertion: In free expansion of an ideal gas, the
Mv 2 ( γ – 1) entropy increases.
∆T = Reason: Entropy increases in all natural
2RJ
process
450. Two thermally insulated identical vessels A and (a) If both Assertion and Reason are correct and
B are connected through a stopcock. A contains the Reason is a correct explanation of the
a gas at STP and B is completely evacuated. If Assertion.
the stopcock is suddenly opened then (b) If both Assertion and Reason are correct but
(a) Temperature is halved Reason is not a correct explanation of the
(b) Internal energy of the gas is halved Assertion.
(c) Internal energy of the gas and pressure are (c) If the Assertion is correct but Reason is
halved incorrect.
(d) Temperature and internal energy of the gas (d) If both the Assertion and Reason are
incorrect.
remain the same
(e) If the Assertion is incorrect but the Reason is
(e) Pressure and internal energy of the gas remain correct.
the same AIIMS-2007
Kerala CEE 2021 Ans. (a) :
Ans. (d) : The expansion of the gas is not due to any
external work done on the system and hence, the
temperature and internal energy of the gas remain same.
• There is no work done by or on the system, also there
is no heat interaction, therefore internal energy of the
system remains same internal energy,
Expansion of a gas against vacuum is called free or
∆U = mcV∆t unrestricted expansion process.
∆W = 0 (Temperature is same) Pressure →decrease, Volume → increase
• A and B are of equal capacity, so when the stopcock dQ & dW = 0 , U = Constant
is suddenly opened the pressure in both cylinder enthalpy constant & but entropy increases. So, option
decrease and volume of both cylinder increase. (a) is correct.

673
455. A Carnot engine operating between two
(J) Heat Engine, Carnot’s Cycle reservoirs has efficiency 1/3. When the
and Refrigeration (COP) temperature of cold reservoir raised by x, its
efficiency decreases to 1/6. The value of x, if the
453. Other is labelled as Reason R temperature of hot reservoir is 99°C, will be
Assertion A: Efficiency of reversible heat (a) 62 K (b) 16.5 K
engine will be highest at -273º C temperature of (c) 33 K (d) 66 K
cold reservoir, JEE Main-01.02.2023, Shift-II
Reason R: The efficiency of Carnot's engine Ans. (a) :
depends not only on temperature of cold
reservoir but if depends on the temperature of
 T 
hot reservoir too and is given as η =  1 – 2 
 T1 
(a) A is true but R is false
(b) Both A and R are true R is correct
explanation Let T2 is the temperature of sink and T1 is the
(c) Both A and R are true but R NOT the correct temperature of source,
explanation of A Given, T1 = 99°C + 273 = 372 K
(d) A is false but R is true 1 1
JEE Main-30.01.2023, Shift-II η1 = , η2 =
3 6
Tsink T2 T
Ans. (b) : η = 1 − η1 = 1 – =1– 2
Tsource T1 372
Given than Tsink = –273ºC = 0K 1 T
=1– 2
0 3 372
η = 1− ; η =1 T2 1 2
T =1– =
So, Efficiency is maximum at, Tsink = –273ºC 372 3 3
454. A Carnot engine with efficiency 50% takes heat T 2 = 248K (sink temperature)
from a source at 600K, In order to increase the Now the sink temperature (T2 = 248K) is increased by x
efficiency to 70%, keeping the temperature of 1
& efficiency become ,
sink same, the new temperature of the source 6
will be. T2 + x
(a) 900K (b) 300K η2 = 1 –
T1
(c) 1000K (d) 360K
1 248 + x
JEE Main-25.01.2023, Shift-I =1–
6 372
T
Ans. (c) : η = 1 − sink 248 + x 1 5
Tsource =1– =
372 6 6
50 T 5
= 1 − sink 248 + x = × 372
100 600 6
Tsink 1 248 + x = 5 × 62
=
600 2 248 + x = 310
⇒ Tsink = 300 x = 62K
To increase in efficiency by 70%, 456. A Carnot engine whose low temperature
70 300 reservoir is at 350 K has an efficiency of 50% It
= 1− is desired to increase this to 60%. If the
100 ′
Tsource temperature of the low temperature reservoir
300 70 remains constant, then the temperature of high
= 1− temperature reservoir must be increased by

Tsource 100
how many degrees?
300 3 (a) 15 (b) 175
=

Tsource 10 (c) 100 (d) 50
′ = 1000K
Tsource (e) 120
Kerala CEE -2018
674
Ans. (b) : Efficiency of Carnot engine– T1 – ( T2 – 50 )
T 0.4 =
η = 1 – cold T1
Thot T – T2 + 50 T1 – T2 50
0.4 = 1 = +
When, η = 50% T1 T1 T1
50 T
= 1 – cold 0.4 = 0.2 +
50
100 Thot T1
350
0.5 = 1 – ⇒ Thot = 700K 0.2 =
50
Thot T1
When, η = 60% T1 = 250K
60 T Put the value of T1 in equation (i), we get–
= 1 – cold
'
100 Thot T – T2
0.2 = 1
60 350 T1
=1– '
100 Thot T2 = T1 – 0.2 T1
' T2 = 250 – 0.2 × 250 = 250 – 50
Thot = 875 K
T2 = 200K
Hence the temperature of high reservoir is increased
459. If the temperatures of source and sink of a
by–
Carnot engine having efficiency η are each
'
Thot – Thot = 875K – 700K decreased by 100 K, then the efficiency
= 175 K or 175ºC (a) remains constant (b) becomes 1
457. The Carnot cycle of a reversible heat engine (c) decreases (d) increases
consists of (e) becomes zero
(a) one isothermal and two adiabatic processes Kerala CEE- 2013
(b) two isothermal and one adiabatic processes Ans. (d) : In starting the efficiency–
(c) two isothermal and two adiabatic processes T T – T2
(d) two isobaric and two isothermal processes η = 1– 2 = 1
T1 T1
(e) two isochoric and two adiabatic processes
On decreased the temperature by 100 K, efficiency–
Kerala CEE - 2016
T – 100 T1 – T2
Ans. (c) : The carnot cycle of a reversible heat engine η = 1– 2 =
consists of two isothermal and two adiabatic processes. T1 – 100 T1 – 100
458. A Carnot engine operating between Thus the efficiency will increase.
temperatures T1 and T2 has efficiency 0.2. 460. A Carnot engine whose efficiency is 40%,
When T2 is reduced by 50 K, its efficiency receives heat at 500 K. If the efficiency is to be
increases to 0.4. Then, T1 and T2 are 50%, the source temperature for the same
respectively exhaust temperature is
(a) 200 K, 150 K (b) 250 K, 200 K (a) 900 K (b) 600 K
(c) 300 K, 250 K (d) 300 K, 200 K (c) 700 K (d) 800 K
(e) 300 K, 150 K (e) 550 K
Kerala CEE- 2014 Kerala CEE - 2011
Ans. (b) : By Carnot’s ideal heat engine– Ans. (b) : Carnot efficiency relation–
T T – T2
η =1– 2 η= 1
T1 T1
Where, η1 = 0.2 & η2 = 0.4 T1 – T2
Case–1 = 0.4
For the first condition– T1
T T T1 – T2 = 0.4 T1
η1 = 1 – 2 ⇒ 0.2 = 1 – 2
T1 T1 T2 = 0.6 T1
T1 – T2 T1' – T2
0.2 = .....(i) Case–2 = 0.5
T1 T'1
For the second condition– 0.6 6
T1' = T1 = × 500
T – 50 0.5 5
η2 = 1 – 2
T1 T'1 = 600 K

675
461. A refrigerator with coefficient of performance T
1 Ans. (d) ∵ η =1– 2
releases 200 J of heat to a hot reservoir. T1
3
Then the work done on the working substance 50 273 + 17
Initially, = 1–
is 100 T1
100 1 290
(a) J (b) 100 J = 1− ⇒ T1 = 580K
3 2 T1
200
(c) J (d) 150 J 60 273 + 17
3 Final = 1–
100 T1'
(e) 50 J
Kerala CEE - 2010 3 290
= 1− ⇒ T'1 = 725 K
Ans. (d) : The coefficient of performance of a 5 T1'
refrigerator is given by– Hence, change in source temperature = (725–580) K
Q2 Q2 = 145 K
β= =
W Q1 – Q 2 464. Which of the following statements is incorrect?
(a) All reversible cycles have same efficiency
1 Q2
= (b) Reversible cycle has more efficiency than an
3 200 – Q 2 irreversible one
200 – Q2 = 3Q2 (c) Carnot cycle is a reversible one
4Q2 = 200J (d) Carnot cycle has maximum efficiency in all
Q2 = 50 J cycles
∴ W = Q1 – Q2 UPSEE - 2014
= 200 – 50 = 150 J Ans. (a) : Efficiency of all reversible cycles depends
462. In a Carnot engine, the temperature of upon temperature of source and sink which will be
reservoir is 927ºC and that of sink is 27ºC. If different.
the work done by the engine when it transfers 465. In a Carnot's cycle, the working substance
heat from reservoir to sink is 12.6 × 106J, the absorbs heat Q1 at temperature T1 and rejects
quantity of heat absorbed by the engine from heat Q2 at temperature T2. The change of
the reservoir is entropy during the Carnot's cycle is
(a) 16.8 × 106 J (b) 4 × 106 J Q Q
(c) 7.6 × 10 J6
(d) 4.25 × 106 J (a) 1 (b) 2
T1 T2
(e) 20. 8 × 106 J
Q Q
AIEEE 2003, DCE 2006 (c) 1 + 2 (d) Zero
Karnataka CET 2007 T1 T2
Kerala CEE - 2009 UPSEE - 2011
Ans. (a) : Given, Ans. (d) : In a carnot cycle the working substance
T1 = 927 + 273 = 1200K, absorbs heat Q1 at temperature T1 and rejects heat Q2 at
temperature T2.
T2 = 273 + 27 = 300K
Change in entropy during heat absorption–
T 300 3
For a carnot engine, ( η) = 1 – 2 = 1 – = Q
T1 1200 4 ∆S1 = 1
T1
We know,
Change in entropy during heat rejection–
W
η= Q
Q1 ∆S2 = – 2
T2
W 12.6 ×106 For carnot’s cycle–
∴ Q1 = =
η 3/ 4 Q1 Q 2
=
= 4 × 4.2 × 106 = 16.8 × 106 J T1 T2
463. A Carnot engine with sink’s temperature at Therefore net change in entropy is ∆S1 + ∆S2 = 0
17ºC has 50% efficiency. By how much should 466. A Carnot engine used first an ideal mono
its source temperature be changed to increase atomic gas and then an ideal diatomic gas. If
its efficiency to 60% the source and sink temperature are 410ºC and
(a) 225 K (b) 128ºC 69ºC respectively and the engine extract 1000 J
(c) 580 K (d) 145 K of heat in each cycle, then area enclosed by P-V
Kerala CEE 2007 diagram is
676
(a) 100 J (b) 300 J Ans. (c) : Given,
(c) 500 J (d) 700 J The temperature of freezer (T2) = –13°C
UPSEE - 2010 T2 = –13+273= 260K
Ans. (c) : Efficiency of Carnot engine does not depend Coefficient of performance (β) = 5
upon the nature of gas. The coefficient of performance is defined as–
T1 = 410°C = 410 + 273 = 683K T2 260
T2 = 69°C = 69 + 273 = 342K β= ⇒ =5
T1 – T2 T1 – 260
Q1 = 1000J
260
 T – T2  T1 – 260 =
Area of P–V diagram (W) = Q1  1  5
 T1  T1 – 260 = 52
 683 – 342  T1 = (52+260)K = 312 K
= 1000   = 499.26 J
 683  T1 = (312–273)°C
≃ 500 J T1 = 39°C
467. A heat engine is a device 470. An engine is supposed to operate between two
(a) Which converts mechanical energy into heat reservoirs at temperature 727ºC and 227ºC.
energy The maximum possible efficiency of such an
engine is :
(b) which converts heat energy into mechanical
(a) 1/2 (b) 1/4
energy
(c) 3/4 (d) 1
(c) absorbs heat from a sink at a lower
temperature and rejects to the source at high UPSEE - 2005, MP PET 2006
temperature DCE 1999, BHU 2004
(d) None of the above RPMT 2006
UPSEE - 2009 Karnataka CET 2006
Ans. (b) : Heat engine is a device which converts heat Ans. (a) : Given,
into work (mechanical energy) continuously through a T1 = 727°C = 727+273 = 1000 K
cyclic process. T2 = 227°C = 227 + 273 = 500 K
468. An ideal heat engine exhausting heat at 27ºC is T
η = 1– 2
to have 25% efficiency. It must take heat at T1
(a) 127ºC (b) 227ºC
500 1
(c) 327ºC (d) None of these η = 1− = 1–
UPSEE - 2008 1000 2
UP CPMT- 2002 1
η=
Ans. (a) : Given, 2
1 471. A Carnot engine whose low temperature
T2 = 27°C = (27+273)K = 300K, η=25% = reservoir is at 7ºC has an efficiency of 50%. It
4 is desired to increase the efficiency to 70%. By
T2 how many degrees should the temperature of
η =1–
T1 the high temperature reservoir be increased?
(a) 840 K (b) 280 K
1 300
=1– (c) 560 K (d) 380 K
4 T1 UPSEE - 2005
300 3 Kerala CEE 2007
=
T1 4 Ans. (d) : Efficiency is defined as–
300 × 4 T
T1 = = 400 K η =1– 2
3 T1
T1 = (400–273)°C = 127°C Initially,
469. An ideal refrigerator has a freezer at a 50 273 + 7
= 1–
temperature of -13ºC. The coefficient of 100 T1
performance of the engine is 5. The
1 280
temperature of the air (to which heat is =1–
rejected will be 2 T1
(a) 325ºC (b) 325ºK 280 1
(c) 39ºC (d) 320ºC =
T1 2
UPSEE - 2008
BCECE- 2007, BITSAT- 2010 T1 = 560K

677
Let new temperature of higher temperature reservoir is When temperature of sink is reduced by 40% then its
T1' – new temperature–
T'2 = T2 – 40% of T2
70 280
=1– ' = T2 –0.4 T2
100 T1 T'2 = 0.6T2
280 3 Now, efficiency of Carnot’s engine–
=
T1' 10  T'   T 
η ' =  1 – 2  × 100 = 1 – 0.6 2  ×100
280 ×10  T1   T1 
T1' = = 933K
3  2
=  1 – 0.6 ×  × 100 = 0.6 ×100 = 60%
∴ Increase in temperature = 933 – 560 = 373 K  3
The nearest option is option (d). Hence, correct answer 474. A Carnot's engine has an efficiency of 25%
is 380 K. when its sink is at 27°C. If it has to be
472. A Carnot engine absorbs an amount Q of heat increased to 40%, what should be the
from a reservoir at an absolute temperature T temperature of the sink keeping the
and rejects heat to a sink at a temperature of temperature of the source constant?
T/3. The amount of heat rejected is: (a) 320 K (b) 375 K
(a) Q/4 (b) Q/3 (c) 240 K (d) 300 K
(c) Q/2 (d) 2Q/3 AP EAMCET (22.09.2020) Shift-II
UPSEE - 2004 Ans. (c) : Efficiency of carnot’s engine,
Ans. (b) : We Know that, η = 25% = 0.25
T W Q1 – Q 2 Temperature of sink,
η = 1– 2 = = T2 = 27°C = 273 + 27 = 300K
T1 Q1 Q1
Temperature of source = T1
Where, Q1 = heat absorbed, Q2= heat rejected
T T
T/3 W ∵ η = 1 – 2 ⇒ 0.25 = 1 – 2
1– = T1 T1
T Q1
T2
2 W Q1 – Q 2 = 1 – 0.25 = 0.75
= = T1
3 Q1 Q1
T2 300
2 Q T1 = = = 400 K
= 1– 2 0.75 0.75
3 Q1
T1 = 400 K
Q2 1 When efficiency, η = 40% = 0.4
=
Q1 3 T
1 – 2 = 0.4
Q1 Q T1
Q2 = =
3 3 T2
= 1 – 0.4 = 0.6
473. The efficiency of an ideal Carnot engine T1
working between temperature T1 and T2 is 1/3.
T2 = 0.6T1= 0.6 × 400
If the temperature of the sink is reduced by
40%, then its efficiency will be = 240 K
(a) 50% (b) 25% 475. A Carnot engine takes 3×106 calories of heat
(c) 60% (d) 75% from reservoir at 627°C and gives it to a sink at
27°C. The work done by the engine is
AP EAMCET (22.09.2020) Shift-II
(a) 0 J (b) 8.4×106J
Ans. (c) : Given, 6
(c) 4.2×10 J (d) 16.8×106J
1
Efficiency of carnot engine ( η) = AP EAMCET (22.09.2020) Shift-I
3 BCECE- 2017
T2 Ans. (b) : Given,
η =1–
T1 T1 = 627 + 273 = 900K
Where, Q1 = 3×106 cal = 3×4.2×106 J
T1 = temperature of source T1 = 27 + 273 = 300K
T2 = temperature of sink Efficiency of Carnot engine,
T T T W T

1
=1– 2 ⇒ 2 =1– ⇒ 2 =
1 2 η= = 1– 2
3 T1 T1 3 T1 3 Q T1

678
W T Ans. (c) : Ideal heat engine, efficiency is given as–
= 1– 2
Q T1 W Q1 – Q 2
η= =
W 300 Q1 Q1
=1–
4.2 × 3 × 10 6
900 Q2 T
η = 1– =1– 2
W 2 Q1 T1
=
4.2 × 3 ×106 3 T2
2 =1– η
W = × 4.2 × 3 × 106 T1
3
T1 1
= 8.4 × 106 J = .…(i)
476. An electrical refrigerator with β = 5 extracts T2 1 – η
5000 J from the contents of the refrigerator. When heat engine operated in backward direction, then
During this process, find the electrical energy coefficient of performance given as–
utilised by its motor. Q Q2 T2 1
(a) 1 kJ (b) 0.5 kJ α= 2 = = = ….(ii)
(c) 0.8 kJ (d) 1.2 kJ W Q1 – Q 2 T1 – T2 T1 − 1
AP EAMCET (22.09.2020) Shift-I T2
Ans. (a) : Given, From equation (i) and (ii),
Coefficient of performance of refrigerator– 1 1
β=5 α= =
1 1
Amount of heat removed (Q) = 5000J –1 –1
1– η 1– η
Electrical energy utilized by the motor
= Work done by the motor (W) 1– η 1– η  1 
= =   –1
Q 1–1+ η η η
β=
W 479. When the temperature difference between the
Q 5000 source and sink increases, the efficiency of the
∴ W= = = 1000J heat engine
β 5
= 1kJ (a) Decreases
477. The efficiency of a Carnot's engine is 100% (b) Increases
only when (c) Is not affected
(a) ideal gas is used as a working substance (d) May increases or decrease
(b) temperature of the sink is equal to absolute AP EAMCET (18.09.2020) Shift-II
zero Ans. (b) : The efficiency (η) of the Carnot engine is
(c) source temperature is equal to the temperature given by-
of the sink
(d) source temperature is equal to absolute zero TL
η = 1–
AP EAMCET (21.09.2020) Shift-II TH
Ans. (b) : The efficiency of Carnot’s engine is given as
Where,
T
η =1– 2 TL = Temperature of the sink
T1 TH = Temperature of source
Where, T1 = temperature of source • When the difference between the source and sink
T2 = temperature of sink increases, it's means source temperature increases while
For efficiency 100%,η = 1 sink temperature decreases.
T T • If the temperature of the source (TH) is increases then
1 = 1– 2 ⇒ 2 = 0
T1 T1 the efficiency of the Carnot engine increases.
∴ T2 = 0 480. An ideal Carnot’s engine with an efficiency of
30% operates between a source and a sink. If
478. An ideal heat engine has an efficiency η. The
coefficient of performance of the engine when the temperature of the source is 500 K, that of
driven backward will be the sink is
(a) 27 oC (b) 57 oC
1 1 o
(a) 1-   (b) η −   (c) 77 C (d) 107 oC
 η  η AP EAMCET (17.09.2020) Shift-II
1 1 Ans. (c) : Given,
(c)   − 1 (d)
η 1− η Efficiency of Carnot engine (η) = 30% = 0.3
AP EAMCET (21.09.2020) Shift-I Temperature of source (T1) = 500K

679
We know, 482. A Carnot engine of efficiency 40%, takes heat
T from a source maintained at a temperature of
η =1– 2 500K. It is desired to have an engine of
T1 efficiency 60%. Then, the source temperature
T2 T for the same sink temperature must be
0.3 = 1 – ⇒ 2 = 1 – 0.3 (a) 650 K (b) 750 K
500 500
(c) 550 K (d) 850 K
T2 = 0.7 × 500 = 350K AP EAMCET (21.04.2019) Shift-II
= (350 – 273)°C
Ans. (b) : We know that,
= 77°C Efficiency of Carnot engine,
481. Two heat engines X and Y of same efficiency
 T 
are connected in series in such a way that the η =  1 − 2  × 100
sink of X works as source of Y. X receives heat  T1 
at 900 K and rejects some heat to its sink at TK Where, T2 = sink temperature
and in turn Y rejects heat to its sink at 400 K, T1 = source temperature
then the temperature T is For 40% efficiency, T1 = 500 K
(a) 550 K (b) 600 K 40  T 
= 1 − 2 
(c) 650 K (d) 700 K 100  500 
AP EAMCET (23.04.2019) Shift-I T
Ans. (b) : According to the question a two heat engines 0.4 = 1 − 2
500
are shown in the figure are connected in series T2
combination, 0.6 =
500
T2 = 300K
For 60% efficiency the source temperature for the same
sink temperature,
60 T
= 1− 2
100 T1
T2 6
= 1−
T1 10
T2 4
= = 0.4
T1 10
10
× T2 = T1
4
10
T1 = × 300 [∵ T2 = 300K]
4
Given, T1 = 750K
Temperature of source of X heat engine (T1) = 900K 483. Match the temperature of the source and sink
Temperature of sink of Y heat engine (T2) = 400K (T1 and T2 respectively) of a Carnot heat engine
Let, The Temperature of sink of X which act as source given in List-I with the corresponding
of Y is T2 efficiencies given in List – II.
So, efficiency of X heat engine, List-I List-II
A. T1 = 500 K, T2 = 300 K (i) 0.2
T T
ηX = 1 – 2 = 1 – 2 B. T1 = 500 K, T2 = 350 K (ii) 0.3
T1 900 C. T1 = 800 K, T2 = 400 K (iii) 0.4
T3 400 D. T1 = 450 K, T2 = 360 K (iv) 0.5
Similarly, ηY = 1 – =1– The correct match is
T2 T2
Codes
∵ ηX = ηY A B C D
T2 400 (a) (iii) (iv) (ii) (i)
1– = 1–
900 T2 (b) (iv) (iii) (ii) (i)
2
(c) (iii) (i) (iv) (ii)
T2 = 900 × 400 (d) (iii) (ii) (iv) (i)
T2 = 600K AP EAMCET (21.04.2019) Shift-I
680
Ans. (d) : We know, Efficiency of Carnot engine is, Or
γ−1 γ−1
T T2  V1   V  1
η =1− 2 =  = =
T1 
T1  V2   32V  (32) γ –1
Where, T2 = Sink temperature 7
T1 = Source temperature For diatomic gas, γ =
5
Now, check the efficiencies:
T2 1 1 1 1
300 ∴ = = = =
(a) When T1 = 500 K, T2 = 300 K η = 1 − = 0.4 T1 (32) 5 −1 (25 ) 5 −1 (25 ) 5 4
7 7 2

500
350 Now efficiency of Carnot cycle,
(b) When T1 = 500 K, T2 = 350 K η = 1 − = 0.3 T
500 η =1− 2
400 T1
(c) When T1 = 800 K, T2 = 400 K η = 1 − = 0.5 1
800 = 1−
360 4
(d) When T1 = 450K, T2 = 360K, η = 1 − = 0.2 3
450 η = = 0.75
484. Assertion (A): A room can be cooled by 4
opening the door of a refrigerator in it. 486. A reversible Carnot heat engine converts
Reason (R): Heat always flows from a body at 1
th of its input heat into work. When the
higher temperature to a body at lower 4
temperature. temperature of the sink is reduced by 50K, its
(a) Both (A) and (R) are true and (R) is the 1
correct explanation of (A) efficiency becomes 33 % . The initial
3
(b) Both (A) and (R) are true but (R) is not the temperatures of the source and the sink
correct explanation of (A) respectively are
(c) (A) is true but (R) is false (a) 600 K, 550 K (b) 600 K, 450 K
(d) (A) is false but (R) is true (c) 300 K, 150 K (d) 450 K, 350 K
AP EAMCET (23.04.2018) Shift-2 AP EAMCET (20.04.2019) Shift-1
Ans. (d) : When the door of a refrigerator is kept open, Ans. (b) : Given,
heat rejected by the refrigerator to room will be more Q
than the heat taken by the refrigerator from the room by Work done, W =
4
an amount equal to work done by the compressor.
Where,
Therefore, temperature of room will increase and so it
Q = Heat input
will be warmed gradually.
Let, the source temperature = T1 and sink Temperature
From second law of thermodynamics, heat cannot flow
= T2
from colder body to hotter body.
W Q/4
485. A diatomic ideal gas is used in Carnot’s engine Efficiency, η = =
Q Q
as working substance. During adiabatic
expansion of the cycle, if the volume of the gas Sink Temperature reduced by 50K
increases from V to 32 V, then the efficiency of 1
Then, T2 – 50K and η =
the engine is 4
(a) 0.25 (b) 0.5 T2 1
(c) 0.67 (d) 0.75 η = 1− = .....(i)
T1 4
AP EAMCET (22.04.2018) Shift-II
When temperature of sink is reduced by 50 K,
Ans. (d) : Given
100
efficiency become %
3
100 1
η= =
3 × 100 3
So,
T − 50 1
1− 2 =
For an adiabatic process, T1 3
TVγ−1 = constant T2 50 1
1− + = .....(ii)
∴ T1V1γ−1 = T2 V2 γ−1 T1 T1 3

681
Put the value of equation (i) in equation (ii), T2 '
η2 = 1 −
1 50 1 T1 '
+ =
4 T1 3 73 200
η2 = 1 − = .....(ii)
50 1 1 1 273 273
= − =
T1 3 4 12 η1 200 / 473 273
= = = 0.577
T1 = 600 K η2 200 / 273 473
From equation (i), η1
= 0.577
T
1− 2 =
1 η2
T1 4 489. A Carnot's heat engine works between the
T 3 temperature 427°C and 27°C. What amount of
Then, 2 = heat should it consume per second to deliver
T1 4
mechanical work at the rate of 1.0 kW?
3 (a) 0.417 kcal/s (b) 4.17 kcal/s
T2 = × 600
4 (c) 41.7 kcal/s (d) 0.212 kcal/s
= 450 K BITSAT-2013
487. Freezing compartment of a refrigerator is at Ans. (a) : Given that,
0oC and room temperature is 27.3oC. Work T1 = 427°C = 427 + 273 = 700 K
done by the refrigerator to freeze 1 g of water T2 = 27°C = 27+273 = 300 K
at 0oC is (Lice= 80 cal g-1) W = 1.0 kW
(a) 336 J (b) 33.6 J We know efficiency of heat engine,
(c) 3.36 J (d) 40 J T 700 − 300 400
AP EAMCET (23.04.2018) Shift-1 η = 1− 2 = =
T1 700 700
Ans. (b) : Coefficient of performance of refrigerator is
4
T2 η=
α= 7
T1 − T2 Since,
T2 = 0°C = 0+273 = 273K W
T1 = 27.3°C +273 = 300.3 ≃ 300 K η=
Q
273
α= = 10.11 ≃ 10 W 1
300 − 273 ∴ Q= = = 1.75kJ / sec
η 4
Now, if Q2 is heat extracted and W is the work done,
7
Q Q As we know,
α = 2 or W=
W α 1 cal = 4.186 J.
Where Q2 = heat extracted form 1g of water to make it Hence,
ice. 1.75
Q2 = Q = Lice= 80 cal = 80×4.2 = 336 J Q= = 0.417kcal / sec
4.186
So, W = 336/10
= 33.6 J 490. An ideal engine is working between
temperature 400 K and 300 K. It absorbs 600
488. A Carnot engine works first between 200°C cal heat from the source. The work obtained
and 0°C and then between 0°C and –200°C. per cycle from the engine is
The ratio of its efficiency in the two cases is
(a) 630 J (b) 630 cal
(a) 1.0 (b) 0.577
(c) 0.34 (d) 0.68 (c) 2400 cal (d) zero
BITSAT-2007 CG PET- 2011
Ans. (b) : For case I : T1 = 200°C = 273 + 200 = 473 K Ans. (a) : Given that,
T2 = 0°C + 273 = 273 K T1 = 400 K
T2 = 300 K
T
η1 = 1 − 2 Q = 600 cal
T1
T W
273 200 We know η = 1 − 2 or
η1 = 1 − = .....(i) T1 Q
473 473
W T
For case II : T′1 = 0°C + 273 = 273K = 1− 2
T′2 = –200°C + 273 = 73 K Q T1

682
W 300 100 493. An ideal gas heat engine operates in a Carnot's
= 1− = cycle between 227ºC and 127ºC. It absorbs
600 400 400
6×104 J at high temperature. The amount of
W 100
= heat converted into work is
600 400 (a) 1.6 ×104 J (b) 1.2 ×104 J
100 × 600 600 (c) 4.8 ×10 J4
(d) 3.5 × 104 J
W= = = 150 cal
400 4 Manipal UGET-2012
W =150×4.2 = 630J (1 cal = 4.18 ≃ 4.2J ) Ans. (b) : Given
491. A Carnot engine working between 300 K and T1 = 227°C + 273 = 500 K
600 K has a work output of 800 J per cycle. T2 = 127°C + 273 = 400 K
What is the amount of heat energy supplied to Q = 6×104 J
the engine from source per cycle? W=?
(a) 1200 J/cycle (b) 1600 J/ cycle By using the relation
(c) 1400 J/ cycle (d) None of the above
W T
DPMT 1999 = 1− 2
Punjab PMT 2002,2005 Q T1
Kerala CEE 2004  400 
DCE 2009 W = 6 ×10 4 1 − 
 500 
CG PET- 2007
 100 
Ans. (b) : Given = 6 × 104  
T1 = 300 K  500 
T2 = 600 K = 6×104 (0.2)
W = 800 J W = 1.2 ×104 J
Q=? 494. The efficiency of Carnot engine is η, when its
We know Efficiency of heat engine. hot and cold reservoirs are maintained at
T2 Work done temperature T1 and T2, respectively. To
η = 1− =
T1 Heat taken increase the efficiency to 1.5 η, the increase in
T W temperature (∆T) of the hot reservoir by
1− 2 = keeping the cold one constant at T2 is
T1 Q
T1T2 0.5T2η
300 800 (a) (b)
1− = ( T − η)(1 − 1.5η) (1 − 1.5η)(1 − η)
600 Q
800 1 (c)
T1

T2
(d)
(1 − η)(1 − 1.5η)
=
Q 2 ( T − η) (1 − 1.5η) T1T2
Q = 1600J TS-EAMCET-04.05.2018, Shift-1
Ans. (b) : We know Efficiency of carnot cycle is
492. An ideal heat engine working between
T
temperature T1 and T2 has an efficiency η, the η =1− 2
new efficiency if temperature of both the T1
source and sink are doubled, will be T
(a) η/2 (b) η 1− η = 2
T1
(c) 2η (d) 3η
T1 1
JIPMER- 2007 = .......(i)
CG PET- 2006 T 2 (1 − η)
Ans. (b) : We know that, efficiency of heat engine In second case
T2 To increase the efficiency to 1.5 η temperature
η =1−
T1 by ∆Τ
When temperature of source and sink are double then T2
1.5η = 1 −
2T2 T1 + ∆T
η' = 1−
2T1 T2
1 − 1.5η =
T2 T1 + ∆T
η' = 1−
T1 T1 + ∆T 1
= ..............(ii)
η' = η T2 (1 − 1.5η)

683
Eq (ii) – Eq (i) we get. Ans. (*) : Efficiency of binary cycle is given by
∆T 1 1 ηcc = η1 + η2 − η1η2
= −
T2 1 − 1.5η 1 − η  T   T   T  T 
=  1 − 2  + 1 − 3  − 1 − 2 1 − 3 
1 − η − 1 + 1.5η  T1   T2   T1  T2 
=
(1 − η)(1 − 1.5η) T T T T T T
0.5ηT2 = 1− 2 +1 − 3 −1 + 3 + 2 − 2 . 3
∆T = T1 T2 T2 T1 T1 T2
(1 − η)(1 − 1.5η) T
= 1− 3
495. A Carnot engine with efficiency η operates T1
between two heat reservoirs with temperatures
T1 and T2, where T1 > T2. If only T1 is changed T
⇒ ηcc = 1 − 3
by 0.4%, the change in efficiency is ∆η1, T1
whereas if only T2 is changed by 0.2%, the 497. A Carnot engine whose efficiency is 40%,
∆η1 receives heat at 500 K. If the efficiency is to be
efficiency is changed by ∆η2. The ratio is
∆η2 50%, the source temperature for the same
approximately. exhaust temperature is
(a) +2 (b) –4 (a) 900 K (b) 600 K
(c) +3 (d) +4 (c) 700 K (d) 800 K
TS-EAMCET-04.05.2019, Shift-1 TS-EAMCET-11.09.2020, Shift-1
Ans. (b) : Efficiency of a carnot engine is
Ans. (a) : As we know that,
T
Efficiency of carnot engine η =1− 2
T T1
η =1− 2
T1 Given, η1 = 0.4
T1 = 500K
T1 = Source temperature
T T2
T2 = Sink temperature 0.4 = 1 − 2 , = 1 − 0.4 = 0.6
If T1 is changed by 0.4% 500 500
∆η ∆T1 ∆T2 T2 = 0.6 × 500 = 300 K
= + (Combination of the error ) Now, In second case
η T1 T2
η2 = 0.5, T2 = 300 K
∆η1 0.4
= + 0 ...........(i)  300 
η 100 0.5 = 1 − 
 T1 
Similarly,
T2 is change by 0.2% then 300
= 1 − 0.5
∆η2 0.2 T1
= 0+ ...........(ii)
η 100 300
T1 =
So, form eq (i) and (ii) we get 0.5
∆η1 0.4 T1 = 600K
= =2
∆η2 0.2
498. A Carnot engine absorbs heat from a reservoir
496. A Carnot engine C1 operates between maintained at temperature 1000 K. The engine
temperature T1 and T2 (T1 > T2). A second rejects heat to a reservoir whose temperature is
Carnot engine C2 uses all the heat rejected by T. If the magnitude of a absorbed heat is 400J
the engine C1 and operates between and work performed is 300J, then the value of
temperature T2 and T3 (where T2 > T3). The T is
efficiency of this combined (C1 and C2 together) (a) 250K (b) 500K
engine is (c) 750K (d) 1750K
T T TS-EAMCET-05.05.2018, Shift-2
(a) 1 − 3 (b) 1 − 3
T1 T2 Ans. (a) : Given
T1 = 1000K
( T + T3 )  T  T  Q1 = 400 J
(c) 1 − 2 (d) 1 − 1 − 2  1 − 3 
T1  T1   T2  W = 300 J
TS-EAMCET.14.09.2020, Shift-2 T2 = T

684
Efficiency of carnot engine is Ans. (b) : Given
W T η1 = 50% = 0.5
η= = 1− 2
Q T1 T2 = 500K sink Temperature.
300 T We know that efficiency of carnot engine
= 1− T
400 1000 η =1− 2
T 3 1 T1
= 1− =
1000 4 4 500
0.5 = 1 −
1000 T1
T1 =
4 500
= 0.5
T = 250K T1
499. The freezer in a refrigerator is located at the ∴ T1 = 1000 K
top section so that Now the temperature of sink is changed to T'2 and
(a) The entire chamber of the refrigerator is temperature of source is kept constant and its efficiency
cooled quickly due to convection is to be raised to 60% i.e. 0.6
(b) The motor is not heated
T'
(c) The heat gained from the environment is high 0.6 = 1 − 2
(d) The heat gained from the environment is low 1000
AMU- 2017 T2 '
= 1 − 0.6 = 0.4
JCECE-2014 1000
JCECE-2009 T2′ = 0.4×1000 = 400K
Ans. (a) : Freezer is the source for the refrigerator’s 502. A heat engine is working between 27°C and
coldness. When the freezer is placed on top, the cold air 127°C . If 1000J of heat is absorbed at the
produced from it is denser than the warmer air in the source, the amount of heat rejected to the sink
bottom. So cold air sinks down and the warm air is and the useful work done, in J. are
force to rise up so when the warm air rises up and it get respectively,
cold in the freezer. (a) 250;750 (b) 750;250
This is an example of convection process. (c) 150;850 (d) 400;600;
500. A Carnot engine takes heat from a reservoir at COMEDK 2011
627ºC and rejects heat to a sink at 27ºC. Its Ans. (b) : Given
efficiency will be T1 = 127ºC
(a) 3/5 (b) 1/3 T2 = 27 ºC
(c) 2/3 (d) 200/209 Q1 = 1000 J
JCECE-2009 Q2 = ?
Ans. (c) : Given For Heat engine
T1 = 627°C +273 = 900K Q T
T2 = 27°C + 273 = 300 K η = 1− 2 = 1− 2
Q1 T1
We know efficiency of carnot cycle
T Q 2 T2 Q 300
η =1− 2 ∴ = or 2 =
T1 Q1 T1 1000 400
300 Q2 3
η =1− =
900 1000 4
600 Q 2 = 750 J
η=
900 Work done
2 W = Q1 – Q2
η=
3 = 1000 – 750
501. The efficiency of Carnot engine is 50% and W = 250J
temperature of sink is 500 K. If the 503. Two Carnot engines A and B are operated in
temperature of source is kept constant and its series. The engine A receives heat from the
efficiency is to be raised to 60%, then the source at temperature T1 and rejects the heat to
required temperature of the sink will be : the sink at temperature T. The second engine B
(a) 600 K (b) 400 K receives the heat at temperature T and rejects
(c) 500 K (d) 100 K to its sink at temperature T2. For what value of
JCECE-2003 T the efficiencies of the two engines are equal
685
T1 – T2 (c) If Assertion is correct but Reason is incorrect.
(a) (b) T1T2
2 (d) If both the Assertion and Reason are
T1 + T2 incorrect.
(c) T1T2 (d)
2 AIIMS-26.05.2018(M)
COMEDK 2013 Ans. (a) : Its efficiency is maximum as no dissipation
Ans. (c) : We know efficiency of a Carnot engine of energy takes place against friction in such a heat
T2 engine.
η =1−
T1 506. Assertion: The Carnot cycle is useful in
T1 = Temperature of source understanding the performance of heat
T2 = Temperature of sink engines.
For engine A, Reason : The Carnot cycle provides a way of
T determining the maximum possible efficiency
ηA = 1 − achievable with reservoirs of given
Tl
temperatures.
For engine B, (a) If both Assertion and Reason are correct and
T2 the Reason is a correct explanation of the
ηB = 1 −
T Assertion.
According to question (b) If both Assertion and Reason are correct but
ηA = ηB Reason is not a correct explanation of the
T T2 Assertion.
∴1 − = 1 − (c) If the Assertion is correct but Reason is
T1 T
incorrect.
T T2
= (d) If both the Assertion and Reason are
T1 T incorrect.
T2 = T2.T1 (e) If the Assertion is incorrect but the Reason is
correct.
T = T1T2
AIIMS-2006
504. If sink is at a temperature of –39°C and source Ans. (a) : Carnot cycle represent a process of an ideal
at 0ºC, then efficiency will be heat engine which has a maximum efficiency when
(a) 39.4% (b) 14.2% converting heat energy into mechanical energy.
(c) 35.2% (d) 45.5%
The efficiency of a Carnot depends only on the
COMEDK 2020
temperature of heat source and the cold sink.
Ans. (b) : T2 = (273 –39)K = 234 K
T1 = 0 + 273 = 273 K 507. A Cannot engine whose efficiency is 50% has
an exhaust temperature of 500 K. If the
 T2  efficiency is to be 60% with the same intake
Efficiency =1 −  
 T1  temperature, the exhaust temperature must be
T −T (in K
η= 1 2 (a) 800 (b) 200
T1
(c) 400 (d) 600
273 − 234
η= AIIMS-27.05.2018(M)
273
Ans. (c) :
39
η= = 0.14 2 T
273 Carnot efficiency (η) =1 − 2
T1
η=14.2%
50 500
505. Assertion: The efficiency of a reversible engine =1 −
100 T1
is maximum.
Reason: In such a device no dissipation of T1 = 1000K
energy takes place.
(a) If both Assertion and Reason are correct and Similar, According to condition intake temp. is same
Reason is the correct explanation of
Assertion. 60 T
=1 − 2
(b) If both Assertion and Reason are correct, but 100 1000
Reason is not the correct explanation of T2 = 400 K
Assertion.
686
508. In a heat engine, the temperature of the source At B
and sink are 500 K and 375 K. If the engine 2P0V0 = nRT
consumes 25×105 J per cycle, the work done T = 2T0
per cycle is
(a) 6.25×105J (b) 3×105 J Maximum thermal efficiency of the reversible engine of
5
(c) 2.19 × 10 J (d) 4×104 J Carnot’s cycle
AIIMS-2017 T T 1
η=1 − 0 =1 − 0 = = 50%
Ans. (a) : Given that T 2T0 2
Temperature of source T1 = 500 K η= 50%
Temperature of sink T2 = 375 K
Q1 = 25 × 105 J 510. What is the coefficient of performance of a
Efficiency of a Carnot’s heat engine refrigerator that has a hot reservoir
temperature of 27ºC and a cold reservoir
W T
η= =1 − 2 temperature of –23ºC?
Q1 T1 (a) 4 (b) 5
 T2  (c) 6 (d) 7
W = Q1 1 −  AIIMS-26.05.2019(E) Shift-2
 T1 
Ans. (b) : Given, T1 = 27°C = 27 + 273 = 300 K
 375 
W = 25 × 105  1 −  T2 = –23°C + 273 = 250 K
 500  Coefficient of performance (C.O.P.),
W = 6.25 ×105 J COP =
T2
=
250
509. N moles of a monoatomic gas is carried round T1 – T2 300 – 250
the reversible rectangular cycle ABCDA as COP = 5
shown in the diagram. The temperature at A is 511. Determine efficiency of Carnot cycle if in
T0. The thermodynamic efficiency of the cycle adiabatic expansion volume becomes 3 times of
is: initial value and γ = 1.5
1 1
(a) 1 – (b) 1 –
2 3
1 1
(c) 1 + (d) 1 +
2 3
AIIMS-25.05.2019(M) Shift-1
Ans. (b) : Given that, V1 = V, V2 = 3V, γ = 1.5
For a adiabatic process,
PVγ = C
(a) 15% (b) 50%
or TVγ−1 = C
(c) 20% (d) 25% γ−1 γ−1
AIIMS-2004 T1 (V)1 = T2 V2
Ans. (b) : T1(V)γ−1 = T2 × (3V)γ−1
γ −1
T1  3V 
= 
T2  V 
T1
= ( 3) = ( 3)
γ−1 1.5−1
= 3
T2
T2 1
η =1 − =1 −
T1 3
1
η =1 −
3
Given,
At A temperature (T0) volume (V0) and 512. If sink and source temperature of a
pressure (P0) refrigerator are 4ºC and 15ºC respectively.
Ideal gas equation Then efficiency of refrigerator is
PV = nRT (a) 0.076 (b) 0.0382
At point A (c) 0.019 (d) 1
P0V0 = nRT0 AIIMS-25.05.2019(M) Shift-1

687
Ans. (b) : Given, sink temperature of refrigerator (T2) = 515. Carnot engine efficiency is equal to 1/7. If the
4°C, source temperature of refrigerator (T1) = 15°C temperature of the sink is reduced by 65K, the
Efficiency of refrigerator is efficiency becomes 1/4. The temperature of
 T   source and the sink in the first case are
273 + 4 
η = 1 – 2  = 1 –  respectively.
 T1   273 + 15  (a) 610 K, 520 K (b) 520 K, 606.67 K
 277  11 (c) 606.67 K, 520 K (d) 520 K, 610K
= 1 – =
 288  288 BCECE-2016
= 0.0382 1
Ans. (c) : Given, η =
513. A Carnot engine works between 27ºC and 7
127ºC. Heat supplied by the source is 500 J. W TH − TL
Then heat ejected to the sink is: Carnot engine ( η) = =
QH TH
(a) 1000J (b) 667J
(c) 375J (d) 500J T
η=1 − L
AIIMS-25.05.2019(E) Shift-2 T H

Ans. (c) : Given that, 1 T


=1 − L
T1 = 273 + 27oC = 300K 7 TH
T2 = 273 + 127oC = 400K
TL 6
T1 =
Carnot efficiency ( η) =1 − TH 7
T2
7TL = 6TH ….(i)
T −T
η= 2 1 Similarly,
T2
1 ( T − 65 )
400 − 300 100 =1 − L
η= = 4 TH
400 400
1 TL − 65 3
η= =
4 TH 4
Work done = η × Heat supplied by source 4TL − 260 = 3TH
Heat rejected to the sink 8TL − 520 = 6 TH (∵ 6TH = 7TL)
= Heat supplied – Work done
TL = 520K
1
= 500 − × 500 = 500 – 125 From equation (i), we get–
4
= 375 J 7
TH = × TL
514. For a refrigerator, heat absorbed from source 6
is 800 J and heat supplied to sink is 500 J then 7
find coefficient of performance. TH = × 520 = 606.67K
6
5 8 T = 606.67 K
(a) (b) H
8 5
516. In a Carnot engine, the temperature of
5 3 reservoir is 9270C and that of sink is 1270C. If
(c) (d)
3 5 the work done by the engine when it transfer
AIIMS-25.05.2019(E) Shift-2 heat from reservoir to sink is 12.6×106J. The
Ans. (c) : Given that, quantity of heat absorbed by the engine from
Heat absorbed from source (Q1) = 800 J the reservoir is–
Heat supplied to the sink (Q2) = 500J (a) 18.9 × 106 J (b) 20.5 × 106 J
Coefficient of performance,
(c) 15.7 × 106 J (d) 12.6 × 106 J
Q2 500 5
α= = = BCECE-2018
Q1 − Q 2 800 − 500 3
Ans. (a) : Given that, T1 = 927 + 273 = 1200 K
5 T2 = 127 + 273 = 400 K
COP =
3 W = 12.6 × 106 J, Q1 = ?

688
Solving equation (i) and (ii), we get
T1 = 372 K
5
and T2 = T1
6
5
T2 = × 372
6
T2 = 310K
The temperature of source is 372 K and sink
Q1 = W + Q2 .....(i) temperature is 310 K.
T −T Work done 518. An ideal heat engine exhausting heat at 770C is
Efficiency(η) = 1 2 =
T1 Heat supplied to have 30% efficiency. It must take heat at:
1200 − 400 (a) 1270C (b) 2270C
η= 0
(c) 327 C (d) 6730C
1200
800 2 BCECE-2004
η= =
1200 3 Ans. (b) : Given that,
2 W η = 30 % = 0.30
η= =
3 Q1 TE = 77oC = 77 + 273 = 350 K
3 Heat engine efficiency,
Q1 = W × Q T
2 ηH.E. =1 − L =1 − L
3 QH TH
Q1 =12.6 ×106 ×
2 350
0.30 =1 −
Q1 = 18.9 × 106 J TH
517. A Carnot reversible engine converts 1/6 of heat −350
input into work. When the temperature of the 0.30 − 1 =
sink is reduced by 62K, the efficiency of TH
Carnot's cycle becomes 1/3. The temperature of 350
the source and sink will be– – 0.70 = –
TH
(a) 372K, 310K (b) 181K, 150K
(c) 472K, 410K (d) none of the above TH = 500K
BCECE-2007 or TH = 227°C
Ans. (a) : Given that, 519. For a heat engine operating between
1 1 temperature t1 ºC and t2 ºC, its efficiency will
η1 = , η2 =
6 3 be
We know that t –t t –t
W 1 (a) 1 2 (b) 1 2
η1 = = t 2 t 1 + 273
Q 6
t t
T2 1 (c) 1 (d) 1 – 2
η1 =1 − = t2 t1
T1 6
MHT-CET 2020
T1 − T2 1
= T
T1 6 Ans. (b) : The efficiency of the cycle ( η) = 1 – 2
T1
6T1 − 6T2 = T1
Let, T1 = t1 + 273
6T1 − 6T2 − T1 = 0
5T1 = 6T2 T 2 = t2 + 273
T t + 273
T2 5
= .... (i) ∴ η =1– 2 =1– 2
T1 6 T1 t1 + 273
In second case t –t
η= 1 2
1 T1 – (T2 − 62) t1 + 273
=
3 T1 520. A thermodynamic system undergoes a cyclic
T1 = 3T1 – 3T2 + 186 process ABC as shown in the diagram. The
2T1 − 3T2 = −186 ....(ii) work done by the system per cycle is :

689
(c) η = 1 – (60 / 80) = 25%
(d) η = 1 – (40 / 60) = 33%
∴ The least efficiency of carnot’s engine is in case of
option (a).
523. A Carnot engine working between 300 K and
400 K has 800 J of useful work. The amount of
heat energy supplied to the engine from the
source is :
(a) 2400 J (b) 3200 J
(c) 1200 J (d) 3600 J
Karnataka CET-2016
(a) 750 J (b) –1250 J
(c) –750 J (d) 1250 J Ans. (b) : Given that,
Karnataka CET-2019 TH = 400K
TL = 300K
Ans. (c) : A thermodynamic system undergoes a cyclic
process ABC. The work done by the system can be W = 800J
predicted by calculating the total area under the curve. T W
The efficiency of carnot engine η = 1 – L =
1 TH Q H
Work done by system = × ( 5 – 10 ) × ( 400 – 100 )
2 300 800
1− =
1500 400 Q H
W =–
2 1 800
W = –750J =
4 QH
521. A Carnot engine takes 300 calories of heat from
a source at 500 K and rejects 150 calories of ⇒ QH = 3200J
heat to the sink. The temperature of the sink is 524. The efficiency of a Carnot engine which
: operates between the two temperature T1 = 500
(a) 125 K (b) 250 K K and T2 = 300 K is :
(c) 750 K (d) 1000 K (a) 75% (b) 50%
Karnataka CET-2018 (c) 40% (d) 25%
Ans. (b) : Given that, Karnataka CET-2015
Q1 = 300 cal, Q2 = 150 cal. Ans. (c) : Given that,
T1 = 500 K, T2 = ? T1 = 500K, T2 = 300K
The efficiency of a Carnot engine is,
T
T Q Efficiency of carnot's engine ( η) = 1 – 2
1– 2 = 1– 2 T1
T1 Q1
 300 
T2 150 ∴ η = 1 –  × 100
=  500 
500 300
= 0.4×100
150
T2 = 500 × η = 40%
300
525. What is the source temperature of the Carnot
T2 = 250K engine required to get 70% efficiency?
Hence, the temperature of the sink is 250K. Given, sink temperature = 270C.
522. For which combination of working (a) 10000C (b) 900C
0
temperatures, the efficiency of Carnot's engine (c) 270 C (d) 7270C
is the least ? Karnataka CET-2014
(a) 100 K, 80K (b) 40 K, 20 K Ans. (d) : Given,
(c) 80 K, 60 K (d) 60 K, 40 K Efficiency η = 70%
Karnataka CET-2017 Sink temperature (T ) = 27°C
2
Ans. (a) : We know that the efficiency of Carnot’s = 27 + 273 = 300 K
TL
Engine ( η) = 1 – Source temperature (T 1) = ?
TH As we know that efficiency is given by
(a) η = 1 – (80 /100) = 20% T
η = 1− 2
(b) η = 1 – (20 / 40) = 50% T1

690
300 528. If γ is the ratio of specific heats and R is the
70% = 1 − universal gas constant, then the molar specific
T1
heat at constant volume CV is given by:
70
= 1−
300
(a) γR (b)
( γ − 1) R
100 T1 γ
300 R γR
0.7 = 1 − (c) (d)
T1 γ −1 γ −1
300 Karnataka CET-2008
= 1 − 0.7
T1 Ans. (c) : Gas constant (R): It is the proportionality
constant used to relate the energy scale to the
300
T1 = temperature scale.
0.3 R= CP – Cv (i)
T1 = 1000 K Heat capacity ratio is the ratio of specific heat capacity
or T1 = 1000 – 273 = 7270C at constant pressure to that of constant volume.
526. For which combination of working Cp
temperatures of source and sink, the efficiency γ =
Cv
of Carnot's heat engine is maximum ?
(a) 600 K, 400 K (b) 400 K, 200 K CP = γ CV (ii)
(c) 500 K, 300 K (d) 300 K, 100 K Put the value of C p in equation (i)
Karnataka CET-2013 γ C v – C v = R
T Cv (γ – 1) = R
Ans. (d) : Efficiency of carnot engine η = 1 – L R
TH Cv =
γ –1
400
(a) η1 = 1 – = 0.33 529. A Carnot engine taken heat from a reservoir at
600
6270C and rejects heat to a sink at 270C. Its
200 efficiency will be :
(b) η2 = 1 – = 0.50
400 (a) 3/5 (b) 1/3
300 (c) 2/3 (d) 200/209
(c) η3 = 1 – = 0.40 Karnataka CET-2006
500
100 Ans. (c) : Given that,
(d) η4 = 1 – = 0.66 T1 = 627°C = 627 + 273 = 900 K
300
T2 = 27°C = 27 + 273 = 300 K
So the efficiency of carnot’s heat engine is maximum
We know, efficiency of carnot engine,
for TL = 300K and TH = 100K
T
Hence, Option (d) is correct. η =1– 2
527. The efficiency of Carnot's heat engine is 0.5 T1
when the temperature of the source is T1 and 300 1
that of sink is T2. The efficiency of another η = 1– = 1–
900 3
Carnot's heat engine is also 0.5. the
2
temperatures of source and sink of the second η=
engine are respectively. 3
T 530. A monoatomic gas is suddenly compressed to
(a) 2T1, 2T2 (b) 2T1 , 2 (1/8) th
of its initial volume adiabatically. The
2
ratio of its final pressure to the initial pressure
(c) T1 + 5, T2 − 5 (d) T1 + 10, T2 − 10
is (given the ratio of the specific heat of the
Karnataka CET-2010 given gas to be 5/3)
Ans. (a) : We Know that efficiency of carnot engine (a) 32 (b) 40/3
T (c) 24/5 (d) 8
η = 1– L
TH Karnataka CET-2006
Where, TL → low temperature (sink) Ans. (a) : Given that,
Initial volume = V
TH → high temperature (Source)
Efficiency remains same when TL and TH are increased V
Final volume =
by same ratio. 8

691
5 T2
Adiabatic index, γ = Efficiency of a carnot engine η = 1 –
3 T1
We know for adiabatic process PVγ = constant Where T1 is the temperature of the source and T2 is the
P1 V1γ = P2 V2γ temperature of the sink.
γ T
P2  V1  η =1– 2
=  T1
P1  V2 
300
γ 0.25 = 1 –
  T1
P2  V 
=  300
P1  V  0.75 =
  T1
 8
P2 300
= (8)
5/3 T1 = = 400 K
P1 0.75
5
T1 = 400 – 273
P2
P1
( )
= 23 3 ⇒
P2
P1
= 25 T1 = 127°C
533. A Carnot cycle has the reversible processes in
P2 the following order:
= 32 (a) Isothermal expansion, adiabatic expansion,
P1
isothermal compression and adiabatic
531. A Carnot's engine is made to work between compression.
2000C and 00C first and then between 00C and (b) Isothermal compression, adiabatic expansion,
–2000C. The ratio of efficiencies of the engine in isothermal expansion, and adiabatic
the two cases is: compression.
(a) 1 : 1.73 (b) 1.73 : 1 (c) Isothermal expansion, adiabatic, compression,
(c) 1 : 2 (d) 1 : 1 isothermal compression, and adiabatic
Karnataka CET-2002 expansion.
Ans. (a) : As we know that the efficiency of Carnot’s (d) Adiabatic expansion, isothermal expansion,
T adiabatic compression and isothermal
engine ( η) = 1 – 2 compression.
T1
st J&K CET- 2010
In 1 case when temperature is between 200°C to 0°C.
T2 = 0°C = (0 + 273) K = 273 K Ans. (a) : A carnot cycle is define as an ideal reversible
closed thermodynamic cycle in which four successive
T1 = 200°C = (200 + 273) K = 473 K
operations are involved. isothermal expansion, adiabatic
273
η1 = 1 – = 0.423 expansion, isothermal compression and adiabatic
473 compression. During these operations, the expansion
For 2nd case when temperature is between 0°C to – and compression of the substance can be done up to the
200°C desired point and back to the initial state.
T1 = (0° + 273) = 273 K
T2 = (– 200 + 273) = 73 K
73
η2 = 1 – = 0.733
273
0.423 1
∴ η1 : η2 = = = 1:1.73
0.733 1.73
532. The temperature of the sink of a Carnot engine
is 27°C and its efficiency is 25%. Then
temperature of the source is
(a) 227°C (b) 27°C
(c) 327°C (d) 127°C
J&K CET- 2011 534. A Carnot's engine working between 27°C and
127°C has a work output of 200 J per cycle.
Ans. (d) : Given that,
The energy supplied to the engine from the
T2 = 27°C = 27 + 273 = 300 K source in each cycle is
25 (a) 400 J (b) 800 J
η = 25% = = 0.25
100 (c) 600 J (d) 500 J
T1 = ? J&K CET- 2009
692
Ans. (b) : Given that, T1 = 1500°C + 100°C = 1600°C
T2 = 27°C = 27 + 273 = 300 K T1 = (1600 + 273) K = 1873 K
T1 = 127°C = 127 + 273 = 400 K 1873 – 773 1100
η= = = 0.587 ≃ 0.59
Work done per cycle W = 200 J 1873 1873
The efficiency of engine is defined as the ratio of work η = 59%
done to the heat supplied 537. The efficiency of a frictionless engine can be
W T 100% if the temperature of the sink is
η= = 1– 2
Q T1 (a) 0° C
(b) 0 K
300 1
η =1– = (c) equal to that of source
400 4 (d) less than that of the source
Since J&K CET- 1998
W Ans. (b) : The efficiency of a friction less engine can be
η= 100%. If the temperature of the sink is 0 Kelvin.
Q
T
1
=
200 Efficiency of engine η = 1 – L
4 Q TH
Q = 800 J Where, T L is temperature of the sink.
At TL = 0 K (For friction less engine)
535. An ideal gas heat engine operates in a Carnot
η = 100%
cycle between 227°C and 127°C. It absorbs
6×10 cals at the higher temperature. The
4 538. The efficiency of a Carnot engine is 60%. If the
amount of heat converted into work is temperature of the sink is 27°C, the
4 4 temperature of the source is
(a) 4.8×10 Cal (b) 1.2×10 Cal (a) 187.5°C (b) 207°C
(c) 3.5×104 Cal (d) 1.6×104 Cal (c) 477°C (d) 750°C
AP EAMCET-03.09.2021, Shift-II J&K CET- 1998
AIPMT-2005, 2006 Ans. (c) : Given that
Karnataka CET-2004 Efficiency of engine = 60%
J&K CET- 2003, 2001 Sink Temperature, T2 = 27°C = 27 + 273
Ans. (b) : Given that, T2= 300 K
T2 = 127°C = (127 + 273) K = 400 K Source Temperature,T1 = ?
T1 = 227°C = (227 + 273) K = 500 K As we know that,
W TL T – T2
For Carnot cycle, = 1– Efficiency of carnot engine η = 1
Qh TH T1
The amount of heat converted into work T – 300
 400  η= 1
W = 1 – T1
 × 6 × 10
4

 500  60 T1 – 300
W = 1.2 × 104 Cal. =
100 T1
536. The temperatures T1 and T2 of heat reservoirs
in the ideal Carnot engine are 1500°C and 3 T – 300
= 1
500°C respectively. If T1 increases by 100°C. 5 T1
What will be the efficiency of the engine? 5(T1 – 300) = 3T1
(a) 62% (b) 59% 5T1 – 1500 = 3T1
(c) 95% (d) 100% 2T1 = 1500
J&K CET- 2002 1500
Ans. (b) : Given that, T1 =
2
T1 = 1500°C = 1500 + 273 = 1773 K T1 = 750 K = (750 – 273)°C
T2 = 500°C = 500 + 273 = 773 K T1 = 477°C
We know, efficiency of carnot engine, 539. Calculate the efficiency of the engine if Carnot
T T – T2 cycle operates at T1 = 550 K and T2 = 320 K
η = 1– 2 = 1 producing 2.3 kJ of mechanical work per
T1 T1
cycle?
It is clearly given that T1 is increased by 100°C (a) 0.418 (b) 0.622
T1 – T2 (c) 0.823 (d) 0.902
η=
T1 J&K-CET-2014
693
Ans. (a) : Given that, T2 = 240 K
T1 = 550 K T
η =1– 2
T2 = 320 K T1
T2
Efficiency of carnot engine η = 1 – 20
=1–
240
T1 100 T1
320 240 1 4
= 1– = 1– =
550 T1 5 5
= 1– 0.582
5
η = 0.418 T1 = × 240
4
540. The efficiency of a Carnot engine kept at the T1 = 300 K
temperatures of 27°C and 127°C is
543. A refrigerator is to maintain eatables kept
(a) 20% (b) 25% inside at 9 degree Celsius. If the room
(c) 30% (d) 40% temperature is 36 degree Celsius, then what is
J&K-CET-2013 the coefficient of performance?
Ans. (b) : Given that, (a) 10.44 (b) 20.62
T2 = 27°C = 27 + 273 = 300 K (c) 1.44 (d) 19.82
T1 = 127°C = 127 + 273 = 400 K J&K-CET-2019
We know that, AMU- 2015
T2 Ans. (a) : Given that,
Efficiency of a Carnot engine ( η) = 1 –
T1 Temperature inside the refrigerator
Tlow
 300 
η = 1 –  × 100 = 9°C = (9 + 273) K = 282 K
 400  Room temperature, THigh = 36°C = (36+273) K = 309 K
η = 0.25 × 100 TL
Coefficient of performance (β) =
η = 25% ( TH – TL )
541. An ideal Carnot’s engine works between 227°C 282 282
and 57°C. Find the efficiency of the engine = =
(a) 22% (b) 34% ( 309 – 282 ) 27
(c) 55% (d) 13.5% = 10.44
J&K-CET-2017 544. A heat engine absorbs 360 J of energy by heat
and performs 25 J of work in each cycle. The
Ans. (b) : Given that, energy expelled to the cold reservoir in each
T1 = 227°C = (227 + 273) K = 500 K cycle is
T2 = 57°C = (57 + 273) K = 330 K (a) 335 J (b) 14.4 J
T2 (c) 360 J (d) 385 J
Efficiency of Carnot engine η = 1 –
T1 UPSEE - 2016
Ans. (a) : According to first law of thermodynamics.
330
η =1– = 0.34
500 ∫Q=∫W
η = 34% Q1 – Q2 = W
or
542. The temperature of source of a Carnot engine
of efficiency 20% when the heat exhausted is at QH – QL = W
240 K is 360 – QL = 25
(a) 1200 K (b) 600 K ⇒ Q L = 335 J

(c) 540 K (d) 300 K 545. Consider an engine that absorbs 130 cal of heat
from a hot reservoir and delivers 30 cal heat to
J&K-CET-2016 a cold reservoir in each cycle. The engine also
Ans. (d) : The efficiency of a carnot engine is consumes 2 J energy in each cycle to overcome
T2 friction. If the engine works at 90 cycles per
η =1– minute, what will be the maximum power
T1
delivered to the load? [Assume the thermal
Where T1 is the temperature of source and T2 is the equivalent of heat is 4.2 J/cal]
temperature of sink in Kelvin (a) 816 W (b) 819 W
η = 20% (given) (c) 627 W (d) 630 W
T1 = ? WB JEE 2020
694
Ans. (c) : Given, work done in one cycle T2
= [(130 – 30)×4.2 – 2] J =1 − 0.6 = 0.4
400
W = 100 × 4.2 – 2 T2 =160K = (160 − 273)o C
W = (420 – 2)
W = 418 J T2 = −113o C
Work done for 90 cycle, W = 90 × 418 548. The relation between seebeck coefficient S and
W Peltier coefficient π is given by
Power(P) =
t π
(a) S = πT (b) S =
90 × 418 T
P=
60 π2 π
P = 627 W (c) S = (d) S = 2
T T
546. A Carnot's engine has an efficiency of 50% at UP CPMT-2012
sink temperature 50°C. Calculate the
Ans. (b) : We know that-
temperature of source.
(a) 133°C (b) 143°C dε
Peltier coefficient, π = T
(c) 100°C (d) 373°C dT
BCECE- 2008 dε
Seebeck coefficient, S =
UP CPMT-2007 dT
Ans. (d) : Given, η = 50% π
∴ π = TSor S =
Temperature of the sink, T
T2 = 50°C = 50+273 = 323K • Peltier coefficient- The amount of heat energy
Temperature of source ⇒ T1 = ? absorbed or evolved at one of the junctions of a
T thermocouple when one ampere current flows for one
η=1– 2 second (One coulomb) is called Peltier coefficient. it is
T1
denoted by π.
50 T
= 1– 2 549. A thermoelectric refrigerator works on
100 T1 (a) Joule effect (b) Seebeck effect
T2 (c) Peltier effect (d) Thermionic effect
= 1 – 0.5 UP CPMT-2005
T1
Ans. (c) : Thermoelectric refrigerator work on Peltier
323 effect. Peltier effect explains that a thermal gradient
= 0.5
T1 exists at the junctions of dissimilar conductors when an
323 electrical current flows between them.
T1 = 550. If the door of refrigerator is opened while
0.5
connected to supply, the room gets
T1 = 646 K
(a) Cooled
T1 = 373 C (b) Heated
547. The efficiency of a Carnot engine is 60%. If the (c) No effect
temperature of source is 127oC. The sink must (d) Temperature is not given
be maintained at UP CPMT-2003
(a) 113 K (b) + 113oC Ans. (b) : A refrigerator is a heat engine it extracts heat
(c) −113 C
o
(d) −113 K from low temperature reservoir and transfer it to high
UP CPMT-2012 temperature. If a refrigerator’s door is kept open, then
Ans. (c) : We know that, efficiency of a Carnot engine room will become hot, because then refrigerator exhaust
is more heat into the room than earlier. In this way,
T temperature of the room increases and room becomes
η =1 − 2 hot.
T1 Thus, it exhausts more heat into the room than it extract
Where T1 and T2 are the temperature of the source and from it. Thus, a room cannot be cooled by keeping the
sink respectively. door of a refrigerator open.
60 551. Efficiency of engine working at 40°C and 20°C
Here η = 60% = = 0.6
100 is
T1 = 127oC = (127 + 273) K = 400 K, T2 = ? (a) 0.064 % (b) 0.64 %
T (c) 64 % (d) 6.4 %
∴ 0.6 =1− 2
400 UP CPMT-2003

695
Ans. (d) : Given, −500
T1 = 40°C −0.5 =
T1
T1 = (273 + 40) K = 313 K T1 = 1000 K
T2 = 20°C In the second case,
T2 = (20 + 273) K = 293 K
η = 80% = 0.8 T2 = 500 K, T1' = ?
As we know that,
 T T2
Efficiency of the engine (η) = 1− 2 ×100 η = 1−
 T1  T1'
−T2
 293  0.8 − 1 =
= 1− ×100
 313  T1'
 313 − 293  −500
=  −0.2 =
×100 T1'
 313 
20 T1' = 2500 K
= ×100
313 The increment in the source temperature is,
= 0.0638 × 100 T = T1' − T1
= 6.38 % = 2500 – 1000
= 6.4 % = 1500 K
552. A heater is switched on. It attains temperature 554. The following figure shows a Carnot engine
T. After some time it is switched off. If that works between temperatures T1 = 400 K
surroundings are at temperature T0. Then, and T2 = 200 K and drives a Carnot
(a) it is cooled to temperature T0 and in the refrigeration that works between temperatures
process it radiates heat to surroundings. T3 = 350 K and T4 = 250 K. The
(b) it is cooled to temperature less then T0 Q
because surrounding continue to absorb heat. quantity 3 will be
Q1
(c) it is cooled to temperature T0 and in this
process it radiates heat to surroundings and
also receive from surroundings
(d) It is cooled to temperature T0 and then its
temperature begins to rise because initially
heater loses heat and after some time it
receive radiations from surroundings.
UP CPMT-2001
Ans. (c) :
A heater is switched on. It attains temperature T. After
sometime it is switched off if the surrounding
temperature is T0 then it is cooled to temperature T0 and
in this process it radiates heat to surrounding and also (a) 1.5 (b) 2.0
receives from surrounding. (c) 2.25 (d) 1.75
553. The Carnot heat engine have an efficiency of TS EAMCET 05.08.2021, Shift-II
50%. The temperature of sink is maintained at Ans. (d) : The efficiency of the engine is defined by
500K. To increase the efficiency upto 80%, the
increment in the source temperature is W
E=
(a) 1500 K (b) 2500 K Q1
(c) 500 K (d) 2000 K Also,
TS EAMCET 05.08.2021, Shift-I T −T W T1 − T2
E= 1 2 ⇒ =
Ans. (a) : In the first case, T1 Q1 T1
η = 50% = 0.5, T2 = 500K The coefficient of performance of the refrigerator is
T1 = ? defined by,
We know, efficiency of heat engine K = Q4/W
T T4
η = 1− 2 And K=
T1 T3 − T4
500 Q4 T4
0.5 = 1 − ⇒ =
T1 W T3 − T4

696
The work done by the engine is used to drive the Ans. (b) : Given : η = 30% = 0.3
refrigerator so w is the same T2 = 77ºC + 273 = 350 K, T1 = ?
(T − T ) Q
W= 1 2 1
We know, Carnot efficiency,
T1 Τ
Then η = 1− 2
Τ1
T4 Q Q3T1
And = 3 −1 = −1
T3 − T4 W Q1 ( T1 − T2 ) 350
0.3 = 1 − ⇒ T1 = 500 K,
T1
Q3  T4   T − T   T3   T1 − T2 
∴ = + 1  1 2  =    T1 = (500 – 273)°C = 227º C
Q1  T3 − T4   T1   T3 − T4   T1 
557. A Carnot engine working between 200 K and
1 − ( T2 / T1 ) 500 K has a work output of 900 J per cycle. The
=
1 − ( T4 / T3 ) amount of heat energy supplied to the engine
from the source in each cycle is
∴ (a) 1000 J (b) 1600 J
200 1 (c) 1500 J (d) 900 J
1− 1−
Q3
= 400 = 2 = ( 2 − 1) / 2 = 1/ 2 = 1 × 7 = 7 SRMJEEE - 2014
Q1 1 − 250 1 − 5 ( 7 − 5 ) / 7 2 / 7 2 2 4 Ans. (c) : Given, T1 = 500 K, T2 = 200 K
350 7 W = Q1 – Q2 = 900 J/cycle
Q3 Efficiency of Carnot engine
= 1.75 Τ W Q1 – Q 2
Q1 η = 1− 2 = =
Τ1 Q1 Q1
555. Consider a two stage Carnot engine. In the first
stage heat Q1 is absorbed at temperature T and 200 3
η = 1− = ;
heat Q2 is expelled at temperature αT (where α 500 5
< 1). In the second stage heat Q2 is absorbed at 3 Q − Q2
temperature αT and heat Q3 is expelled at ∴ = 1
5 Q1
temperature βT (β < α). The efficiency of the
Carnot engine will be 3 Q − Q 2 900 5
∴ = 1 = ⇒ Q1 = × 900 = 1500J
(a) 1 – α – β (b) 1 – α 5 Q1 Q1 3
(c) 1 – β (d) 1 – αβ 558. A Carnot engine working between 27ºC and
TS EAMCET 04.08.2021, Shift-I 127ºC, draws 600 J of heat from the reservoir
Ans. (c) : Consider, a two stage Carnot engine. in one cycle. The work done by the engine
The total work done by the two stage system is, (a) 100 J (b) 150 J
W1+W2. (c) 200 J (d) 250 J
The heat intake of the system Q1, SRMJEEE - 2016
So, Efficiency, Ans. (b) : Given :
Sink temperature (T2) = 27 + 273 = 300 K
W + W2 ( Q1 − Q 2 ) + ( Q 2 − Q3 ) Q Source Temperature (T1) = 127ºC = 127 + 273 = 400 K,
η= 1 = = 1− 3
Q1 Q1 Q1 Q1 = 600 J
T1 = T  T W
Efficiency of the Carnot engine η = 1 − 2 =
Q Q Q   T1 Q
Now, 1 = 2 = 3 T2 = αT ( α < 1) 
T1 T2 T3   300 1
∴ η = 1− =
T3 = βT ( β < α )  400 4
Where we assume Q2 is absorbed by the second stage at Work done by engine W = ηQ1
temperature T2 = αT. This implies the efficiency can be 1
written, ∴ W = × 600J = 150J
4
T βT 559. The temperature of source and sink of a heat
η = 1− 3 = 1−
T1 T engine are 127oC and 27oC, respectively. An
inventor claims its efficient to be 26%, then
η = 1− β
(a) it is impossible
556. An ideal heat engine exhausting heat at 77ºC is (b) it is possible with high probability
to have 30% efficiency. It must take heat at (c) it is possible with low probability
(a) 127ºC (b) 227ºC (d) Data are insufficient
(c) 327ºC (d) 673ºC JIPMER-2017
SRMJEEE - 2012 AIPMT-2001
697
Ans. (a) : Given, T1 = 127°C + 273 = 400K Ans. (d) : Given,
T2 = 27°C + 273 = 300K and η = 26% T1 = 427°C + 273 = 700 K
Maximum efficiency is obtained by– T2 = 177°C + 273 = 450 K
Heat input (Q) = 1k cal
 T 
η = 1 – 2  ×100 T
 T1  So, η=1– 2
T1
 300 
= 1 –  ×100 η = 1–
450 5
=
 400 
700 14
= 25%
Now, Work done = η × heat input (Q)
∴ Efficiency 26%, Hence, it is impossible for this heat
5
engine. = × 1k cal
14
560. In Carnot engine efficiency is 40% at hot
reservoir temperature T. For efficiency 50%, = 0.36 k cal
what will be temperature of hot reservoir? = 0.36 × 4.18 kJ
= 1.51 kJ
T 2T
(a) (b) 562. Temperature of a cold reservoir of a Carnot
5 5
engine is 1270C. If the efficiency of the Carnot
6 engine is 20%, then the temperature of the hot
(c) 6T (d) T
5 reservoir is–
UPSEE - 2015, 2006 (a) 5000C (b) 2270C
0
JIPMER-2011 (c) 273 C (d) 4000C
Ans. (d) : We know that, efficiency of a Carnot engine AP EAMCET-08.07.2022, Shift-I
is, Ans. (b) : Given, Temperature of cold reservoir

Efficiency (η) =
Work done W
= = 1– 1
T ( T2 ) = 127 C + 273 = 400K
Work input Q T and Efficiency ( η) = 20% = 0.20
Where, T1 = temperature of sink
T1 = ?
T = temperature of source (hot reservoir)
Efficiency is 40% at hot reservoir temperature T. We know,
T2
T Efficiency of Carnot engine ( η ) = 1 −
0.4 = 1− 1 T1
T
T1 400
= 0.6 0.20 = 1 −
T T1
T1 = 0.6 T ____(i) 400
0.80 =
Let the hot reservoir temperature T ' for efficiency 50% T1
T1 400
0.5 = 1− T1 =
T' 0.80
T1 T1 = 500K
= 0.5
T' Then,
Putting the value of T1 from equation (i) T1 = 500 − 273 = 227°C
0.6T 563. A Carnot engine operating between
= 0.5
T' temperatures 600K and 300K absorbs 800 J of
0.6 heat from the source. The mechanical work
T' = T done per cycle is
0.5
(a) 400 J (b) 650 J
6
T' = T (c) 750 J (d) 600 J
5 AP EAMCET-05.07.2022, Shift-II
561. The maximum amount of work that a Carnot Ans. (a) : Given,
engine can perform per kilocalorie of heat T1 = 600K, T2 = 300K
input if it absorbs heat at 427°C and releases
Q = 800 J , W = ?
heat at 177°C is
Efficiency of a Carnot cycle is given by,
(a) 2.39 kJ (b) 6.66 kJ
T W
(c) 4.66 kJ (d) 1.51 kJ η=1– 2 =
Assam CEE-31.07.2022 T1 Q

698
300 W Ans. (b) : Efficiency of Carnot engine is given by,
1− =
600 800 T
η =1− 2
300 W T1
=
600 800 Where T1 is temperature of source and T2 is temperature
W= 400 J of sink.
564. When the absolute temperature of the source of Hence, we see that efficiency depend on the
a Carnot heat engine is increased by 25 %, its temperature of sink and source only and not on the
efficiency increases by 80%. The new efficiency substance. So efficiency is independent of the working
of the engine is
substance.
(a) 12% (b) 24%
(c) 48% (d) 36% 566. Work done by a gas in the process shown in the
AP EAMCET -2016 figure will be ---------
Ans. (d) : Let the initial efficiency of engine be η and
T1 is an absolute source temperature.
When it is increased by 80% the new efficiency
80
= η+ η
100
= η + 0.8η = 1.8 η
∴According to the question, (a) Positive (b) Negative
100 − T2 (c) Zero (d) Can't be determined
η= ....... (i) AP EAMCET-24.08.2021, Shift-I
100
125 − T2 Ans. (b) :
1.8 η = ........(ii)
125
From equation, (i) and (ii)
We get,
η 100 − T2 125
= ×
1.8η 100 125 − T2
1 100 − T2 125 The work of clock wise cycle is positive i.e. Turbine
= × while the work of anti clockwise cycle is negative i.e.
1.8 100 125 − T2
compressor.
5 100 − T2 5 Hence, given cycle is anti clockwise so work done by
= ×
9 4 (125 − T2 ) gas is negative.
9 (100 – T2) = 500 – 4T2 567. By leaving the door of a small standard
900 – 9T2 = 500 – 4T2 domestic refrigerator open, it is not possible to
400 = 5T2 cool a room, because is violates the –––––
400 (a) First law of thermodynamics
T2 = = 80 K
5 (b) Second law of thermodynamics
Then new efficiency, (c) Law of conservation of momentum
T 80 (d) Law of conservation of energy
η' = 1− 2 = 1−
T1 125 AP EAMCET-06.09.2021, Shift-I
125 − 80 45 Ans. (b) : Clausius' statement of second law states that
= =
125 125 it is impossible to transfer heat from lower temperature
45 to higher temperature without help of external agent.
η' = ×100 = 36% So, to remove heat from room, taking room as a system,
125
heat must be rejected to surrounding. So, the situation
565. The efficiency of Carnot heat engine ––––– violates second law of thermodynamics.
(a) is independent of the temperature of the
source and the sink 568. A Carnot engine whose heat sink is at 27°C has
(b) is independent of the working substance an efficiency of 40% By how much should its
(c) can be 100% source temperature be changed so as to
(d) is no affected by the thermal capacity of the increase its efficiency to 60%?
source or the sink. (a) 250 K (b) 100 K
AP EAMCET-24.08.2021, Shift-I (c) 500 K (d) 350 K
NEET OCT-2020 AP EAMCET-20.08.2021, Shift-II
699
Ans. (a) : Temperature of sink, TL = 300K Efficiency is given as 40%
η = 40% = 0.4 40 T
=1– L
TL 100 500
η = 1− TL 60
TH =
500 100
300 TL = 300 K
0.4 = 1 − ⇒ TH = 500K
TH For efficiency 60%
∴ New efficiency, η' = 60% 60 300
=1–
TL 100 TH
∴η' = 1 − 300 60
TH =1–
300 TH 100
0.6 = 1 − ⇒ TH' = 750K 300 40
TH' =
Increase in source temperature, TH 100
∆TH' = TH' − TH = 750 − 500 300 × 100
TH =
= 250 K 40
TH = 750 K
569. A refrigerator with coefficient of performance
0.25 releases 250 J of heat to a hot reservoir. 571. A Carnot engine having an efficiency 1/5 as a
heat engine, is used as a refrigerator. If the
The work done on the working substance is –––
work done on the system is 50 J, the amount of
100 energy absorbed from the reservoir at lower
(a) J (b) 150 J
3 temperature is ______
(c) 200 J (d) 50 J (a) 90 J (b) 99 J
AP EAMCET-19.08.2021, Shift-II (c) 200 J (d) 1 J
Ans. (c) : Given, coefficient of performance (COP) = AP EAMCET-25.08.2021, Shift-II
0.25 1 Work done
Ans. (c) : Given, η = =
For a refrigerator, 5 Heat input
Work done = 50 Joule
We know, the efficiency of Carnot engine
Q – QL
Qcold η= H
0.25 = QH
250 − Q cold
Coefficient of performance of a refrigerator,
Qcold = (250 × 0.25) – 0.25 Qcold
1– η
62.5 α=
Qcold = η
1.25
1
Qcold = 50 J 1–
W = Qhot – Qcold α= 5 = 4×5
1/ 5 5 1
W = (250 – 50)
α=4
W = 200 J
Q
570. A Carnot engine whose efficiency is 40% takes Also, α = L (Where, W is the work done)
in heat from source maintained at a W
temperature of 500 K. It is desired to have an QL = α × W
engine of efficiency 60%. Then, the intake QL = 4 × 50
temperature for the same exhaust (sink) QL = 200 Joule
temperature must be ___ 572. Three designs are proposed for an engine
(a) 1200 K (b) 750 K which is to operate between 500 K and 300 K.
(c) 600 K (d) 800 K Design A claims to produce 150 J of work per
AP EAMCET-25.08.2021, Shift-II 1000 J of heat input, design B 450 J of work per
1000 J and design C 300 J of work per 1000 J.
Ans. (b) : For Carnot engine, the efficiency is given by
Which of the designs would you choose?
T
η = 1– L
TH (a) A (b) B
Where, TL = Low temperature (sink) (c) C (d) None is suitable
TH = High temperature (source) = 500K AP EAMCET-25.08.2021, Shift-II

700
Ans. (c) : Given that, T1 = 500 K, T2 = 300 K Ans. (c) : Given,
T Three Carnot engines operate in series,
Efficiency of the engine (η) = 1 – L
TH
300 2
=1– = = 0.4
500 5
For design A,
Work done = 150 J, Heat input = 1000 J
Work done
ηA =
Heat input
150
ηA = = 0.15
1000
For design B,
Work done = 450 J, Heat input = 1000 J T1 > T2 > T3 > T4
450 ∴ The three engines are equally efficient
ηB = = 0.45 Therefore,
1000
For design C, η1 = η2 = η3
Work done = 300 J, Heat input = 1000 J T2 T T
1– = 1– 3 = 1– 4
300 T1 T2 T3
ηC = = 0.3
1000 T2 T3 T4
= =
Design B has maximum efficiency and our priority is to T1 T2 T3
select engine with maximum efficiency but since B is Take,
an impossible engine so the next engine to be selected is T2 T3
C. =
T1 T2
Note: Official answer given by commission is (b).
( T2 ) = T1T3
2
573. Three Carnot engines operate in series between
a heat source at temperature T1 and heat sink T2 = T1T3 ……(i)
at a temperature T4. There are two other
Again take,
reservoirs at temperatures T2 and T3. The three
engines are equally efficient if ____ T3 T4
=
(given that T1 > T2 > T3 > T4) T2 T3
( T3 )
2
= T2 .T4

T3 = T2 .T4 …… (ii)

Put the value T3 = T2 .T4 in equation (i), we get


Then,
T2 = T1 T2 .T4
1 1 1
T2 = ( T1 ) 2
( T2 ) 4 ( T4 ) 4

1
1– 1 1
T2 4
= ( T1 ) 2
( T4 ) 4

3 1 1
T2 4
= ( T1 ) 2
( T4 ) 4

( )( )
4 4
1 1
(a) 1/2 2
T2 = (T1.T4) & T3 = (T1 .T4) 1/3 T2 = ( T1 ) 2 3
( T4 ) 4 3

(b) T2 = (T13.T4)1/4 & T3 = (T1.T43)1/4


T2 = ( T12 )
1 1

(c) T2 = (T12.T4)1/3 & T3 = (T1.T42)1/3


3
( T4 ) 3
(d) T2 = (T1.T42)1/3 & T3 = (T12.T4)1/3 T2 = ( T12 .T4 )
1
3

AP EAMCET-23.08.2021, Shift-I
701
Again put the value T2 = T1 .T3 ( ) is equation (ii) 576. A Carnot refrigerator extracts 35.0 kJ as heat
during each cycle, operation with a coefficient
of performance of 4.60. Find the energy per
T3 = T4 . T1.T3 cycle transferred as heat to the surroundings.
(a) 42.6 kJ (b) 53.2 kJ
T3 = ( T4 ) ( T1 ) 4 ( T3 )
1 1 1
2 4
(c) 63.9 kJ (d) 72.5 kJ
1 AMU-2011
1–
= ( T1 ) ( T4 )
1 1
4 4 2
T3
Ans. (a) : Given, Heat extracts (QL) = 35 kJ , α = 4.60
3 1 1
T3 4
= ( T1 ) 4
( T4 ) 2 Coefficient transferred as heat to surrounding
 1
( ) ((T ) ) Q H = Q L 1 + 
4 4
T3 = ( T1 )
1 3 1 3
4
4
2  α 
 1 
= 351 +
(T )
1
T3 = ( T1 )
1
 4.60 
3 2 3
4

T3 = ( T1.T4 2 3 )
1 = 42.6 kJ
577. An ideal Carnot engine working with source
574. An electric refrigerator extracts 2000 calories temperature T1 and sink temperature T2, has
from ice trays. If the coefficient of performance T
efficiency η. Then the value of the ratio 1 is
is 5, then the work done by the motor in ______ T2
calories
1 1− η
(a) 5 (b) 400 (a) (b)
(c) 2000 (d) 10000 1− η 1
AP EAMCET-05.10.2021, Shift-I 1
(c) (d) η
Ans. (b) : Given, heat extract = 2000 cal η
Heat extracted η
Coefficient of performance = (e)
Work done by motor 1− η
2000 Kerala CEE 04.07.2022
5= Ans. (a) : As we know,
W
2000 Work done
W= Carnot efficiency =
5 Heat Applied
W = 400 calories T1 − T2 T2
η= = 1−
575. A Carnot engine whose low-temperature T1 T1
reservoir is at 27 oC, has an efficiency 37.5%. T2
The high temperature reservoir is at ______ = 1− η
(a) 480 oC (b) 327 oC T1
(c) 307 oC (d) 207 oC T1
=
1
AP EAMCET-05.10.2021, Shift-I T2 1 − η
Ans. (d) : Given, T2 = 27 + 273 = 300 K and efficiency 578. If the door of a refrigerator in a room is kept
37.5 open, the temperature of room will be:
(η) = 37.5% = = 0.375 (a) Increase (b) Decrease
100
From Carnot equation of efficiency, (c) Remain constant (d) Uncertain
SRMJEEE-2019
T
η = 1− 2 Ans. (a) : When the door of a refrigerator in a room is
T1 open, temperature of refrigerator will increase and
300 thermostat will try to cool it down and during this
0.375 = 1 – process room temperature will increase.
T1
579. A domestic refrigerator is loaded with food and
300 the door closed. During a certain period the
= 1 – 0.375 machine consumes 1 kWH of energy and the
T1
internal energy of the system drops by 5000 kJ.
300 The quantity of heat transferred from the
= 0.625
T1 system to surrounding will be
(a) –8.6 MJ (b) 86 MJ
300
T1 = = 480 K = 480 – 273 = 207°C (c) –86 MJ (d) –8.6 KJ
0.625 SRMJEEE-2019
702
Ans. (a) : Given, (a) The efficiency of the Carnot cycle is highest
Power consumed regarded as work consumed which is for case-I
taken as negative by sign convention (b) The efficiency of the Carnot cycle is highest
∴ W = –1 kWH = –1 × 3600 kJ = –3600 kJ for case-II
∆U = –5000 kJ (c) The efficiency of the Carnot cycle is highest
Applying first law of thermodynamics, for case-III
Q – W = ∆U (d) The efficiency of case-II is higher than case-
III
Q – (–3600) = –5000
(e) The efficiency of the Carnot cycle is same for
∴ Q = –5000 – 3600
all three case
Q = –8600 kJ
Q = –8.6 MJ Kerala CEE-2019
580. The expression for the efficiency of a Carnot's Ans. (c) : Efficiency of Carnot engine-
engine is T −T
η= 1 2
(a) 1– (T1/T2) (b) 1 – T T1
(c) (T2/T1) – 1 (d) 1 – (T2/T1) In case I,
SRMJEEE-2017
ηI = 1
( T + ∆T ) − T2
Ans. (d) : The expression for the efficiency of a
T1 + ∆T
 T 
Carnot's engine is 1 − 2  . ( T − T ) + ∆T
ηI = 1 2
 T1 
T1 + ∆T
Carnot engine is a theoretical thermodynamic cycle
proposed by Leonard Carnot. In case II,
581. Inside the engine of an automobile, the cylinder T − ( T2 + ∆T ) ( T1 − T2 ) − ∆T
ηII = 1 =
compresses the air from approximately T1 T1
standard temperature and pressure to one- In case III,
twentieth of the original volume and a pressure
of 50 atm. What is the temperature of the T − ( T2 − ∆T ) ( T1 − T2 ) + ∆T
ηIII = 1 =
compressed air? T1 T1
(a) 500 K (b) 682 K So, From above efficiency we can conclude the order
(c) 550 K (d) 1000 K as-
(e) 200 K ηIII > ηII > ηI
Kerala CEE-2019 583. An ideal mono atomic gas is taken round the
Ans. (b) : Given, P1 = 1atm, P2 = 50atm, V1 = V, cycle ABCDA as shown in figure. The work
V done during the cycle is :
V2 = & T1 = 273K,T2 = ?
20
From ideal gas equation-
PV= n RT
Taking ratio for two condition,
P1V1 T1
=
P2 V2 T2
1× V 273 20 273 273 × 5
= ⇒ = ⇒ T2 = (a) PV (b) 2PV
V T2 50 T2 2
50 × PV
20 (c) (d) zero
T2 = 682K 2
IIT 1983
582. A Carnot engine is operating between a hot
UPCPMT 1990
body and cold body maintained at
temperatures T1 and T2 respectively. Consider UPCPMT 1991
the following three cases BHU 1994
Case-I : The temperature of the hot body is AMU1995
changed to T1+∆T and cold body is at T2 UPSEE 2001,2019
Case-II : The temperature of the hot body is at BCECE-2003
T1 and cold body is changed to T2+∆T Karnataka CET-2003,2005
Case-III : The temperature of the hot body is Odisha JEE 2009
T1 and cold body is changed to T2–∆T NEET 2013
703
Ans. (a) : Work done in a process is equal to the area (c) As the refrigerator works, β increases due to
under the curve P – V diagram the formation of ice
(d) It is necessary to defrost the refrigerator to
increases the value of β
J&K-CET-2017
Ans. (a) : Refrigerator is a device which can extract
heat from low temperature reservoir TL and transfer it to
high temperature reservoir TH its cop is
Work done during the cycle = area of ABCDA TL
W = BA × DA COP =
TH – TL
= (2P – P) × (2V – V)
= PV COP of refrigerator is denoted by α is higher when the
584. When the gas expands with temperature using difference in temperature between the inside and
the relation V = KT2/3 for the temperature outside regions is smaller α is independent of formation
change of 40 K, the work done is of ice.
(a) 20.1 R (b) 30.2 R 1
(c) 26.6 R (d) 35.6 R 586. Consider a reversible engine of efficiency .
6
J&K-CET-2014 When the temperature of the sink is reduced by
Ans. (c) : Given that, 62oC, its efficiency gets doubled. The
V = KT2/3 temperature of the source and sink respectively
–1
2 3 are.
dV = K T dT
3 (a) 372 K and 310 K (b) 273 K and 300 K
Temperature change (∆T) = 40K (c) 99ºC and 10ºC (d) 200ºC and 37ºC
Work done, (dW) = PdV TS EAMCET (Engg.)-2017
From ideal gas equation, AIPMT-2007
PV = nRT
Ans. (a) : Given that,
nRT nRT
P= = 1
V 2 Initial efficiency of reversible engine =
KT 3 6
dW = P. ∆V Decreasing temperature = 62°C
–1
nRT 2 1 1
dW = 2/3
. K T 3 dT Final efficiency = 2 × = ]
KT 3 6 3
2nR  1– 3 – 3 
1 2 T − TL
= T  dT As we know that, η = H
3  TH

2 40 1 T
= nR ∫ dT (∵ n = 1) =1– L …..(i)
3 0 6 TH
2 According to question,
∴ dW = R[T]040
3 1 ( T − 62 )
=1– L
2
= R ( 40 – 0 ) 3 TH
3
80 T – TL + 62
= R = H
3 TH
= 26.66R 1 TH – TL 62
= +
dW = 26.67R 3 TH TH
585. Pick out the correct statement from the From equation (i), we get
following
(a) The β of a refrigerator is higher when the 1 = 1 + 62
difference in temperature between the inside 3 6 TH
and outside regions is smaller 1 1 62
(b) The value of β may be much higher than 3 – 6 = T
H
100%

704
1 62
=
6 TH
TH = 372 K
Putting the value of TH in equation (i), we get
1 T
=1– L
6 372
TL 5
=
372 6
5 × 372
TL =
6
TL = 5 × 62 Work done for engine A = Work done for engine B
TL = 310K WA = WB
So, 800 – T = T – 300
587. A Carnot engine working between 200 K and
2T = 300 + 800
500 K has work done equal to 800 Joules.
2T = 1100
Amount of heat energy supplied to the engine
1100
from the source is T=
2
4000 2000
(a) J (b) J T = 550K
3 3 589. 300 cal. of heat is given to a heat engine and it
800 1600 rejects 225 cal. of heat. If source temperature is
(c) J (d) J
3 3 227°C, then the temperature of sink will be –––
°C.
TS EAMCET(Medical)-2015
JEE Main-29.06.2022, Shift-I
Ans. (a) : Given that, Ans. (102) : Given that,
Initial temperature (TL) = 200K Heat to engine (Q1) = 300 cal
Final temperature, (TH) = 500 K Rejected heat (Q2) = 225 cal
Work done = 800 Joule Source temperature (T1) = 227 + 273 = 500K
T – TL Work done Q T
Efficiency, η = H = Efficiency of engine (η) = 1 – 2 = 1 − 2
TH Heat supply Q1 T1
500 – 200 800 Q 2 T2
= =
500 Heat supply Q1 T1
225 T2
300
=
800 =
500 Heat supply 300 500
500 × 225
800 × 5 T2 = = 375 K
Heat supply = 300
3 = (375 – 273)ºC
4000 T2= 102°C
=
3 590. A Carnot engine has efficiency of 50%. If the
588. Two Carnot engines A and B are connected in temperature of sink is reduced by 40oC, its
series in such a way that the work outputs are efficiency increases by 30%. The temperature
of the source will be:
equal when the temperatures of hot and cold
(a) 166.7 K (b) 255.1 K
reservoirs of A are 800 K and T and engine B
(c) 266.7 K (d) 367.7 K
are T and 300 K respectively. Then the
JEE Main-28.07.2022, Shift-I
temperature T is
Ans. (c) : Given that,
(a) 400 K (b) 450 K
Initial efficiency (η1) = 50% = 0.5
(c) 500 K (d) 550 K Reduced temperature = 40°C
TS EAMCET (Medical)-02.05.2018, Shift-I Increased efficiency = 30%
Ans. (d) : Given, Initial temperature of A = 800 K New efficiency (η2) = η1 × 1.3 = 0.5 × 1.3
Final temperature of B = 300 K η2 = 0.65

705
Initial efficiency, First case,
T 100 2
η1 = 1 – L ηI = 1 − =
TH 300 3
Second case,
TL
0.5 = 1 – ηII = ηI + ηII – ηI. ηII
TH
 200   100   200   100 
TL 1 = 1 −  + 1 −  − 1 −  1 − 
=  300   200   300   200 
TH 2
 1   1   1  1 
TH = 2TL ….(i) =   +   −   
 3   2   3  2 
New efficiency,
1 1 1
η2 = 1 – L
( T – 40 ) = + −
3 2 6
TH
2 + 3 −1 4 2
( TL – 40 ) = = =
0.65 = 1 – 6 6 3
TH Therefore, η(first case) = η(second case)

0.35 = L
( T – 40 ) 592. In a Carnot engine, the temperature of
TH reservoir is 527oC and that of sink is 200 K. If
the work done by the engine when it transfers
0.35 × 2TL = TL – 40 (∵ TH = 2TL) heat from reservoir to sink is 12000 kJ. The
0.70 TL = TL– 40 quantity of heat absorbed by the engine from
40 reservoir is ___ × 106J.
TL = JEE Main-27.06.2022, Shift-I
0.30
Ans. (16) : Given,
TL = 133.33K
Temperature of reservoir (Treservoir) = 527+273 = 800K
TH
∴ = 133.33 Temperature of sink (Tsink) = 200K
2 Work done by engine (W) = 12000 kJ = 12×106J
TH = 266.67 K Heat absorbed (Q) = ?
591. In 1st case, Carnot engine operates between T2 W
temperatures 300 K and 100 K. In 2nd case, as Efficiency of Carnot engine (η) = 1 − =
T1 Q1
shown in the figure, a combination of two
engines is used. The efficiency of this 200 12 × 106
combination (in 2nd case) will be: 1− =
800 Q1
600 12 × 106
=
800 Q1
3 12 × 106
=
4 Q1
Q1 = 16×106 J
593. The efficiency of a Carnot's engine, working
between steam point and ice point, will be:
(a) 26.81% (b) 37.81%
(c) 47.81% (d) 57.81%
JEE Main-26.06.2022, Shift-I
(a) same as the 1st case Ans. (a) : According to question-
st
(b) always greater than the 1 case Steam point (T1) = 100°C = 373 K
Ice point (T2) = 0°C = 273 K
(c) always less than the 1st case
We know that, steam point and ice point for water are
(d) may increase or decrease with respect to the
100°C and 0°C respectively.
1st case
JEE Main-27.07.2022, Shift-II  T 
η = 1 − 2  × 100
Ans. (a) : Efficiency of Carnot engine,  T1 
T  273 
η = 1– Lower = 1 −  ×100
THigher  373 

706
100 25 300
η= ×100 = 1−
373 100 Tsource
η = 26.81% 1 300
= 1−
Hence, the efficiency of the Carnot engine working 4 Tsource
between the ice point and the steam point would be
1 300
26.81%. 1− =
594. A heat engine operates with the cold reservoir 4 T source

at temperature 324 K. 3 300


=
The minimum temperature of the hot 4 Tsource
reservoir, if the heat engine takes 300 J heat
from the hot reservoir and delivers 180 J heat Tsource = 400K = 127°C
to the cold reservoir per cycle, is ––––––– K. According to question, if efficiency increased by 100%
JEE Main-26.06.2022, Shift-II then new efficiency,
Ans. (540) : Given, TL = 324 K, Q1 = 300 J, Q2 = 180 J η' = 50%
TH = ? 300
We know, Carnot efficiency, 1− = 0.5
Tsource
 T   Q 
η = 1 − L  = 1 − 2  1 − 0.5 =
300
 TH   Q1  Tsource
324 300 − 180
1− = 5
1− =
300
TH 300 10 Tsource
324 120
1− = 5
=
300
TH 300 10 Tsource
120 324 Tsource = 600 K = 327°C
1− =
300 TH Increase in temperature = 327°C–127°C
180 324 = 200°C
=
300 TH 596. A Carnot engine takes 5000 kcal of heat from a
reservoir at 727ºC and gives heat to a sink at
3 324
= 127ºC. The work done by the engine is
5 TH
(a) 3×106J (b) Zero
5 × 324 (c) 12.6×106J (d) 8.4×106J
TH =
3 JEE Main-24.06.2022, Shift-II
TH = 5×108 Ans. (c) : Given,
TH = 540 K Q1 = 5000 kcal = 5000 × 103 cal = 5×106 cal
595. A Carnot engine whose heat sinks at 27ºC, has T1 = 727°C = 727+273 = 1000 K
an efficiency of 25%. By how many degrees T2 = 127°C = 127+273 = 400 K
should the temperature of the source be W T
changed to increase the efficiency by 100% of = 1− 2
Q1 T1
the original efficiency?
(a) Increases by 18ºC (b) Increases by 200ºC W 400
= 1−
(c) Increases by 120ºC (d) Increases by 73ºC 5 × 106 1000
JEE Main-24.06.2022, Shift-I W 600
=
Ans. (b) : Given, 5 × 106 1000
η = 25% W 6
Tsink = 27°C = 273+27 = 300K =
5 × 106 10
Efficiency is given by,
6
T W = × 5 × 106
η = 1 − sink 10
Tsource W = 3×106 cal
300 W = 3×106 × 4.2 J
25% = 1 −
Tsource W = 12.6 × 106 J

707
597. Let η1 is the efficiency of an engine at T1 = 447° Desired effect Q1
C and T2 = 147°C While η2 is the efficiency at (COP)Refrigerator = Work input = W
η
T1 = 947°C and T2 = 47°C, the ratio 1 will be: T2 280 280
η2 = = = = 14
T1 − T2 300 − 280 20
(a) 0.41 (b) 0.56
(c) 0.73 (d) 0.70 Now,
JEE Main-25.07.2022, Shift-II Q1 = Q2 + W R
Ans. (b) : Given, Q1 = Q2 + 1 [∵ WR = 1J ]
T1 = 447°C = 447+273 = 720K
T2 = 147°C = 147+273 = 420K Q1–Q2 = 1
We know that efficiency of an engine is,
T Now,
η = 1− 2
T1 Q2 Q
COP = = 2
So, Q 1 − Q 2 1
T 420 720 − 420 300 Q
η1 = 1 − 2 = 1 − = = 14 = 2 ⇒ Q 2 = 14
T1 720 720 720 1
Thus, for η2 ∴ Q1 = Q2+1 = 14+1 = 15 J
T2 599. A heat engine operates between a cold
η2 = 1 −
T1 reservoir at temperature 300K and a hot
Here, T1 = 947°C = 947+273 = 1220K reservoir at temperature T1 K. It takes 200 J of
T2 = 47°C = 273+47 = 320K heat from the hot reservoir and delivers 120 J
320 1220 − 320 900 of heat to the cold reservoir in a cycle. The
η2 = 1 − = =
1220 1220 1220 minimum temperature (T1) of the hot reservoir
η is
Asked 1 = ? (a) 450K (b) 400K
η2
(c) 500K (d) 350K
300
η1 300 1220 1220 AP EAMCET-11.07.2022, Shift-II
∴ = 720 = × = = 0.56
η2 900 720 900 2160 Ans. (c) : Given data,
1220 T1 = ?, Q1 = 200J
η1 T2 = 300K, Q2 = 120J
= 0.56
η2 Work done (W) = Q1–Q2 = 200–120 = 80J
598. Heat is flowing from a refrigerator whose W 80
∵ η= = = 0.40
inside temperature is 280 K to a room at 300 K. Q1 200
Then the amount of heat delivered to the room
for each joule of electrical energy consumed in T 300
For maximum efficiency ( ηmax ) = 1 − 2 = 1 −
joules is T1 T1
(a) 12 (b) 7
(c) 15 (d) 30 η ≤ ηmax
AP EAMCET (Medical)-24.04.2019, Shift-I  300 
Ans. (c) : Given, 0.40 ≤  1 − 
 T1 
T1 = 300 K
T2 = 280 K 300
≤ 1 − 0.4
T1
300
≤ 0.6
T1
300
T1 ≥
0.6
Q1= Heat taken from cold reservoir T1 ≥ 500K
Q2= Heat transfered to room So, minimum temperature (ηmin) = 500K

708
600. A Carnot engine operates between a source and Ans. (b) : Given, T2 = 300 K, T1 = 400 K, W = 400 J
a sink. The efficiency of the engine is 40% and T
the temperature of the sink is 27ºC. If the Efficiency (η) = 1 − 2
efficiency is to be increased to 50% then the T1
temperature of the source must be increased by 300
(a) 80 K (b) 120 K = 1−
400
(c) 100 K (d) 160 K
AP EAMCET-04.07.2022, Shift-I 100
η=
Ans. (c) : Given, η1= 40% 400
Sink temperature (T2) = 27°C = 27+273 = 300K 1
From Carnot equation of efficiency, η=
4
T2
η = 1− W
T1 ∵ η=
Q
For first efficiency,
1 W
T2 = [ ∵ W = 400 J]
η1 = 1 − 4 Q
T1
1 400
300 =
40% = 1 − 4 Q
T1
Q = 4 × 400 = 1600 J
40 300
= 1− Heat exhausted = 1600–400 = 1200 J
100 T1
602. A Carnot engine operates between heat
300 40
= 1− reservoirs differing in temperature by 80ºC.
T1 100 The efficiency of the Carnot engine is 20%. The
300 60 temperature of the cold reservoir is
=
T1 100 (a) 440 K (b) 400 K
T1 = 500 K (c) 250 K (d) 320 K
Thus, AP EAMCET-11.07.2022, Shift-I
For second efficiency first efficiency is to be Ans. (d) : Given,
increased to 50% then, TH – TL = 80 K = 80ºC ....(i)
T2 η η
η2 = 1 − = 20% or = 0.2
T1 ' Efficiency of Carnot engine,
T2 T
50% = 1 − η = 1− L
T1 ' TH
50 300 T − TL
= 1− η= H
100 T1 ' T
H
1 300
= 1− 0.2 =
80
2 T1 ' TH
1 300
1− = 80
2 T1 ' TH = = 400
0.2
1 300 TH = 400 K
=
2 T1 ' From equation (i),
T1 ' = 600 TH –TL = 80K
The required increased in temperature of the source is, 400–TL = 80K
T'–T = 600–500 = 100 K TL = 400 – 80K
601. The work done by a Carnot engine operating TL = 320 K
between 300 K and 400 K is 400 J. The energy
exhausted by the engine is 603. A gas in a closed container undergoes the cycle
(a) 800 J (b) 1200J ABCDA as shown in the figure. The net heat
(c) 400 J (d) 1600J absorbed by the gas after it has completed 20
AP EAMCET-05.07.2022, Shift-I cycles is

709
Ans. (a) :

(a) +5.0kJ (b) -5.0kJ Latent heat of fusion of ice = 330 J/gm
(c) +2.5kJ (d) -2.5kJ L = 330 × 1000 J/kg
AP EAMCET-11.07.2022, Shift-I Since, we know that
Ans. (a) : T Q
Efficiency of Carnot cycle (η) = 1 − L = 1 − R
TH QA
TL Q R
=
TH Q A
330 Q R
=
430 Q A
43
QA = QR
33
∵ QA = QR+W
43
Since we know that area under the curve of P-V ∴ Q R = Q R + 60kJ
diagram gives us work done. 33
So,  43 − 33 
QR   = 60kJ
Work done in one complete cycle =  33 
Area under the curve on P-V diagram 60 × 33
QR = = 6 × 33kJ
1 10
dW = [10 + 15] × 20
2 Q R = 6 × 33kJ
1 Q = ML
dW = [ 25 × 20] = 250 Joule

2 kJ
Since we know that for a cyclic process net heat transfer ∴ 6×33 = m×330
kg
is equal to net work transfer.
6
m=
∫ dQ = ∫ dW 10
kg

∴ dQ = dW = 250J [for one cycle] m = 0.6kg


And net heat transfer for 20 cycle, 605. A Carnot's engine is working between a source
dQ = 250×20J at constant temperature T1 and a sink at
dQ = 5000 J 1
constant temperature T2 has efficiency as .
dQ = +5kJ 8
604. A Carnot engine operating between 430 K and Upon decreasing the temperature of sink by
330 K does a work 60 kJ. The amount of ice, 1
50ºC, the efficiency becomes . Then the
that can melt from its exhaust is 4
values of T1 and T2 are
(Latent heat of fusion of ice = 330 J g–1)
(a) T1 = 127ºC, T2 = 77ºC
(a) 0.6 kg (b) 0.75 kg
(b) T1 = 400ºC, T2 = 315ºC
(c) 1.2 kg (d) 0.4 kg (c) T1 = 215ºC, T2 = 100ºC
AP EAMCET-12.07.2022, Shift-I (d) T1 = 100ºC, T2 = 215ºC
AP EAMCET(Medical)-2016, 2014 AP EAMCET-12.07.2022, Shift-I
710
Ans. (a) : 250
For Ist condition, 0.25 = 1 −
TH
TH = 333.33K
T
and η2 = 1 − L
TH '
250
0.50 = 1 −
TH′
1
η= TH' = 500K
8
So, temperature of sink increase by-
T2
η =1− ∆T = TH' – TH
T1
∆T = 500 – 333.33
T2 1 7 ∆T = 166.7K
= 1− =
T1 8 8 1
∆T = × 103 K
T2 7 6
= ..…(i)
T1 8 607. A Carnot engine develops 100 hp and operates
For 2nd condition, between 300 K and 500 K. Find its thermal
efficiency.
(a) 25% (b) 35%
(c) 40% (d) 38%
AP EAMCET-24.09.2020, Shift-II
Ans. (c) : Given,
TL = 300K, TH = 500K, η = ?
1
η = , T2' = (T2 − 50)K Thermal efficiency is given as:
4 T
T2' 1 3 η = 1− L
= 1− = TH
T1 4 4
300
T2 − 50 3 T 50 3 η = 1−
= ⇒ 2− = [from eqn. (i)] 500
T1 4 T1 T1 4
500 − 300 200
7 50 3 50 1 η= =
− = ⇒ = ⇒ T1 = 400K 500 500
8 T1 4 T1 8
2
Putting the value of T1 in equation (i), we get η=
5
T2 7
= ⇒ T2 = 350K η = 0.4
400 8
or η = 40%
∵ Values of T1 and T2 are 400 K (127°C) and 350 K
608. A Carnot engine takes heat from a source at
(77°C). 627 °C and rejects heat to sink at 27 °C. In its
606. The temperature of the sink of a Carnot engine 10 cycles of operation, it rejects 600 J of heat
is 250K. In order to increase the efficiency of energy to the sink. The heat absorbed per cycle
the Carnot engine from 25% to 50%, the of operation is
temperature of the source should be increased (a) 6000 J (b) 1800 J
by
(c) 180 J (d) 1333 J
1 1
(a) × 103 K (b) × 103 K AP EAMCET (Medical)-07.10.2020, Shift-I
3 2 Ans. (c) : Given,
1
(c) 200 K (d) × 103 K TL = 27°C = 27°C + 273
6 TL = 300 K
AP EAMCET-04.07.2022, Shift-II TH = 627°C = 627+273 = 900 K
T TH = 900K
Ans. (d) : Efficiency of Carnot engine (η) = 1 − L
TH Heat rejected per cycle of operation,
η1 = 25%, η2 = 50%, TL = 250K 600
QR = J
T 10
η1 = 1 − L
TH QR = 60 J

711
Let heat absorbed per cycle of operation be QA. Q
T Q  
η=  
∵ Efficiency of Carnot cycle (η) = 1 − L = 1 − R 6
TH QA Q
TL Q R 1
∴ = η=
TH Q A 6
300 60 Let the temperature of sink and source be TL and TH.
∴ = T
900 Q A Then efficiency of engine (η) = 1 − L
QA = 60×3 J TH
Q A = 180 J 1 T
= 1− L
609. A thermodynamic system is taken around the 6 TH
cycle ABCDA and the P – V diagram for the TL 1 5
whole process is as shown in the diagram. The = 1− =
TH 6 6
work done during the cycle is
TL 5
= ....(i)
TH 6
When temperature of sink is reduced then,
TL' = TL–62°C and efficiency η becomes twice
T '
∴ 2×η = 1 − L
TH
1 (T − 62)
2× = 1− L
6 TH
1 (T − 62)
= 1− L
3 TH
 TL − 62  2
 =
(a) 12 PV (b) 120 PV  TH  3
(c) 20 PV (d) 60 PV TL 62 2
− = ....(ii)
AP EAMCET (Medical)-07.10.2020, Shift-I TH TH 3
Ans. (b) : Since we know that area under the curve on From equation (i) and (ii), we get
P-V diagram gives us work transfer.
5 62 2
So, net work transfer for the given thermodynamic − =
system is equal to area under the curve. 6 TH 3
dW = Area of rectangle = AD × DC 62 5 2
dW = (25P–5P) × (10V–4V) = −
TH 6 3
dW = 20P × 6V
dW = 120 PV 62 1
=
TH 6
610. A reversible engine converts one-sixth of the
heat supplied into work. When the temperature TH = 6 × 62
of the sink is reduced by 62ºC, the efficiency of TH = 372K
the engine is doubled. The temperatures of the
sources and sink are TH =372–273 = 99°C
(a) 99ºC, 37ºC (b) 80ºC, 37ºC T 5
(c) 95ºC, 37ºC (d) 90ºC, 37ºC From equation (i), L =
TH 6
AP EAMCET -2011
AIPMT-2000 5
TL = × TH
Ans. (a) : Given, 6
Q 5
W = , Heat supplied = Q TL = × 372
6 6
Output energy TL = 5×62 = 310K
Efficiency of the engine (η) =
Input energy TL = 310–273 = 37°C

712
611. Two cylinders A and B fitted with pistons TL
contain equal number of moles of an ideal ∴ 2η = 1 −
3TH
monoatomic gas at 400 K. The piston of A is
free to move while that of B is held fixed. Same TL
= 1 − 2η
amount of heat energy is given to the gas in 3TH
each cylinder. If the rise in temperature of the TL
gas in A is 42 K, the rise in temperature of the = 3(1 − 2η) ....(ii)
gas in B is TH
(a) 21 K (b) 35 K From equation (i) and (ii), we get
(c) 42 K (d) 70 K (1–η) = 3(1–2η)
AP EAMCET -2009 1–η = 3 – 6η
Ans. (c) : From the first law of thermodynamics, 5η = 2
dQ = dU + dW 2
For a cylinder, pressure is constant. η = = 0.4
5
µR η = 0.4 = 40%
W= ( T1 – T2 )
γ –1
613. In the cyclic process shown in the P-V
For a monoatomic gases, diagram calculate the work done.
5
µ = 1, γ=
3
1× R
W= × ( 442 – 400 )
5 
 − 1
3 
3
= R × 42
2
W = 63R
For a cylinder A, (∆U) = 0
Q = 0 + 63 R 2 2
For a cylinder B, volume is constant.  V -V   P -P 
(a) π  2 1  (b) π  2 1 
W=0  2   2 
Q = µCv ∆T π
(c) ( P2 − P1 )( V2 − V1 ) (d) π(P2V2–P1V1)
For monoatomic gas, 4
3 AMU-2010
Cv = R
2 Ans. (c) : Work done in the process will be equal to the
As heat given to both cylinder is same. area under the curve.
3 Work done = Area under the curve
∴ 63R = R∆T
2 π
W = d1d 2 ____(i)
∆T = 42 K 4
612. On tripling the absolute temperature of the Where, d1 & d2 are the major and minor axis along x
source, the efficiency of a Carnot's heat engine and y axis respectively.
becomes double that of the initial efficiency. d1 = P 2 – P 1
Then the initial efficiency of the engine is _____ d 2 = V2 – V1
(a) 20 % (b) 50%
Putting the value of d1 & d2 in equation (i),
(c) 60% (d) 40%
π
AP EAMCET-26.04.2017, Shift-II W = ( P2 – P1 )( V2 – V1 )
4
Ans. (d) : Let the temperature of source and sink be TH
and TL respectively and efficiency is η. 614. A Carnot engine has an efficiency of only 15%.
T If it operates between constant temperature
Then, η = 1− L reservoirs differing in temperatures by 55ºC
TH the temperature of higher temperature
TL reservoir is
= 1− η ....(i)
TH (a) 367 ºK (b) 382 ºK
According to question, tripling the source temperature (c) 418 ºK (d) 421 ºK
efficiency of Carnot engine becomes double. AMU-2019

713
Ans. (a) : Efficiency of Carnot cycle, 373 − 273
=
T − TL 373
η= H
TH 100
= ×100
η = 15 % = 0.15 373
= 26.80
Difference in temperature of two reservoirs,
∆T = TH – TL = 55°C = 55 K. 1
617. A Carnot engine having an efficiency of as
55 10
∴ 0.15 = heat engine, is used as a refrigerator. If the
TH
work done on the system is 10 J, the amount of
55 energy absorbed from the reservoir at lower
TH = = 366.67 ≃ 367K temperature is
0.15
(a) 1 J (b) 90 J
615. A quarter horse power motor runs at a speed
(c) 99 J (d) 100 J
of 600 rpm. Assuming 40% efficiency, the work
done by the motor in one rotation will be
APEAMCET (Medical)- 24-04-2019 Shift- 1
(a) 7.46 J (b) 74.6 J
NEET -2017
(c) 7400 J (d) 7.46 erg
AIPMT 2015
AMU-2015 AIPMT- 2015
Ans. (a) : Given, AIEEE 2007
600 rev = 1min Ans. (b) : For a heat engine,
600 rev = 60 sec Work output
η=
60 Heat input
1 rev = sec
600
W  1
1 η= ∵ η =
1 rotation = sec Q  10
10
1 10
1 =
t= sec 10 Q1
10
Q1 = 100 J
1
∵ P = HP So, energy released at lower temperature,
4
Q2 = Q1 – W
E
P= Q 2 = 100 – 10
t Q 2 = 90 J
E=P×t×η 618. A refrigerator works between 4°C and 30°C. It
1  40 is required to remove 600 calories of heat every
=  HP  × t ×
4  100 second in order to keep the temperature of the
refrigerated space constant. The power
1 1 40
= × 746 × × required is (Take, 1 cal = 4.2 Joules)
4 10 100 (a) 23.65 W (b) 236.5 W
746 40 (c) 2356 W (d) 2.365 W
= × = 7.46
40 100 NEET-2016
°
616. The efficiency of an ideal heat engine working Ans. (b) : Given, T L = 4 C = 273 + 4 = 277K
between the freezing point and boiling point of TH = 30°C = 273 + 30 = 303K
water, is Desired effect TL
COP = =
(a) 6.25% (b) 20% Heat supply TH − TL
(c) 26.8% (d) 12.5% 277 277
NEET-2018 COP = = = 10.65
303 − 277 26
Ans. (c) : Given, Q
Freezing point of water = 0° = 273 + 0° = 273K ∵ COP = = 10.65
W
°
Boiling point of water = 100 C = 273 + 100 = 373K 600 × 4.2
Efficiency of engine, = = 10.65
W
T 600 × 4.2
η = 1– L W= = 236.61Watt
TH 10.65

714
619. The temperature inside a refrigerator is t2 °C 5 TH = 253 + 1265
and the room temperature is t1 °C. The amount 5TH = 1518
of heat delivered to the room for each joule of 1518
electrical energy consumed ideally will be TH =
5
t1 t + 273 TH = 303.6K
(a) (b) 1
t1 − t 2 t1 − t 2 TH = 30.6
t 2 + 273 t1 + t 2 TH ≃ 31°C
(c) (d)
t1 − t 2 t1 + 273 621. A Carnot engine whose sink is at 300 K has an
NEET-2016 efficiency of 40%. By how much should the
temperature of source be increased so as to
Ans. (b) : Coefficient of performance of refrigerator
increase its efficiency by 50% of original
efficiency ?
(a) 275 K (b) 325 K
(c) 250 K (d) 380 K
AIPMT-2006
Ans. (c) : Given, sink temperature (T2) = 300K, and
efficiency (η1) = 40%
T
η1 = 1 − 2
T1
40 300
= 1−
100 T1
40 −300
−1 =
100 T1
Qc
( COP )R = −60 −300
=
Wnet
100 T1
t 2 + 273
= T1 = 500 K
t1 – t 2 Let temperature of source be increases by T1 + x
QH = W + Qc if the efficiency (η) increase by 50% of original
Q H Qc efficiency
= +1
W W 40
η1 = 40% + × 50%
t + 273 100
= 2 +1
t1 − t 2 = 0.4 + 0.4 × 0.5
= 0.4 +0.2
QH t1 + 273
= = 0.6
W t1 – t 2 300
0.6 = 1 −
620. The coefficient of performance of a refrigerator 500 + x
is 5. If the temperature inside freezer is −20°C, −300
the temperature of the surroundings to which it 0.6 – 1 =
rejects heat is 500 + x
(a) 31°C (b) 41°C −300
−0.4 =
(c) 11°C (d) 21°C 500 + x
AIPMT-2015 0.4 × (500 + x) = 300
° 200 + 0.4x = 300
Ans. (a) : Given, TL = – 20 C = 273 – 20 = 253K
COP = 5 0.4x = 300 – 200
We know, 1000
x=
Desired effect TL 4
COP = =
Heat supplied TH − TL x = 250 K
253 622. The efficiency of Carnot engine is 50% and
5= temperature of sink is 500 K. If the
TH − 253 temperature of source is kept constant and its
5 × (TH – 253) = 253 efficiency is to be raised to 60%, then the
5 TH – 1265 = 253 required temperature of the sink will be

715
(a) 600 K (b) 500 K Ans. (a) :Process (AB) = Isobaric process it means
(c) 400 K (d) 100 K pressure is constant.
AIPMT-2002 Process (BC) = Isothermal expansion process, it means
Ans. (c) : Given, η = 50 % temperature is constant
TL = 500 K Process (CD) = Isochoric process, it means volume
The efficiency of the Carnot engine– constant and temperature decrease.
T Process (DA) = Isothermal compression, it means
η = 1− L temperature constant and volume reduced.
TH
624. A Carnot engine takes 300 calories of heat at
500
0.5 = 1 − 500 K and rejects 150 calories of heat to the
TH sink. The temperature of the sink is
500 (a) 1000 K (b) 750 K
0.5 =
TH (c) 500 K (d) 250 K
TH = 1000 K UPSEE 2019
If the temperature of source is kept constant and it Ans. (d) : Given,
efficiency is to be raised to 60% Heat reject (QL) = 150 cal
' T Heat takes (QH) = 300 cal
η = 1− L
TH Temperature of source (TH) = 500 K
TL Temperature of sink (TL) = ?
0.6 = 1 − Output
1000 Efficiency of engine =
TL Input
0.4 =
1000 QH TH
=
TL = 400 K Q H − Q L TH − TL
623. A cyclic process ABCD is shown below in the 300 500
=
given P-V diagram. In the following answers 300 − 150 500 − TL
the one that represents the same process as in
300 500
P-T diagram =
150 500 − TL
2 × (500 – TL) = 500
1000 – 2TL = 500
– 2TL = 500 – 1000
– 2TL = – 500
TL = 250 K
625. A Carnot engine works between 727ºC and
27ºC. The efficiency of the engine is
(a) 30 % (b) 70%
(c) 96% (d) 100%
UPSEE 2020
Ans. (b) : Given, TL = 27°C + 273 = 300 K
TH = 727°C + 273 = 1000 K
The efficiency of Carnot engine–
T
η = 1− L
TH
300
= 1−
1000
700
= × 100
BITSAT- 2008 1000
EAMCET-2006 η = 70%

716
626. When the temperature of the source of a Ans. (c) : In adiabatic process, the entropy change will
Carnot engine is at 400 K, its efficiency is 25%. be zero. There will be no heat transfer for adiabatic
The required increase in temperature of the process i.e. ∆Q = 0
source to increase the efficiency to 50% is ∆Q
(a) 800 K (b) 600 K ∆S = =0
T
(c) 100 K (d) 400 K
(e) 200 K 629. The theory of refrigerator is based on
Kerala CEE 2021 (a) Joule- Thomson effect
(b) Newtons' particle theory
1
Ans. (b) : Given, η= = 25% (c) Joule's effect
4 (d) None of the above
TH = 400 K CG PET- 2009
1
η' = 50% = Ans. (d) : The theory of refrigerator is based on second
2 law of thermodynamics.
TH = ? According to the second law of thermodynamics “it is
The efficiency of Carnot engine is– impossible for a self acting machine unless energy is
T provided by any external agency to transfer heat from a
η = 1– L
TH body having lower temperature to high temperature.
1 T 630. Even Carnot engine cannot give 100%
= 1− L efficiency because we cannot
4 400
(a) prevent radiation
3 TL
= (b) find ideal sources
4 400 (c) reach absolute zero temperature
400 × 3
TL = = 300 K (d) eliminate friction
4 CG PET- 2006
For 50% efficiency, Ans. (c) : We know that the efficiency of a Carnot
T engine is given as:
η' = 1− L
TH T
η=1– 2
1 300 T1
= 1−
2 TH For 100% efficiency i.e., η = 1, T2 must be equal 0 K. 0
1 300 K is the absolute zero temperature i.e. the lowest limit
= of the thermodynamic temperature scale. Absolute zero
2 TH
indicates the temperature at which the particles in a
TH = 600 K matter are essentially motionless, which is physically
impossible.
(K) Second Law of 631. When you make ice cubes, the entropy of water
thermodynamics and Entropy (a) does not change
(b) increases
627. During phase change, entropy (c) decreases
(a) remains constant (d) may either increase or decrease depending on
(b) always increases the process if used
(c) always decreases AIIMS-2006
(d) may increase or decrease COMEDK 2018
AP EAMCET (22.09.2020) Shift-II Ans. (c) : In a solid, molecules are closely packed and
Ans. (d) : The entropy of a system undergoing a phase well arranged. Thus, when the water is change from the
transition increases if the phase transition is towards liquid state to the solid state, the arrangement of
higher internal energy and decreases if the phase molecules is varying. The amount of entropy can be
transition is towards lower internal energy therefore defined as the measure of the molecule disorder
during phase change entropy of the system may increase otherwise called as randomness of a system. So, as the
or decrease. disorderness decreases from liquid state to the solid
628. For which of the following processes, entropy state, the entropy is also decreasing.
change is zero? 632. The change in the entropy of a 1 mole of an
(a) Isobaric (b) Isothermal ideal gas which went through an isothermal
(c) Adiabatic (d) Isochoric process from an initial state (P1,V1,T) to the
CG PET- 2017, 2013 final state (P2,V2,T) is equal to :
717
(a) Zero (b) R ln T ∆Q
∆S =
V V T
(c) R ln 1 (d) R ln 2
V2 V1 Where, ∆Q = Heat absorbed by the system
T = Absolute temperature
BCECE-2006
635. Entropy of the universe tends to be
Ans. (d) : Work done by n moles of the gas volume
(a) zero (b) maximum
change from V1 to V2,
(c) minimum (d) constant
V
W = nRT ln 2 (∵ n = 1) SRMJEEE - 2014
V1 Ans. (b) : Entropy of the universe increases, hence it
V2 tends to be maximum.
W = RT ln
V1 ∆Q
636. Find the ratio of in an isobaric process,
First law of thermodynamics, ∆ W
the ratio of molar specific heat capacities of the
∆Q = ∆U + W (for isothermal process, ∆U =
Cp
0) gas used is =γ
∆Q = W C v

From second law of thermodynamics, γ γ −1


(a) (b)
V2 γ −1 γ
RTln
∆Q V1 V γ +1 γ
∆S = = = Rln 2 (c) (d)
T T V1 γ γ +1
AP EAMCET-05.10.2021, Shift-I
633. If ‘∆Q’ is the amount of heat supplied to ‘n’
moles of a diatomic gas at constant pressure, Ans. (a) : We know that,
‘∆U’ is the change in internal energy and ‘∆W’ ∆Q = nC p ∆T ….(i)
is the work done, then ∆W : ∆U : ∆Q is and ∆U = nC v ∆T ….(ii)
Taking ratio of equation (i) and (ii), we get-
(a) 1 : 2 : 3 (b) 2 : 5 : 7
(c) 2 : 3 : 4 (d) 5 : 7 : 9 ∆Q C p
=
MHT-CET 2020 ∆U C v
Ans. (b) : We know, ∆Q

At constant pressure ∆U
∆Q = nCp ∆T ∆Q ∆Q 1
∵ = =
∆U = nCv∆T ∆W ∆Q–∆U 1– ∆U
W = P∆V = n R∆T ∆Q
∴ ∆Q: ∆U: W = Cp: Cv: R 1
=
For diatomic gas, f = 5 1
Gas is ideal and diatomic in nature. 1−
γ
∴ Cv =
fR 5R
= ∆Q γ
=
2 2 ∆W γ − 1
5R 637. No process is possible whose sole result is the
∵ Cp = C v + R = +R
2 transfer of heat from a colder object to a hotter
7R object. This is Clausius statement for
= (a) Zeroth law of thermodynamics
2 (b) First law of thermodynamics
∴ ∆Q : ∆U : W = 7 : 5 : 2 (c) Second law of thermodynamics
Hence, (d) Carnot’s theorem
W : ∆U : ∆Q = 2 : 5 : 7 (e) Principle of refrigeration
Kerala CEE 04.07.2022
634. A measure of degree of disorder of a system is
known as Ans. (c) : There are two statements of second law of
thermodynamics which are
(a) isobaric (b) isotropy
(1) Kelvin-Plank statement
(c) entropy (d) enthalpy (2) Clausius statement
SRMJEEE - 2011 Kelvin-Plank statement:
Ans. (c) : A measure of degree of disorderness of a It is impossible for a heat engine to produce a network
system is known as entropy. in a complete cycle if it exchange heat only with a
The change in entropy is given as: single reservoir.

718
Clausius’s statement: 641. The liquids at temperature 60ºC and 30ºC
It is impossible to design a device, which works on a respectively have masses in the ratio 3 : 4 and
cycle and produces no effect other than heat transfer their specific heats in the ratio 4 : 5. If the two
from a cold body to hot body. liquids are mixed, the resultant temperature is
638. Consider the following thermodynamical (a) 70ºC (b) 50ºC
variables (c) 40ºC (d) 41.25ºC
(1) Pressure (2) Internal Energy EAMCET-2000
(3) Volume (4) Temperature Ans. (d) : Given,
Out of these, the intensive variable (s) is (are) T2 = 60°C + 273 = 333 K
(a) 1 only (b) 1, 4 T1 = 30°C + 273 = 303 K
(c) 1, 2 (d) 1, 2, 4
m  C 
AMU-2018 Mass ratio  1  is 3 : 4 and specific heat ratio  1 
Ans. (b) : Intensive property– The property which is  m2   C2 
independent of mass. i. e. pressure, temperature. is 4 : 5.
Extensive property– The property which is dependent We know,
on mass. i.e volume, Internal energy, Entropy.etc. m1C1 ( 60 − T ) = m 2 C 2 ( T − 30 )
639. The second law of thermodynamics is
m1 C1 T − 30
concerned with the × =
(a) conservation of energy m 2 C 2 60 − T
(b) conservation of number of holes 3 4 T − 30
(c) transformation of heat energy into work × =
4 5 60 − T
(d) conservation of temperature 3 T − 30
AMU-2002 =
5 60 − T
Ans. (c) : Second Law of Thermodynamics– 180 – 3T = 5T – 150
According to 2nd law of thermodynamics complete
8T = 330
conversion of low grade energy (i.e. Heat) into high
grade energy (i.e. work) is impossible in a cycle. T = 41.25° C
640. A liquid of mass m and specific heat s is heated 642. The latent heat of ice is 80 Cal/gm. The change
to a temperature T. Another liquid of mass m/2 in entropy when 10 gram of ice at 0°C is
and specific heat 2s is heated to temperature converted into water of same temperature is
2T. If these two liquids are mixed, the resultant (a) 0.293 Cal/K (b) 2.93 Cal/K
temperature of the mixture will be (c) 80 Cal/K (d) 8 Cal/K
2 8 UPSEE 2019
(a) T (b) T
3 5 Ans. (b) : Given, Lice = 80 cal/gm
3 3 Mass of ice = 10 gm
(c) T (d) T
4 2 Heat required to convert in water,
EAMCET-1992 Q = ml
Ans. (d) : Given, = 10 gm × 80 cal/gm.
Mass of liquid (m1) = m = 800 cal.
Specific heat of liquid (C1) = s T = 0°C = 273 K
Initial temperature of liquid (T1) = T ∆Q
Mass of liquid (m2) = m/2 S=
T
Specific heat of liquid (C2) = 2s 800
Initial temperature of liquid (T2) = 2T S=
273
Let the resultant temperature of the mixture = Tm
S = 2.93 Cal / K
Heat lost by liquid 1 = Heat gained by liquid 2
m1C1 ( T1 − Tm ) = m 2C 2 ( Tm − T2 ) 643. The entropy remain constant in
(a) A cyclic process
m
m × s × ( T − Tm ) = × 2s × ( Tm − 2T ) (b) An isobaric process
2 (c) An isothermal process
T – Tm = Tm – 2T (d) An adiabatic process
T + 2T = 2 Tm UPSEE 2020
3T = 2 Tm Ans. (d) : The entropy remains constant in an adiabatic
3 process. In adiabatic process, no heat enter or leaves
Tm = T
2 from the system.

719

You might also like